documentq1

156
Q1. orthodeoxia is charecteristically seen in ;; 1 copd;; 2 chronic heart failure;; 3 pul embolism;; 4 hepato-pul syndrome ans=hepatopulmonary syn ref=harrison 17e chapter 295 The hepatopulmonary syndrome is characterized by platypnea and orthodeoxia, representing shortness of breath and oxygen desaturation that occur paradoxically upon assuming an upright position. Fall in arterial blood oxygen upon assuming the upright posture. Usually due to right-to-left cardiac or vascular shunting with a posturally induced fall in left sided pressure permitting a corresponding gradient across the shunt. An anatomical component in the form of an interatrial communication and a functional component that produces a deformity in the atrial septum and results in a redirection of shunt flow with the assumption of an upright posture. The former may be an atrial septal defect, a patent foramen ovale, or a fenestrated atrial septal aneurysm. The latter may be cardiac, such as pericardial effusion or constrictive pericarditis; pulmonary, such as emphysema, arteriovenous malformation, pneumonectomy, or amiodarone toxicity; abdominal, such as cirrhosis of the liver or ileus; or vascular, such as aortic aneurysm or elongation Q2. allosteric inhibition with atp affects;; 1 phosphofructokinase;; 2 phospho-eno-pyruvate;; 3 g-pd;; 4 pyruvate kinase A good example is the allosteric regulation of the glycolytic pathway. This catabolic pathway consumes glucose and produces ATP, NADH and pyruvate. A key step for the regulation of glycolysis is an early reaction in the pathway catalysed by phosphofructokinase-1 (PFK1). When ATP levels rise, ATP binds an allosteric site in PFK1 to decrease the rate of the enzyme reaction; glycolysis is inhibited and ATP production falls. This negative feedback control helps maintain a steady concentration of ATP in the cell. However, metabolic pathways are not just regulated through inhibition since enzyme activation is equally important. With respect to PFK1, fructose 2,6- bisphosphate and ADP are examples of metabolites that are allosteric activators. Studies on ATP effects on the allosteric kinetics shown by pyruvate kinase from Phycomyces blakesleeanus NRRL 1555 (-) are reported. Phosphoenolpyruvate showed an allosteric ATP- dependent substrate inhibition. The results supported the existence of spatially distinct catalytic binding sites and the inhibitory binding sites for phosphoenolpyruvate, and ATP showed opposite heterotropic effects with respect to these two types of binding site. With respect to Mg2+ ions, ATP caused a negative heterotropic effect. The global inhibitory effect of ATP was in agreement with the predictions postulated by the two-state concerted-symmetry model of Monod, Wyman and Changeux so Both PFK1 and phosphoenolpyruvate are affected by allosteric inhibition by ATP Pyruvate kinase is the third regulated enzyme of glycolysis. Like PFK, pyruvate kinase is regulated both by allosteric effectors and by covalent modification (phosphorylation). Pyruvate kinase is activated by F-1,6-BP in the liver, a second example of feedforward stimulation. ATP and alanine (a biosynthetic product of pyruvate) act as allosteric inhibitors of pyruvate kinase.

Upload: mayuresh-chaudhari

Post on 27-Nov-2014

248 views

Category:

Documents


0 download

TRANSCRIPT

Page 1: DocumentQ1

Q1. orthodeoxia is charecteristically seen in ;;

1 copd;;

2 chronic heart failure;;

3 pul embolism;;

4 hepato-pul syndrome

ans=hepatopulmonary syn

ref=harrison 17e chapter 295

The hepatopulmonary syndrome is characterized by platypnea and orthodeoxia, representing

shortness of breath and oxygen desaturation that occur paradoxically upon assuming an upright

position.

Fall in arterial blood oxygen upon assuming the upright posture. Usually due to right-to-left cardiac

or vascular shunting with a posturally induced fall in left sided pressure permitting a corresponding

gradient across the shunt.

An anatomical component in the form of an interatrial communication and a functional component

that produces a deformity in the atrial septum and results in a redirection of shunt flow with the

assumption of an upright posture. The former may be an atrial septal defect, a patent foramen ovale,

or a fenestrated atrial septal aneurysm. The latter may be cardiac, such as pericardial effusion or

constrictive pericarditis; pulmonary, such as emphysema, arteriovenous malformation,

pneumonectomy, or amiodarone toxicity; abdominal, such as cirrhosis of the liver or ileus; or

vascular, such as aortic aneurysm or elongation

Q2. allosteric inhibition with atp affects;;

1 phosphofructokinase;;

2 phospho-eno-pyruvate;;

3 g-pd;;

4 pyruvate kinase

A good example is the allosteric regulation of the glycolytic pathway. This catabolic pathway

consumes glucose and produces ATP, NADH and pyruvate. A key step for the regulation of glycolysis

is an early reaction in the pathway catalysed by phosphofructokinase-1 (PFK1). When ATP levels rise,

ATP binds an allosteric site in PFK1 to decrease the rate of the enzyme reaction; glycolysis is

inhibited and ATP production falls. This negative feedback control helps maintain a steady

concentration of ATP in the cell. However, metabolic pathways are not just regulated through

inhibition since enzyme activation is equally important. With respect to PFK1, fructose 2,6-

bisphosphate and ADP are examples of metabolites that are allosteric activators.

Studies on ATP effects on the allosteric kinetics shown by pyruvate kinase from Phycomyces

blakesleeanus NRRL 1555 (-) are reported. Phosphoenolpyruvate showed an allosteric ATP-

dependent substrate inhibition. The results supported the existence of spatially distinct catalytic

binding sites and the inhibitory binding sites for phosphoenolpyruvate, and ATP showed opposite

heterotropic effects with respect to these two types of binding site. With respect to Mg2+ ions, ATP

caused a negative heterotropic effect. The global inhibitory effect of ATP was in agreement with the

predictions postulated by the two-state concerted-symmetry model of Monod, Wyman and Changeux

so Both PFK1 and phosphoenolpyruvate are affected by allosteric inhibition by ATP

Pyruvate kinase is the third regulated enzyme of glycolysis. Like PFK, pyruvate kinase is regulated

both by allosteric effectors and by covalent modification (phosphorylation). Pyruvate kinase is

activated by F-1,6-BP in the liver, a second example of feedforward stimulation. ATP and alanine (a

biosynthetic product of pyruvate) act as allosteric inhibitors of pyruvate kinase.

Page 2: DocumentQ1

Phosphorylation of pyruvate kinase is regulated by blood glucose level, just like PFK. High glucagon

(low blood sugar) causes phosphorylation, which in this case renders the enzyme inactive.

Q3. WHAT IS THE MELANOCYTE -KERATINOCYTE RATIO;;

1 1-1;;

2 1-10;;

3 1-28;;

4 1-36

The melanocyte-keratinocyte ratio varies in different areas of the body and an average ratio of 1 :

36

answer is 1:36

Q4. NEOADJUVANT CHEMOTHERAPY IS USEFUL IN ALL OF THE FOLLOWING CANCERS, EXCEPT ;;

1 BREAST CARCINOMA;;

2 OSTEOSARCOMA;;

3 OVARIAN CARCINOMA;;

4 HEPATOCELLULAR CARCINOMA

Q5. TARDIEUS SPOTS ARE SEEN IN DEATH DUE TO ;;

1 HAEMORRHAGE;;

2 VASOVAGAL SHOCK;;

3 BRAIN TUMOR;;

4 OBSTRUCTION OF RESPIRATORY PASSAGES

ANS= OBSTRUCTION OF RESPIRATORY PASSAGES

Tardieu's spots= spots of ecchymosis under the pleura after death by suffocation.

Q6. luxury perfusion phenomena is seen in ;;

1 thrombolytic therapy;;

2 arf;;

3 encephalitis;;

4 renal artery stenosis

ANS=ENCEPHALITIS( specifically arboviral encephalitis)

The disease process in the CNS arises partly from direct neuronal infection and subsequent damage

and partly from edema, inflammation, and other indirect effects. The usual pathologic picture is one

of focal necrosis of neurons, inflammatory glial nodules, and perivascular lymphoid cuffing; the

severity and distribution of these abnormalities vary with the infecting virus. Involved areas display

the "LUXURY PERFUSION" phenomenon, with normal or increased total blood flow and low oxygen

extraction.

Q7. in which of the following sites is turbulent flow most likely to occur;;

1 ascending aorta;;

2 small arteries;;

3 capillaries;;

4 small veins;;

ANS= ASCENDING AORTA

Turbulent flow means that the blood flows crosswise in the vessel as well as along the vessel, usually

forming whorls in the blood called eddy currents.

Page 3: DocumentQ1

Reynolds’ number is the measure of the tendency for turbulence to occur( Re=(density*

velocity*diameter)/viscosity

Reynolds’ number for flow in the vascular system even normally rises to 200 to 400 in large arteries;

as a result there is almost always some turbulence of flow at the branches of these vessels. In the

proximal portions of the aorta and pulmonary artery, Reynolds’ number can rise to several thousand

during the rapid phase of ejection by the ventricles; this causes considerable turbulence in the

proximal aorta and pulmonary artery where many conditions are appropriate for turbulence: (1) high

velocity of blood flow, (2) pulsatile nature of the flow, (3) sudden change in vessel diameter, and (4)

large vessel diameter. However, in small vessels, Reynolds’ number is almost never high enough to

cause turbulence.

Q8. provenge is a vaccine treatment for which of the following cancers;;

1 hepatoma;;

2 multiple myeloma;;

3 prostate cancer;;

4 hodgkins lymphoma

ans=prostate cancer

In a widely anticipated announcement, the US Food and Drug Administration (FDA) has approved an

immunotherapy, sipuleucel-T (Provenge, Dendreon), for the treatment of asymptomatic or minimally

symptomatic metastatic, castration-resistant prostate cancer (mCRPC).

Often referred to as a vaccine, sipuleucel-T is an autologous active cellular immunotherapy, meaning

that it is made from the patient's own white blood cells and stimulates a patient's immune system to

respond against the cancer. The treatment needs to be manufactured individually for each patient.

"The availability of [sipuleucel-T] provides a new treatment option for men with advanced prostate

cancer who currently have limited effective therapies available," said Karen Midthun, MD, acting

director of the FDA's Center for Biologics Evaluation and Research, in a press statement.

Sipuleucel-T becomes the first product approved by the FDA in a new therapeutic class known as

active cellular immunotherapies, according to Dendreon press materials.

To be approved by the FDA, investigators had to show that sipuleucel-T would extend survival in men

with mCRPC.

The latest data from the pivotal trial, known as IMPACT (Immunotherapy for Prostate

Adenocarcinoma Treatment), were presented at the American Society of Clinical Oncology 2010

Genitourinary Cancers Symposium in March. Finally, FDA approved the vaccine on 29 April 2010, for

use in the treatment of advanced prostate cancer.

According to the FDA press release, the most common adverse reactions reported with the new

product are chills, fatigue, fever, back pain, nausea, joint ache, and headache. The majority of

adverse reactions were mild or moderate. Serious adverse reactions, which were reported in

approximately one quarter of the patients receiving sipuleucel-T, included some acute infusion

reactions and stroke.

Cerebrovascular events, including hemorrhagic and ischemic strokes, were observed in 3.5% of

patients in the sipuleucel-T group, compared with 2.6% of patients in the placebo control group

Q9. zebra bodies are seen in ;;

1 alport syndrome;;

2 fabrys disease;;

3 renal amyloidosis;;

Page 4: DocumentQ1

4 multiple myeloma;

Zebra bodies: Under electron microscope, they are bound by a single membrane, concentric or

lamellated with a periodicity of 5.6-5.8 nm. Zebra bodies are not unique to metachromatic

leukodystrophy and can also be seen in Tay-Sachs disease, Farbry’s disease, mucolipidoses and

mucopolysaccharidoses.

Q10. the abuse liability is greatest with ;;

1 caffeine;;

2 cocaine;;

3 benzodiazepines;;

4 marijuana;;

Q11. transesophageal echocardiography combines ultrasonography with which other procedure;;

1 electrocardiogram;;

2 ercp;;

3 endoscopy;;

4 none of the above;;

Q12. MIESCHERS GRANULOMA IS SEEN IN THE HISTOPATHOLOGY OF ;;

1 erythema nodosum;;

2 erythema induratum;;

3 nodular vasculitis;;

4 erythema marginatum

Q13. work done in respiratory system can be calculated by ;

1 bohr equation;;

2 alveolar gas equation;;

3 single breath nitrogen technique;;

4 relaxation -pressure curve

Q14. maximum number of days during which a pt can be kept for observation for diagnosis of mental

illness is ;;

1 10;;

2 14;;

3 20

4 30;;

Q15. ANOTHER WEIRD Q . WHOSE PERFECT ANSER IS NOT KNOWN TILL TODAY. WHICH IS NOT

INVOLVED IN CARCINOGENESIS OF COLON;;

1 APC;;

2 K-RAS;;

3 BETA CATENIN;;

4 MISMATCH REPAIR

Back to top

Posted: Fri Jun 11, 2010

5:50 pm Post subject:

Q16. the first function to be lost during the onset of spinal anaesthesia is ;;

1touch;;

2temperature;;

3vibration;;

4autonomic activity;

Page 5: DocumentQ1

Guest

Q17. the following are the branches of the internal iliac artewry , except ;;

1ovarian artery;;

2uterine artery;;

3sup vesical artery;;

4obturator artery;;

Q18. dry submarine in custodial torture injuries refers to ;;

1falanga-beating of soles of feet;;

2tying of a plastic bag over the years;;

3immersing head in contaminated water;;

4suspending the accused by ankles;

Q19. the following are radiological features of sigmoid vovulus, except ;;

1inverted U-shaped bowel loop;;

2liver overlap sign;;

3bird of prey sign;;

4cupola sign;

Back to top

vinay. Guest

Posted: Sat Jun 12, 2010 1:08 pm Post subject:

The cupola sign (1) is seen at supine radiography as an arcuate lucency overlying the lower thoracic spine and projecting caudad to the

heart . The superior border is well defined and the inferior margin is poorly delineated. The term cupola is used to indicate the inverted

cupshaped configuration of the lucency.

X-RAY

A normal plain abdominal x-ray will demonstrate a huge air filled distended bowel like the shape of an inverted U, with the convexity of

the U facing the right upper abdominal quadrant. This shape has been described as the kidney bean shape, coffee bean shape, bent inner

tube shape, ace of spades or ‘Omega loop Sign’. You can see an example down in the resource section.

BARIUM ENEMA

With a water soluble barium enema, the dilatation in the sigmoid colon can be demonstrated to be due to a twist, as it will show an area

of complete obstruction with some twisting in the so called bird beak or bird of prey sign.

Colonoscopy could be done in rare cases, which would help to confirm diagnosis, as well as treating the obstruction

the following are radiological features of sigmoid vovulus, except ;;

1 inverted U-shaped bowel loop;;

2 liver overlap sign;;

3 bird of prey sign;;

4 cupola sign;--answer

Page 6: DocumentQ1

Back to top

Posted: Mon Jun 14, 2010 1:13 pm Post subject:

Q20. radiologically -sub0acute haematogenous OM may resemble ;;

1 enchondroma;;

2 chondroma;;

3 ABC;;

4 OSTEOID OSTEOMA

In 1982, Roberts et al modified and expanded Gledhill's classification to 6 forms based on morphology, location, and similarity to

neoplasms, as follows

Modified classification of subacute osteomyelitis...

Type I is metaphyseal.

Type Ia is a punched-out central metaphyseal lesion.

Type Ib is an eccentric metaphyseal cortical erosion.

Type II is diaphyseal.

Type IIa is a localized cortical and periosteal reaction.

Type IIb is a medullary abscess in the diaphysis without cortical destruction but with onionskin periosteal reaction.

Type III is epiphyseal.

Type IIIa is a primary epiphyseal osteomyelitis.

Type IIIb is a lesion that crosses the epiphysis and involves both the epiphysis and the metaphysis.

Type IV is a metaphyseal equivalent.

Type IVa involves the vertebral body with an erosive or destructive process.

Type IVb involves the flat bones of the pelvis.

Type IVc involves the small bones, such as the tarsal bones.

* Type Ia lesions present as a punched-out radiolucency that is often suggestive of eosinophilic granuloma

Type Ib lesions are similar to type Ia lesions but have a sclerotic margin and appear as a classic Brodie abscess.

* Type II lesions erode the metaphyseal cortex and may appear similar to osteogenic sarcoma

* Type III lesions are observed as a localized diaphyseal cortical and periosteal reaction simulating osteoid osteoma

* Type IV diaphyseal lesions most often resemble Ewing sarcoma, with onionskin periosteal reaction

* Type V lesions occur in the epiphysis and appear as a concentric radiolucency.

* Type VI lesions involve the vertebral body with an erosive or destructive process.

Answer is Osteoid osteoma

Q21. blood flow through a vessel varies directly with ;;

1 resistance;;

2 viscosity;;

3 difference of pressure;;

4 length of vessel ;;

Mathematically, blood flow is described by Darcy's law (which can be viewed as the fluid equivalent of Ohm's law) and approximately by

Page 7: DocumentQ1

vinay. Guest

Hagen-Poiseuille equation

The first equation below is Darcy's law, the second is the Hagen-Poiseuille equation:

F =Difference of pressure/Resistance

R =( vL/r^4)(8/pi)

where:

F = blood flow

P = pressure

R = resistance

ν = fluid viscosity

L = length of tube

r = radius of tube

In the last equation it is important to note that resistance to flow changes dramatically with respect to the radius of the tube. This is

important in angioplasty, as it enables the increase of blood flow with balloon catheter to the deprived organ significantly with only a small

increase in radius of a vessel.

Blood flow is inversely proportional to the Resistance and Length of the blood vessel and viscosity of the blood. It is directly proportional

to the Difference in the pressure.

Q22. what percent of testicular volume is directly involved in producing sperms;;

1 20 percent''

2 40 percent;;

3 50 percent;;

4 80 percent;;

In healthy European adult human males, average testicular volume is 18 cm³ per testis, with normal size ranging from 12 cm³ to 30

cm³[5]. The average testicle size after puberty measures up to around 2 inches long, 0.8 inches in breadth, and 1.2 inches in height (5 x

2 x 3 cm). Measurement in the living adult is done in two basic ways:

80 percent of testicle volume is from the sperm producing cells, and that they rely on quite high concs of testosterone to swell and

fucntion

Q23. which one of the following charecteristics is most similar in the systemic and pulmonary circulations;;

1 stroke work;;

2 preload;;

3 afterload;;

4 peak systolic pressure;;

Q24. which muscle relaxant should not be used in myasthenia gravis;;

1 suxamethonium;;

2 pancuronium;;

3 atracurium;;

4 mivacurium;;

Page 8: DocumentQ1

Back to top

vinay. Guest

Posted: Tue Jun 15, 2010 1:25 pm Post subject:

Q24. suxamethonium is a depolarising muscle relaxant and it is contraindicated absolutely in conditions where there is a risk of loss of airway patency; known hypersensitivity to suxamethonium; Myasthenia gravis ; a family history of malig

hyperthermia; Glaucoma and ocular surgery; and genetic disorders of plasma psuedo cholinesterase.

In common with all neuromuscular blocking agents, atracurium paralyses the respiratory as well other skeletal muscles (e.g. muscles of arms, legs, eyel

administered only with adequate anaesthesia and only by an experienced anaesthetist familiar with its pharmacological properties. All facilities for endotracheal in

The neuromuscular block of atracurium is increased during hypothermia and decreases when rewarming the patient.

Atracurium should be adminstered with care to patients with myasthenia gravis, other neuromuscular diseases, and severe electrolyte imbalance, in view of the increased sensitivity of these pa

Severe acidosis may result in a slight prolongation of action of atracurium.

As with other neuromuscular blocking agents, MIVACRON(Mivacurium) may have profound neuromuscular blocking effects in cachect

in whom potentiation of neuromuscular block or difficulty with reversal may be anticipated, the initial dose should be decreased. A test dose of not more than 0.015 to 0.020 mg/kg, which represents the lower end of the dose

is recommended in such patients

Neuromuscular blocking agents may have a profound effect in patients with neuromuscular diseases (e.g., myasthenia gravis and

carcinomatosis), the use of a peripheral nerve stimulator and a dose of not more than 0.015 to 0.020 mg/kg MIVACRON is recomm

In patients who are known to have myasthenia gravis or the myasthenic (Eaton-Lambert) syndrome, small doses of pancuronium bromide may have profound effects. In such patients, a peripheral nerve stimula

value in monitoring the response to administration of muscle relaxants.

Answer is Suxamethonium as unlike the other options its actions are irreversible and CI in M.gravis while the other 3 can be used with caution

Q25. all of the following are associated with FITZ-CURTHS criteria, except ;;

1 bazexs syndrome;;

2 carcinoid syndrome;;

3 erythema gyratum repens;;

4 pityriasis rotunda

Q26. IMP PRODUCTS OF GLYCINE ARE ALL, EXCEPT ;;

1 creatine;;

2 taurine;;

3 heme;;

4 purines;;

creatine it is manufactured in the human body from L-arginine, glycine, and L-methionine

Back to top

Posted: Tue Jun 15, 2010 1:25 pm Post subject:

Q27. chronic reflux nephropathy causes;;

1 MGN;;

2 FSGS;;

3 MPGN;;

4 MCD;;

Page 9: DocumentQ1

FSGS can be classified as follows:

* Primary (idiopathic) FSGS

o FSGS with hyalinosis

o Progression from minimal-change disease

o Progression from immunoglobulin M (IgM) nephropathy

o Progression from mesangial proliferative glomerulonephritis

o Superimposed on other primary glomerulonephritis conditions (eg, membranous glomerulonephritis, immunoglobulin A [IgA]

nephropathy)

o Variants of primary FSGS

+ Collapsing form

+ Cellular variant (endocapillary and extracapillary hypercellularity)

+ FSGS with mesangial hypercellularity

+ FSGS with glomerular tip lesions

* Secondary FSGS

o Drugs

+ Intravenous heroin

+ Analgesics

o Viruses

+ Hepatitis B

+ HIV

+ Parvovirus

o Hemodynamic factors - With reduced renal mass

+ Solitary kidney

+ Renal allograft

+ Renal dysplasia

+ Renal agenesis

+ Oligomeganephronia

+ Segmental hypoplasia

+ VESICOURETERIC REFLUX

o Hemodynamic causes - Without reduced renal mass

+ Massive obesity

+ Sickle cell nephropathy

+ Congenital cyanotic heart disease

o Malignancies

+ Lymphomas

+ Other malignancies

o Scarring - Postinflammatory in postinfectious glomerulonephritis

o Miscellaneous

+ Hypertensive nephrosclerosis

+ Alport syndrome

+ Sarcoidosis

+ Radiation nephritis

Membranoproliferative GN is a form of glomerulonephritis caused by an abnormal immune response. Deposits of antibodies build

a part of the kidneys called the glomerular basement membrane. This membrane helps filter waste and extra fluid from the blood.

The changes in this membrane lead disrupt the body's ability to filter urine. Protein and fluid leak out of the blood vessels into body

tissues, leading to swelling (edema). Nitrogen waste products may build up in the blood (azotemia) because of poor kidney

functioning.

MGN

Primary/idiopathic

85% of MGN cases are classified as primary membranous glomerulonephritis -- that is to say, the cause of the disease is idiopathic

Page 10: DocumentQ1

vinay. Guest

(unknown). This can also be referred to as idiopathic membranous nephropathy. One study has identified antibodies to an M-type

phospholipase A2 receptor in 70% (26 of 37) cases evaluated.

Secondary

The remainder is secondary due to:

* autoimmune conditions (e.g., systemic lupus erythematosus)

* infections (e.g., syphilis, malaria, hepatitis B)

* drugs (e.g., captopril, NSAIDs, gold, mercury, penicillamine, probenecid).

* inorganic salts

* tumors, frequently solid tumors of the lung and colon; hematological malignancies such as chronic lymphocytic leukemia are less

common.

Pathogenesis

MGN is caused by circulating immune complex. Current research indicates that the majority of the immune complexes are formed via

binding of antibodies to antigens in situ to the glomerular basement membrane. The said antigens may be endogenous to (from)

basement membrane, or "planted" from systemic circulation.

The immune complex serves as an activator that triggers a response from the C5b - C9 complements, which form a membrane attack

complex (MAC) on the glomerular epithelial cells. This, in turn, stimulates release of proteases and oxidants by the mesangial and

epithelial cells, damaging the capillary walls and causing them to become "leaky". In addition, the epithelial cells also seem to secrete

an unknown mediator that reduces nephrin synthesis and distribution.

MCD

Minimal Change Disease is one of the most common causes of the Nephrotic Syndrome however, especially in children. In fact, almost

90% of children 10 and under with the Nephrotic Syndrome end up having MCD when biopsied (as opposed to 20% of adults).

Doctors tend to put MCD in two categories, primary and secondary. Primary means that the disease occurs on its own, for no obvious

reasons. This is by far the most common type. Secondary means that we think the MCD was caused by, or is at least associated with,

another medical condition. MCD has been associated with all of the following, usually in adults-

Drugs

NSAIDs, lithium, some antibiotics, bisphophonates

Malignancy

Leukemia, Lymphoma

Infection

Syphillis, HIV, Hepatitis

Allergy

Associated with multiple environmental allergies

secondary MCD is very uncommon.

Answer is FSGS

Back to top

Page 11: DocumentQ1

sujit. Guest

Posted: Wed Jun 16, 2010 5:05 pm Post subject:

Q28. the modern radiation detector used in ct is ;;

1 sodium iodide;;

2 xenon;;

3 ceramic based gadolinium compound;;

4 ZNCDS;;

Q29. the following remains unchanged in pregnancy;;

1 factor 2;;

2 platelet count;;

3 factor 11;;

4 clotting time;;

Back to top

Posted: Fri Jun 18, 2010 1:43 pm Post subject:

Q30. WHICH OF THE FOLLOWING IS NORMAL CAST IN URINE;;

1 granular;;

2 waxy;;

3 epithelial;;

4 hyaline;

Epithelial cell casts

This cast is formed by inclusion or adhesion of desquamated epithelial cells of the tubule lining. Cells can adhere in random order or in

sheets and are distinguished by large, round nuclei and a lower amount of cytoplasm. These can be seen in acute tubular necrosis and

toxic ingestion, such as from mercury, diethylene glycol, or salicylate. In each case, clumps or sheets of cells may slough off

simultaneously, depending of the focality of injury. Cytomegalovirus and viral hepatitis are organisms that can cause epithelial cell

death as well.

Hyaline casts

The most common type of cast, hyaline casts are solidified Tamm-Horsfall mucoprotein secreted from the tubular epithelial cells of

individual nephrons. Low urine flow, concentrated urine, or an acidic environment can contribute to the formation of hyaline casts,

and, as such, they may be seen in normal individuals in dehydration or vigorous exercise. Hyaline casts are cylindrical and clear, with

a low refractive index, so that they can easily be missed on cursory review under brightfield microscopy, or in an aged sample where

dissolution has occurred. On the other hand, phase contrast microscopy leads to easier identification. Given the ubiquitous presence of

Tamm-Horsfall protein, other cast types are formed via the inclusion or adhesion of other elements to the hyaline base.

Granular casts

The second-most common type of cast, granular casts can result either from the breakdown of cellular casts or the inclusion of

aggregates of plasma proteins (e.g., albumin) or immunoglobulin light chains. Depending on the size of inclusions, they can be

classified as fine or coarse, though the distinction has no diagnostic significance. Their appearance is generally more cigar-shaped and

of a higher refractive index than hyaline casts. While most often indicative of chronic renal disease, these casts, as with hyaline casts,

can also be seen for a short time following strenuous exercise.

Waxy casts

Page 12: DocumentQ1

vinay. Guest

Thought to represent the end product of cast evolution, waxy casts suggest the very low urine flow associated with severe,

longstanding kidney disease such as renal failure. Additionally, due to urine stasis and their formation in diseased, dilated ducts, these

casts are significantly larger than hyaline casts. While cylindrical, they also possess a higher refractive index and are more rigid,

demonstrating sharp edges, fractures, and broken-off ends. Waxy casts also fall under the umbrella of “broad” casts, a more general

term to describe the wider cast product of a dilated duct.

Answer is Hyaline casts.

Q31. doppler technique is based on ;;

1 VELOCITY SHIFT;;

2 AMPLITUDE SHIFT;;

3 FREQUENCY SHIFT;;

4 WAVELENTHSHIFT;;

The Doppler effect (or Doppler shift), named after Austrian physicist Christian Doppler who proposed it in 1842, is the change in

frequency of a wave for an observer moving relative to the source of the wave. It is commonly heard when a vehicle sounding a siren

or horn approaches, passes, and recedes from an observer. The received frequency is higher (compared to the emitted frequency

during the approach, it is identical at the instant of passing by, and it is lower during the recession.

Answer is Frequency shift.

Back to top

Posted: Sat Jun 19, 2010 9:54 am Post subject:

Q32. ALL OF THE FOLLOWING are assciated with low complement levels , except ;;

1. lupus nephritis;;

2 mesangiocapillary;;

3 diarrhoea associated with hemolytic uraemic syndrome;;

4 post - infectiouous glomerulonephritis

Deficiency of early components of classical pathway (C1, C4, C2): Autoimmune disease, especially systemic lupus erythematosus

(SLE), is the most common presentation in patients with early component deficiency.

Complement Factor H deficiency is also associated with atypical (diarrhea-negative) hemolytic-uremic syndrome (HUS) and

glomerulonephritis.

C3 deficiency is associated with mesangiocapillary glomerulonephritis.

Nephritis with low complement

1. Postinfectious proliferative glomerulonephritis

1. Poststreptococcal Glomerulonephritis (classic)

2. Subacute Bacterial Endocarditis

3. Osteomyelitis

2. Membranoproliferative glomerulonephritis

3. Systemic Lupus Erythematosus

4. Cryoglobulinemia

5. Diabetes Mellitus

6. Hepatitis C Virus

Page 13: DocumentQ1

vinay. Guest

* Factor H deficiency

o Factor H deficiency is associated with hemolytic-uremic syndrome (HUS), especially in familial cases that involve homozygous factor

H deficiency.

o Factor H deficiency also occurs with membranoproliferative glomerulonephritis.

o Parents have one half the normal levels of factor H.

o HUS associated with factor H deficiency is characterized by verotoxin-negative (diarrhea-negative) HUS.

Atypical and recurrent HUS is also seen in CD46 deficiency.

C3 nephritic factor (C3NeF) and mesangiocapillary glomerulonephritis (MCGN)

* C3Nef is an autoantibody that binds to and stabilizes the C3 convertase (C3bBb).

* The association of C3NeF and MCGN, especially MCGN type II, has been well defined. Different isotypes of C3NeF are recognized, the

main one being an IgG autoantibody against factor H. Factor H is an important regulator of the C3 conversion step in the alternate

pathway. C3NeF inhibits functions of factor H, which leads to overwhelming complement activation at the stage of C3 conversion.

Continuous C3 activation in vivo results in the well-known consequences of very low serum levels of C3 in MCGN.

* C3NeF may act directly within glomeruli to cause local complement activation and ensuing renal damage.

* C3Nef is also associated with partial lipodystrophy.

Answer is None

Q33. drug of choice to treat -theophylline toxicity;;

1 diazepam;;

2 neostigmine;;

3 propranlol;;

4 atropine

Benzodiazepines and other sedative agents

These agents are used to terminate seizures and for seizure prophylaxis in high-risk patients. They help to alleviate nausea and

vomiting and decrease tremors and anxiety induced by theophylline.

Diazepam (Valium)

Depresses all levels of CNS (eg, limbic and reticular formation), possibly by increasing activity of GABA.

Answer is Diazepam

Back to top

Posted: Sun Jun 20, 2010 2:49 pm Post subject:

Treatment

Prehospital Care

* Establish airway, breathing, and circulation (ABCs).

* Intravenous benzodiazepines may abort seizures.

Emergency Department Care

Evaluate ABCs and, if indicated, perform endotracheal intubation.

Page 14: DocumentQ1

* Vascular access for hemoperfusion may be required.

* Endotracheal intubation may be needed in patients who require high-dose benzodiazepines or barbiturates to control seizures.

* Consider gastric lavage (unless contraindicated) if the patient has recently (<1 h) ingested a significant amount or a sustained-

release preparation of theophylline or if theophylline bezoar formation is suspected. Gastric lavage should be considered in intubated

patients. Endoscopic bezoar fragmentation and retrieval may be utilized if lavage is not efficacious.

* Administer activated charcoal.

o Multidose activated charcoal (MDAC) enhances elimination of theophylline.

+ It is a very effective method of elimination, and it is considered the mainstay treatment of theophylline toxicity.

+ It is important to aggressively control nausea and vomiting in order to perform MDAC treatment.

+ It is also important that the patient is able to protect his or her airway in order to prevent aspiration of activated charcoal, which can

be detrimental.

o Administer the cathartic, sorbitol, with the activated charcoal one time.

* Perform whole-bowel irrigation (WBI) in patients with exposure to sustained-release theophylline preparations.

o Administer polyethylene glycol electrolyte solution.

+ Adults: 2 liters per hour until clear rectal effluent

+ Children: 500 mL/h until clear rectal effluent

* Theophylline-induced seizures tend to be resistant to treatment. Benzodiazepines (eg, lorazepam) are considered the first line of

treatment. Historically, phenobarbital prophylaxis was used in patients at high risk for seizures. High-risk cases include the following:

o Acute overdose with theophylline levels higher than 80 mcg/mL

o Chronic toxicity with levels higher than 40 mcg/mL

o Patients older than 60 years or younger than 3 years

* Benzodiazepines (IV) and phenobarbital may be used to treat seizures.

o CAVEAT: Barbiturates can precipitate hypotension.

o Phenobarbital has the added advantage of enhancing the hepatic metabolism of theophylline.

* Hypotension resistant to isotonic fluids (10-20 mL/kg) may require vasopressors with predominantly alpha-agonistic activity (eg,

phenylephrine, norepinephrine).

o In patients with theophylline toxicity, beta-blockade with propranolol has been shown to successfully reverse peripheral beta

receptor-mediated hypotension without apparent effect on concomitant tachycardia.

o However, always consider the risk of beta-adrenergic blockade to patients with preexistent bronchospastic disease.

* Esmolol, a short-acting beta-blocker, has been used successfully for unstable SVT and related hypotension in theophylline

overdose.2

o Exercise caution with beta-blocking agents because of their negative inotropic effects.

o Esmolol is a relatively selective beta1-receptor antagonist; thus, it may not have as much effect on beta2-mediated hypotension as

less-selective agents (eg, propranolol), although it is less likely to induce bronchospasm than other beta-blockers.

* Consider hemoperfusion with the following:

o Symptomatic patients with levels exceeding 90 mcg/mL in acute ingestions

o Theophylline concentrations exceeding 40 mcg/mL (chronic ingestion)

o Presence of life-threatening toxicity

+ Persistent seizures

+ Hypotension that is not responding to IV fluids

+ Ventricular dysrhythmias

o Hemodialysis is an alternative method of elimination enhancement but is considerably less effective than hemoperfusion.

* Correct electrolyte abnormalities in patients with ECG changes (eg, QTc prolongation) and/or ventricular dysrhythmias.

o Hypocalcemia

o Hypophosphatemia

o Hypokalemia

* Current recommendations for treating patients with tachycardia, hypotension, anxiety, and vomiting from theophylline overdose may

include the following:

o Fluid bolus with isotonic fluid (20 mL/kg)

o Metoclopramide or ondansetron to help control vomiting

o Propranolol to increase blood pressure - Reportedly propranolol treatment can correct hypokalemia.3

o Benzodiazepine to decrease anxiety, decrease risk of seizures, and help control vomiting

o Phenylephrine or norepinephrine to further increase blood pressure

o Charcoal hemoperfusion guided by response to treatment, underlying medical problems, and theophylline level

+ Because charcoal hemoperfusion is a somewhat complicated process that is not routinely used lately, most of the centers will

perform routine hemodialysis.

Page 15: DocumentQ1

seema. Guest

+ Hemodialysis in combination with MDAC will most of the time be sufficient for the treatment of severe theophylline toxicity.

Back to top

seema. Guest

Posted: Sun Jun 20, 2010 2:49 pm Post subject:

Factor H is a member of the regulators of complement activation family and is a complement control protein. It is a large (155

kilodaltons), soluble glycoprotein that circulates in human plasma (at a concentration of 500–800 micrograms per milliliter). Its main

job is to regulate the Alternative Pathway of the complement system, ensuring that the complement system is directed towards

pathogens and does not damage host tissue

Back to top

Posted: Sun Jun 20, 2010 2:51 pm Post subject:

Q34. mixing waves of stomach ;;

1 originate in body of stomach;;

2 originate in fundus of stomach;;

3 originate at incisura angularis;;

4 originates in any part of stomach;;

Answer is Fundus.

The food mass enters the stomach, and several minutes later mixing waves pass over the stomach. These are gentle, rippling

peristaltic movements which pass over the stomach every 15 to 25 seconds when there is food in the stomach. The movement of t

waves mixes the food with the secretions of the gastric glands, softening the food mass, and reducing it to a thin liquid called chyme

(pronounced kim). The fundus portion of the stomach is mainly a storage area, and few mixing waves take place there. Foods ma

stored is the fundus for an hour or more without becoming mixed with gastric juice. During this storage time, salivary digestion

continues. The food progresses through the stomach from the fundus to the body where the mixing waves become stronger and ev

stronger as the food reaches the pylorus. At the pylorus, each mixing wave forces a small amount of the stomach contents into the

duodenum.

Q35. all of the following are condensations of the deep cervical fasca of the neck, except ;;

1 the pretracheal layer;;

2 the prevertebral layer;;

3 the spheno mandibular ligaments ;;

4 the stylomandibular ligament;;

The Pretracheal Fascia (p. 823)

* This is a thin layer of deep cervical fascia that is anterior to the trachea and limited to the anterior aspect of the neck.

* It extends inferiorly from the thyroid cartilage and the arch of the cricoid cartilage into the thorax.

* The pretracheal fascia lies deep to the infrahyoid muscles.

* It splits to enclose the thyroid gland, trachea, and oesophagus and blends laterally with the carotid sheath. Thus, it encloses the

Page 16: DocumentQ1

vinay. Guest

viscera of the neck.

The Prevertebral Fascia (p. 825)

* This layer of deep cervical fascia forms part of a tubular sheath for the prevertebral muscles that surrounds the vertebral colum

* It is also continuous with the deep fascia covering the muscular floor of the posterior triangle of the neck.

* The prevertebral fascia extends from the base of the skull to the third thoracic vertebra, where it fuses with the anterior longitudinal

ligament.

* The prevertebral fascia extends inferiorly and laterally as the auxiliary sheath, which surrounds the axillary vessels and brachial

plexus.

The superficial (investing) layer of deep cervical fascia is attached posteriorly to the external occipital protuberance, the nuchal

ligament and the spine of the seventh cervical vertebra. It forms an investment for the trapezius muscle, and from the anterior border

of this muscle it continues forwards, covering the posterior triangle of the neck (it forms part of the roof of this triangle), to the

posterior border of the sternocleidomastoid muscle. Here, the layer divides to enclose the muscle, and at the anterior margin again

forms a single lamina which covers the anterior triangle of the neck and reaches forwards to the median plane where it is continuous

with the corresponding layer from the opposite side. In the median plane, it is adherent to the symphysis menti, the body of the hyoid

bone and to the anterior and posterior surfaces of the manubrium sterni.

Above, the fascial layer is attached to the superior nuchal line, the mastoid process and the whole of the base of the mandible. Between

the angle of the mandible and the mastoid process, the layer divides to ensheathe the parotid gland. The superficial layer, the parotid

fascia, covers the gland and extends upwards to become fixed to the zygomatic arch. From here, it continues upwards as the temporal

fascia to become attached to the superior temporal line. This layer covers the temporalis muscle which also originates in part from this

fascia. The deeper layer of the superficial layer, known as the stylomandibular ligament, passes deep to the parotid gland and ascends

to become attached to the styloid process and the tympanic bone.

The sphenomandibular ligament (internal lateral ligament) is a flat, thin band which is attached above to the spina angularis of the

sphenoid bone, and, becoming broader as it descends, is fixed to the lingula of the mandibular foramen. The ligament is derived from

Meckel's cartilage.

Back to top

Posted: Sun Jun 20, 2010 2:54 pm Post subject:

Q36. RATCHET PRINCIPLE is applicable in relation to ;;

1 performance apprisal;;

2 system analysis;;

3 network analysis;;

4 none of the above

Q37. most common adverse effect on eye in using oral contraceptive is ;;

1 color blindness;;

2 ring scotoma;;

3 optic neuritis;;

4 nystagmus

Q38. THE BENDERS AND THE THE STAGERS ARE SEEN IN ;;

1 sickle cell disease;;

2 caissons disease;;

3 radiation necrosis;;

Page 17: DocumentQ1

vinay. Guest

4 osteoarthritis

Q39. IN GLYCOLYSIS, which of the ions is most imp ;;

1 ca;;

2 mg;;

3 cu;;

4 zn;;

Q40. the most charecteristic finding in lichen planus is ;;

1 civatte bodies;;

2 basal cell degeneration;;

3 thinning f nail plate;;

4 violaceous lesions;;

Back to top

vinay. Guest

Posted: Mon Jun 21, 2010 1:19 pm Post subject:

Q41. NANO SCALE CANTILEVER ARRAY ARE BEING USED FOR ;;

1 TREATMENT of retinal detachment ;;

2 to detect biomarker of cancer cell;;

3 to detect amyloidosis in the early stage ;;

4 to detect early plaques of atherosclerosis

Nanotechnology extends the limits of molecular diagnostics to the nanoscale. Nanotechnology-on-a-chip is one more dimension of

microfluidic/lab-on-a-chip technology. Biological tests measuring the presence or activity of selected substances become quicker, more

sensitive and more flexible when certain nanoscale particles are put to work as tags or labels. Magnetic nanoparticles, bound to a

suitable antibody, are used to label specific molecules, structures or microorganisms. Magnetic immunoassay techniques have been

developed in which the magnetic field generated by the magnetically labeled targets is detected directly with a sensitive magnetometer.

Gold nanoparticles tagged with short segments of DNA can be used for detection of genetic sequence in a sample. Multicolor optical

coding for biological assays has been achieved by embedding different-sized quantum dots into polymeric microbeads. Nanopore

technology for analysis of nucleic acids converts strings of nucleotides directly into electronic signatures. DNA nanomachines can

function as biomolecular detectors for homogeneous assays. Nanobarcodes, submicrometer metallic barcodes with striping patterns

prepared by sequential electrochemical deposition of metal, show differential reflectivity of adjacent stripes enabling identification of the

striping patterns by conventional light microscopy. All this has applications in population diagnostics and in point-of-care hand–held

devices.

Q42. MEDICATIONS KNOWN TO ADVERSELY AFFECT GLYCAEMIC CONTROL IS ;;

1. anticonvulsants;;

2 antihistamines;;

3 antidepressants;;

4 calcium channel blockers;;

Page 18: DocumentQ1

Back to top

vinay. Guest

Posted: Tue Jun 22, 2010 1:00 pm Post subject:

Q43. SONTAGS METHOD S A METHOD TO STUDY;;

1 bone age;;

2 sexual development;;

3 hair growth;;

4 stage of adolescence;;

serial evaluation of children on growth hormone and other therapy

Q44. which one of the following protein segment is charecterised by its ability to bind to - phosphotyrosine -containing peptides;;

1 a helx-loop helix domain;;

2 a leucine zipper;;

3 an src homology 2 domain;;

4 a pleckstin motiff;;

Q45. antoni a and antoni b patterns are seen in ;;

1 schwannoma;;

2 neurofibroma;;

3 meningioma;;

4 teratoma;;

Back to top

Posted: Wed Jun 23, 2010 1:31 pm Post subject:

OEDIPIAL COMPLEX IS PART OF WHICH STAGE OF FREUDS THEORY OF DEVELOPMENT;;

1ORAL STAGE;;--answer

2ANAL STAGE;;

3LATENCY STAGE;;

4 NONE OF THE ABOVE

Freud's Theory of Development – the Oedipus Complex

Oedipus complex is one of the stages in Sigmund Freud's theory of sexual development. Freud's theory actually describes four stages of

development: oral, anal, phallic, and the Oedipus complex. All of these stages are necessary for proper development of the child. The

Oedipus complex is one of the most interesting though because of its description of the family structure.

The first three stages of sexual development make up the foundation for the last phase, the Oedipus complex. The child is born with

what is called polymorphous perversity, where every event that it experiences is sexual. This polymorphous perversity begins with the

first stage: oral. During this phase the child senses a sexual experience while feeding from the mother's breast. At this point the child

develops an object-cathexis which is later intensified during the Oedipus complex stage. Then as the child gets a little older it enters

the next stage, which is anal development. The child learns to control their bowels and therefore reaches a certain level of

independence. After this comes the phallic stage where the child gets a bit more older and realizes the difference between the sexes, or

simply put he figures out that his mother the doesn't have a penis. After all of these phases have been passed the child begins the last

and most important stage called the Oedipus complex. This stage is so crucial that Freud credits any future disorder to a malfunction in

the transitional phase between the Oedipus complex stage and the earlier stages of sexual development.

In the Oedipus complex phase the male child "…develops an object cathexis for his mother, which originally related to the mother's

breast ..." (26). His mother becomes the love-object meanwhile "the boy deals with his father by identifying himself with him. For a

time these two relationships proceed side by side until the boy's sexual wishes in regard to his mother become more

Page 19: DocumentQ1

vinay. Guest

Back to top

Posted: Wed Jun 23, 2010 1:34 pm Post subject:

Q46. actin is a microfilament that is involved wiyj all of the following activities, except ;;

1 endocytosis;;

2 exocytosis;;

3 cell locomotion;;

4 mitotic spindle formation

Q48. OEDIPIAL COMPLEX IS PART OF WHICH STAGE OF FREUDS THEORY OF DEVELOPMENT;;

1 ORAL STAGE;;

2 ANAL STAGE;;

3 LATENCY STAGE;;

4 NONE OF THE ABOVE

Q49. the ligament of landsmeer is found in ;;

1 phalange;;

2 hip;

3 wrist;;

4 elbow

ANSWER PHALANGE

The capsule, extensor tendon, and skin are very thin and lax dorsally, allowing for both phalanx bones to flex more than 100° until the

base of the middle phalanx makes contact with the condylar notch of the proximal phalanx.

At the level of the PIP joint the extensor mechanism splits into three bands. The central slip attaches to the dorsal tubercle of the

middle phalanx near the PIP joint. The pair of lateral bands, to which contribute the extensor tendons, continue past the PIP joint

dorsally to the joint axis. These three bands are united by a transverse retinacular ligament, which runs from the palmar border of the

lateral band to the flexor sheath at the level of the joint and which prevents dorsal displacement of that lateral band. On the palmar

side of the joint axis of motion, lies the oblique retinacular ligament [of Landsmeer] which stretches from the flexor sheath over the

proximal phalanx to the terminal extensor tendon. In extension, the oblique ligament prevents passive DIP flexion and PIP

hyperextension as it tightens and pulls the terminal extensor tendon proximally

Q50. culture medium used for chancroid is all, except ;;

1 blood agar;;

2 enriched blood agar;;

3 chocolate agar;;

4 mc coy cell line

Significant progress in the microbiologie diagnosis of chancroid occurred when Hammond and coworkers11 (1978) described a medium

consisting of gonococcal agar base, 1% bovine hemoglobin, 1% Iso- VitaleX enrichment (BBL Microbiology Systems, Cockeysville, Md),

and vancomycin hydrochloride (2.04 |xmojVL=^ mg/L). Using this selective solid medium, they successfully recovered H ducreyi from

eight of 16 genital ulcérations. Sottnek et al12 demonstrated good growth (90% + sensitivity) on a medium containing heart infusion

agar (BBL), 10% fetal bovine serum, and vancomycin hydrochloride (2.04 p-mol/L)- This group believed that the sensitivity of

Hammond's medium was increased by the addition of 10% fetal bovine serum-/ Oberhofer and Back13 found Miieller-Hinton agar to

be superior to trypticase soy agar for isolation of H ducreyi. Muelle''' Hinton solid medium was also more sensitive than either

heatinactivated or noninactivated clotted human blood in their study; only 12 of 15 specimens positive on Müeller-Hintot' agar were

positive on clot media. Sng and coworkers14 isolated 13 strains ofH ducreyi from genital ulcers using a specify agar (Bacto Protease

No. 3 agar, Difco Laboratories) with added soluble 0.1% starch, 1% IsoVitaleX (BBL), 15% human blood, and vancomycin in the s

Page 20: DocumentQ1

vinay. Guest

concentration as previously reported.

In Kenya in 1983, Nsanze et al" studied 201 patients with genital ulcérations. They recovered Hducreyi from 143 specimens (71%) on

gonococcal base agar with 2% bovine hemogl0' bin and 5% fetal alfserum (GC-HgS) with vancomycin hydro- chloride (2.04 ¡xmol/L)

and 122 specimens (61%) on Müellei-- Hinton agar with both 5% chocolatized horse blood (MH-HB' and vancomycin added. When

used together, the recover)' rate was 81%; consequently, these investigators recommended the routine use of both media,

incorporated into a singly biplate to acilitate their use." These results were supported by another Kenyan study reported by Dylewski

and coworkers" who found an isolation rate of 61% on GC-HgS, 48% of MH-HB, and 68% using both media in a study of 111 men

wit'1 clinical chancroid. Both media contained vancomycin hydro- chloride (2.04 |i.mol/L). A repeated culture at 48 hours yielded an

additional 8% positive cultures. A review of culture-negative patients revealed 11 with primary syphilis, thus resulting in an overall

recovery rate of 75% using one culture on both media, 82% if a second culture was obtained." A second culture, however, may prove

impractical in many venereal disease clinics where follow-up tends to be poor.

McCoy cells are applied for culture of various microorganisms, which are dependent in their development on the eukaryotic host cell.

During their interaction with pathogens the cells are subjected to various changes leading often to lethality of infected cells.As recent

reports show the cells are actually important for culturing viruses, chlamydia, vaccine studies, development of models for C.

trachomatis or cytotoxic activity. A new direction of McCoy application is the creation of McCoy- Plovdiv serum-free cell line.

Answer is MC Coy cell line.

Back to top

vinay. Guest

Posted: Thu Jun 24, 2010 1:18 pm Post subject:

Q51. THE cerebellum of a

human brain reaches its adult

size at the age of ''

1 6 months;;

2 12 months;;

3 15 months;;

4 24 months

Q52. FAIRBANKS CHANGES IN

THE KNEE JOINT SIGNIFIES ;;

1 patellofemoral pathology;;

2 chronic mcl injury;;

3 early deg joint disease;;

4 discoid meniscus;

Q53. mitral valve calcification is

best visualised with ---

projecton of chest radiograph;;

1 posteroanterior;;

2 anteroposterior;;

3 lateral;;

4 lordotic;;

Back to top

Page 21: DocumentQ1

Posted: Thu Jun 24, 2010 1:20 pm Post subject:

Q1 ALCOHOLIC PARANOIA..WHICH OF THESE IS SEEN??

1. IMPULSIVE BEHAVIOUR

2. HALLUCINATION

3. FIXED DELUSION

4. AGITATION

alcoholic paranoia A type of alcohol-induced psychotic disorder in which delusions of a self-referential or persecutory nature are

prominent. Alcoholic jealousy is sometimes included as a form of alcoholic paranoia.

alcoholic jealousy A type of chronic, alcohol-induced psychotic disorder, characterised by delusions that the marital or sexual partner is

unfaithful. The delusion is typically accompanied by intense searching for evidence of infidelity and direct accusations that may lead to

violent quarrels. It was formerly regarded as a distinct diagnostic entity, but this status is now controversial. Synonyms: amorous

paranoia; conjugal paranoia.

Answer is fixed delusion.

Q2 .WHICH OF FOLLOWING IS NOT PROVEN PART OF COGNITIVE BEHAV THERAPY??

1. PRE CONTEMPLATION

2. CONTEMPLATION

3. ACTION

4. CONSOLIDATION

Change implies phenomena occurring over time. However, this aspect was largely ignored by alternative theories of change. Behavior

change was often construed as an event, such as quitting smoking, drinking, or over-eating. The Transtheoretical Model construes

change as a process involving progress through a series of five stages.

Precontemplation is the stage in which people are not intending to take action in the foreseeable future, usually measured as the next

six months. People may be in this stage because they are uninformed or under-informed about the consequences of their behavior. Or

they may have tried to change a number of times and become demoralized about their ability to change. Both groups tend to avoid

reading, talking or thinking about their high risk behaviors. They are often characterized in other theories as resistant or unmotivated

or as not ready for health promotion programs. The fact is traditional health promotion programs are often not designed for such

individuals and are not matched to their needs.

Contemplation is the stage in which people are intending to change in the next six months. They are more aware of the pros of

changing but are also acutely aware of the cons. This balance between the costs and benefits of changing can produce profound

ambivalence that can keep people stuck in this stage for long periods of time. We often characterize this phenomenon as chronic

contemplation or behavioral procrastination. These people are also not ready for traditional action oriented programs.

Preparation is the stage in which people are intending to take action in the immediate future, usually measured as the next month.

They have typically taken some significant action in the past year. These individuals have a plan of action, such as joining a health

education class, consulting a counselor, talking to their physician, buying a self-help book or relying on a self-change approach. These

are the people that should be recruited for action- oriented smoking cessation, weight loss, or exercise programs.

Action is the stage in which people have made specific overt modifications in their life-styles within the past six months. Since action is

observable, behavior change often has been equated with action. But in the Transtheoretical Model, Action is only one of five stages.

Not all modifications of behavior count as action in this model. People must attain a criterion that scientists and professionals agree is

sufficient to reduce risks for disease. In smoking, for example, the field used to count reduction in the number of cigarettes as action,

or switching to low tar and nicotine cigarettes. Now the consensus is clear--only total abstinence counts. In the diet area, there is some

consensus that less than 30% of calories should be consumed from fat. The Action stage is also the stage where vigilance against

relapse is critical.

Maintenance is the stage in which people are working to prevent relapse but they do not apply change processes as frequently as do

people in action. They are less tempted to relapse and increasingly more confident that they can continue their change.

Answer is consolidation.

Page 22: DocumentQ1

suman. Guest

Q3 WHICH ONE IS NOT A COGNITIVE DYSFUNCTION

1. THOUGHT BLOCK

2. CATASTROPHIZATIION

3. SELECTIVE ABSRACTION

4. OVERGENERALISATION

Cognitive distortions can cloud perceptions, judgments and assumptions of a depressed individual. They include distorted ideas and

false assumptions, which need to be checked in order to restore mental health.

Overgeneralization is one of the common cognitive distortions that a person can experience during anxiety or depression.

Over-generalization: You take a negative event as a pattern of your life. We naturally relate new experiences to our old ones. So we

commonly generalize based on our past experience. Over-generalizations also make up many of our stereotypes of other people. It’s

like taking something that happened before and thinking that it will always happen again i.e. “I always screw up.”

A thought block is a disorder of thinking whereby the client’s capacity to maintain a train of thought is constantly interrupted.

selective abstraction is a type of cognitive distortion in which focus on one aspect of an event negates all other

Cognitive distortions

These are maladaptative thinking patterns that distort reality in a negative way, and make us perceive the world as being more hostile

than it actually is. Arbitrary inference, selective abstraction, over-generalisation, magnification, and minimisation are examples of

cognitive distortions.

Arbitrary inference refers to the drawing of an unjustified conclusion. For example, a businessman never takes his wife on any of his

official trips. His wife is upset. She concludes that he is concealing something from her, perhaps an extramarital affair. She neglects

other possible explanations.

Selective abstraction is the focussing of attention on one detail without regard to the rest of the picture. For example, a young man is

depressed because he does not have a motorcycle. He feels that no girl will take him seriously unless he has a "bike". He does not

consider that he has several assets such as intelligence and a pleasing personality. To him, these are of little importance.

Over-generalisation is the drawing of a general conclusion based upon a limited event. For example, a father discovers that his

adolescent son has been smoking. He is distraught. He concludes that the boy has picked up this habit from bad company. He

concludes that the boy is probably taking drugs as well. He concludes that his son is untrustworthy, and requires close supervision. He

neglects to take into account the possibility that his son, like many other youngsters his age, has probably merely been experimenting

with the experience.

Magnification is making mountains of molehills. Failing in an important examination is an unhappy event, but it is not the end of the

world. A sensible student would grieve briefly, then pick up the pieces of his life and begin studying again.

Minimisation is an undervaluation of positive attributes. A woman may have low self-esteem because she is not well-off. She neglects

the respect that she commands for being an efficient employee, a good mother, a caring wife, a cheerful neighbour, and a loyal friend.

Repeated intrusive (automatic) thoughts

When we are unhappy, our sadness is often sustained by repeated, intrusive thoughts. These push themselves into consciousness

preoccupy or even dominate the mind, leaving little opportunity for the experience of happier thoughts

Answer is catastrophization.

Page 23: DocumentQ1

Back to top

Posted: Fri Jun 25, 2010 1:14 pm Post subject:

Q54. MOST COMMON MANIFESTATION OF LYME CARDITIS;;

1 left ventricular dysfunction;;

2 concomitant myopercarditis;;

3 conduction abnormalities;;

4 rt ventricular dysfunction

Cardiovascular manifestations of Lyme disease often occur within 21 days of exposure and include fluctuating degrees of atrioventricular

(AV) block, acute myopericarditis or mild left ventricular dysfunction and rarely cardiomegaly or fatal pericarditis. AV block can vary from

first-, second-, third-degree heart block, to junctional rhythm and asystolic pauses.

About 10 percent of patients with Lyme disease develop evidence of transient cardiac involvement, the most common manifestation

being variable degrees of atrioventricular block at the level of the atrioventricular node. Syncope due to complete heart block is frequent

with cardiac involvement because often there is an associated depression of ventricular escape rhythms. Ventricular tachycardia occurs

uncommonly. Diffuse ST segment and T wave abnormalities and transient, usually asymptomic, left ventricular dysfunction may b

in some patients, although cardiomegaly or symptoms of congestive heart failure are rare. A positive gallium or indium antimyosin

antibody scan may point to suspected cardiac involvement in this disease. The demonstration of spirochetes in myocardial biopsies of

some patients with Lyme carditis suggests that the cardiac manifestations are due to a direct toxic effect, although there is speculation

that immune-mediated mechanisms may be involved as well.

Answer is conduction abnormalities.

Q55. which vaccine loses potency , when frozen;;

1 BCG;;

2 OPV;;

3 MMR;;

4 DPT;;

Diphtheria, tetanus and/or acellular pertussis- containing vaccines Includes DTPa, DTPa- hepB, DTPa-Hib, DTPa-IPV, DTPa- hepB-

DTPa-IPV- Hib, DTPa-IPV/Hib, DTPa-hepB-IPV-Hib, dTpa, DT (CDT), dT(ADT).

DO NOT FREEZE.As

Vaccines loses significant potency when stored at –5ºC to –10ºC.

Safe to store at 2ºC to 8ºC for 24 months in spite of continuous slow decrease in potency of the pertussis component.

BCG (freeze-dried or lyophilised vaccine)

Can be stored at up to –20ºC.

Do not expose to light (ultraviolet and/or fluorscent).

Safe storage for 12 months at 2 to 8 degrees.

Do not expose to light (ultraviolet and/or fluorescent).

Diluent – do not freeze (5) Store between 2ºC and 8ºC.

Measles-mumps-rubella (MMR)

(freeze-dried or lyophilised vaccine)

May be stored in freezer at 0ºC or below.

Protect from light, which may inactivate virus.

Safe storage for 2 years at 2ºC to 8ºC.

Diluent – do not freeze.

Store between 2ºC & 8ºC.

Oral poliomyelitis vaccine (OPV) opened vials

May be stored for up to 2 years at around –20ºC.

Page 24: DocumentQ1

vinay. Guest

Back to

top

vinay. Guest

Posted: Mon Jun 28, 2010 11:46 am Post subject:

Q64. teratogenic potential is a side effect of ;;

1 stavudine;;

2 zidovudine;;

3 efavirinez;;

4 lamivudine

Q65. which one of the following is not part of processing of mrna transcripts in eukaryotic cells;;

1 splicing;;

2 methylation;;

3 addition of a 5u cap;;

4 addition of a polyadenylate tail to the 3 u end

Q66. ABDUCTOR LURCH is a common finding after , which of the following osteotomies;;

1 salters;;

2 chiaris;;

3 triple innominate;;

4 periacetabular osteotomy

Q67. MESNA IS USED AS ANTIDOTE FOR ALL, EXCEPT ;;

1 ifosphamide;;

2 fludarabine;;

3 busulphan;;

4 acrolein

Q68. BOWL OF SPAGHETTI -phenomenon i s seen in ;;

1 adeno virus;;

2 guinea-worm disease;;

3 toxocara;;

4 ebola virus

The freeze-thaw-refreeze cycle can occur until the vial is empty.

Answer is DPT.

Q56. psychoanalysis as a form of psychotherapy includes all of the following except ;;

1 free association ;;

2 resistance;;

3 countertransference;;

4 meditation

Q57. the retinal pigment epithelium is charecterised by which of the following ;;

1 the presence of the photoreceptor perikarya;;

2 phagocytosis of worn0out components of photoreceptor cells;;

3 origin from the inner layer of the optic cup during embryonic develpment;;

4 presence of amacrine cells;;

Page 25: DocumentQ1

Back to

top

Guest

Posted: Mon Jun 28, 2010 11:47 am Post subject:

Adverse effects-Efavirenz

* Psychiatric symptoms, including insomnia, confusion, memory loss, and depression, are common, and more serious

symptoms such as psychosis may occur in patients with compromised liver or kidney function.

* Rash, nausea, dizziness and headache may occur

* Efavirenz can cause birth defects and should not be used in women who might become pregnant

* Safety in children has not been established

* Use of efavirenz can produce a false positive result in some urine tests for marijuana

Back to

top

Guest

Posted: Mon Jun 28, 2010 11:49 am Post subject:

Polyadenylation

Polyadenylation is the addition of a poly(A) tail to an RNA molecule. The poly(A) tail consists of multiple adenosine

monophosphates; in other words, it is a stretch of RNA which only has Adenine bases. In eukaryotes, polyadenylation is part

of the process that produces mature messenger RNA (mRNA) for translation. It therefore forms part of the larger process of

gene expression.

The process of polyadenylation begins as the transcription of a gene finishes. The 3'-most segment of the newly-made

first cleaved off by a set of proteins; these proteins then synthesise the poly(A) tail at the RNA's 3' end. In some genes th

proteins may add a poly(A) tail at any one of several possible sites, polyadenylation can therefore produce more than on

transcript from a single gene, similar to alternative splicing.

The poly(A) tail is important for the nuclear export, translation and stability of mRNA. The tail is shortened over time and

when it is short enough, the mRNA is enzymatically degraded. However, in a few cell types, mRNAs with short poly(A) tails

are stored for later activation by re-polyadenylation in the cytosol. In contrast, when polyadenylation occurs in bacteria, it

promotes RNA degradation. This is also sometimes the case for eukaryotic non-coding RNAs. The wide distribution of

polyadenylation among living organisms indicates that this process evolved early in the history of life on Earth.

Polyadenylation Primer on RNA

RNAs are a type of large biological molecules, whose individual building blocks are called nucleotides. The name poly(A) tail

(for polyadenylic acid tail)[6] reflects the way RNA nucleotides are abbreviated, with a letter for the base the nucleotide

contains (A for adenine, C for cytosine, G for guanine and U for uracil). RNAs are produced (transcribed) from a DNA

template. By convention, RNA sequences are written in a 5' to 3' direction. The 5' end is the part of the RNA molecule that i

transcribed first, and the 3' end is transcribed last. The 3' end is also where the poly(A) tail is found on polyadenylated RNAs.

Messenger RNA (mRNA) is RNA that has a coding region that acts as a template for protein synthesis (translation). The rest of

the mRNA, the untranslated regions, tune how active the mRNA is. There are also many RNAs that are not translated, called

non-coding RNAs. Like the untranslated regions, many of these non-coding RNAs have regulatory roles

In prokaryotes and organelles

In many bacteria, both mRNAs and non-coding RNAs can be polyadenylated. This poly(A) tail promotes degradation by the

degradosome, which contains two RNA-degrading enzymes: polynucleotide phosphorylase and RNase E. Polynucleotide

phosphorylase binds to the 3' end of RNAs and the 3' extension provided by the poly(A) tail allows it to bind to the RNAs

whose secondary structure would otherwise block the 3' end. Successive rounds of polyadenylation and degradation of the 3'

Page 26: DocumentQ1

end by polynucleotide phosphorylase allows the degradosome to overcome these secondary structures. The poly(A) tail

also recruit RNases that cut the RNA in two.[63] These bacterial poly(A) tails are about 30 nucleotides long.

In as different groups as animals and trypanosomes, the mitochondria contain both stabilising and destabilising poly(A) tails

Destabilising polyadenylation targets both mRNA and noncoding RNAs. The poly(A) tails are 43 nucleotides long on average.

The stabilising ones start at the stop codon, and without them the stop codon (UAA) is not complete as the genome only

encodes the U or UA part. Plant mitochondria only have destabilising polyadenylation, and yeast mitochondria have no

polyadenylation at all.

While many bacteria and mitochondria have polyadenylate polymerases, they also have another type of polyadenylation,

performed by polynucleotide phosphorylase itself. This enzyme is found in bacteria, mitochondria, plastids and as a

constituent of the archeal exosome (in those archaea that have an exosome). It can synthesise a 3' extension where the vast

majority of the bases are adenines. Like in bacteria, polyadenylation by polynucleotide phosphorylase promotes degradation

of the RNA in plastids and likely also archaea

Deadenylation

In eukaryotic somatic cells, the poly(A) tail of most mRNAs in the cytoplasm gradually get shorter, and mRNAs with shorter

poly(A) tail are translated less and degraded sooner. However, it can take many hours before an mRNA is degraded. This

deadenylation and degradation process can be accelerated by microRNAs complementary to the 3' untranslated region of an

mRNA. In immature egg cells, mRNAs with shortened poly(A) tails are not degraded, but are instead stored without being

translated. They are then activated by cytoplasmic polyadenylation after fertilisation, during egg activation.

In animals, poly(A) ribonuclease (PARN) can bind to the 5' cap and remove nucleotides from the poly(A) tail. The level of

access to the 5' cap and poly(A) tail is important in controlling how soon the mRNA is degraded. PARN deadenylates less if th

RNA is bound by the initiation factors 4E (at the 5' cap) and 4G (at the poly(A) tail), so this is why translation reduces

deadenylation. The rate of deadenylation may also be regulated by RNA-binding proteins. Once the poly(A) tail is removed,

the decapping complex removes the 5' cap, leading to a degradation of the RNA. Several other enzymes that seem to be

involved in deadenylation have been identified in yeast.

Back to top

Guest

Posted: Mon Jun 28, 2010 11:50 am Post subject:

Q64.

Efavirenz is an orally active, nonnucleoside reverse transcriptase inhibitor (nnRTI) that is specific for human immunodeficiency

virus, type 1 (HIV-1). It is indicated, in combination with other antiretroviral agents, for the treatment of HIV-1 infections.

Other drugs in this class are delavirdine and nevirapine. Efavirenz has a terminal half-life of 5276 hours after a single dose

and 4055 hours after multiple dosing. The shorter elimination time after chronic dosing is a result of P450 enzyme induction

that induces its own metabolism.

n reproduction studies, cynomolgus monkeys were administered oral efavirenz (60 mg/kg/day) throughout pregnancy (post

coital days 20150). This dose produced plasma drug concentrations similar to those achieved in humans with 600 mg/day.

Three of 20 exposed newborns had major congenital malformations, compared with 0 of 20 in nonexposed control monkeys.

The defects observed were 1 case each of anencephaly and unilateral anophthalmia, microphthalmia, and cleft palate. In

pregnant rats, doses producing plasma concentrations similar to those in humans resulted in an increase in fetal resorptions.

Neither mating nor fertility was impaired in rats at these doses. No teratogenic or toxic effects were observed in rabbits gi

doses producing plasma concentrations similar to those in humans

Back to

top

vinay. Guest

Posted: Mon Jun 28, 2010 11:51 am Post subject:

Page 27: DocumentQ1

drug which is not antioxidant??

allopurinol

trimetazidine

amifostine

chloroquine

ans given amifostine...

Amifostine is a pharmacological antioxidant used as a cytoprotectant in cancer chemotherapy and radiotherapy. It is thought

to protect normal tissues relative to tumor tissue against oxidative damage inflicted by cancer therapies by becoming

concentrated at higher levels in normal tissues. The degree to which amifostine nevertheless accumulates in tumors and

protects them against cancer therapies has been debated. Guidelines have been published that direct its use in chemotherapy

and radiation, taking into consideration the concerns of tumor protection.

Back to

top

vinay. Guest

Posted: Tue Jun 29, 2010 12:46 pm Post subject:

Q68. curve of carus is the nomenclature given to ;;

1 normal pelvic axis;;

2 abnormal pelvic axis;;

3 foetal axis;;

4 position of coccyx

Q69. TRANSCELLULAR BIOSYNTHESIS IS REPRESENTED BY ;;

1 prostaglandins;;

2 thromoxanes;;

3 leukotrienes;;

4 lipoxins;;

Q70. NHALATIONAL AGENT THAT CAN BE USED IN SHOCK PATIENTS IS ;;

1 ISOFLURANE;;

2 CYCLOPROPANE;;

3 HALOTHANE;;

4 DESFLURANE

Q71. WHICH of the following statement is true about ocd;;

1 thoughts are perceived as excessive and absurd;;

2 thoughts are inserted by some ext agency;;

3 obsessions are always accompanied by compulsions;;

4 socio-occupational functionning remains intact;;

Back to

top

vinay. Guest

Posted: Wed Jun 30, 2010 12:57 pm Post subject:

Q72. star gazer fetus , term is related to ;;

1 cephalic presentation;;

2 breech;;

3 transverse lie;;

4 face;

ans- breech

Page 28: DocumentQ1

star gazing position is Hyperextension of the fetal head in breech position. This can be evaluated with ultrasound.

Less than 5% of breech babies have their heads in the "star gazing" position, face looking straight upwards and the back of

the head resting against the back of the neck.

Caesarean delivery is absolutely necessary, because vaginal birth with the baby's head in this position confers a high ri

spinal cord trauma and death

Q73. which one of the following is attached to the lingula of the mandible;;

1 stylomandibular ligament;;

2 sphenomandibular ligament;;

3 pterygomandibular typhe;;

4 middle constrictor of pharynx

Q74. best source of complement is ;;

1 horse;;

2 guinea pig;;

3 man;;

4 dog;

Back to

top

vinay. Guest

Posted: Thu Jul 01, 2010 1:01 pm Post subject:

Q76. which of the following antiepileptic drugs has theshortest half-life;;

1 ethosuximide;;

2 phenobarbitol;;

3 phenytoin;;

4 valproate;;

ans: 4 valproate ( ref: KDT 6th edition )

drug --half life

1.ethosuximide--48hours(adults), 32 hrs (children)

2.phenobarbitone---80 to 120 hours

3.phenytoin --12 to 24 hours

4.valproate --10 to 15 hours

Q77. children having minimal change disease have selective proteinuria because of ;;

1 reduction of negative charge on GBM;;

2 EXTENSIVE DAMAGE TO THE gbm;;

3 DEPOSITS OF IGG AND C3 ON GBM;;

4 INCRESED MESANGIAL MATRIX IN THE GLOMERULI;;

ans= reduction of negative charge on GBM

REF- robbins PBD 8e chapter 20

extract

The current leading hypothesis is that minimal-change disease involves some immune dysfunction, eventually resulting in the

elaboration of a cytokine that damages visceral epithelial cells and causes proteinuria. The ultrastructural changes point to a

primary visceral epithelial cell injury, and studies in animal models suggest the loss of glomerular polyanions. Thus, defect

the charge barrier may contribute to the proteinuria. The actual route by which protein traverses the epithelial cell portion of

the capillary wall remains an enigma. Possibilities include transcellular passage through the epithelial cells, passage throu

residual spaces between remaining but damaged foot processes or through abnormal spaces developing underneath the

portion of the foot process that directly abuts the basement membrane, or leakage through foci in which the epithelial cells

have become detached from the basement membrane

Page 29: DocumentQ1

Back to

top

divakar

. Guest

Posted: Fri Jul 02, 2010 1:39 pm Post subject:

Q78. CHROMOSOME WALKING IS A TECHNIQUE USED TO DESCRIBE;;

1 more chromosome around the nucleus;;

2 more pigment of chromosomal dna from one area of chrosome to another;;

3 a method used to locate a gene using a set of clones from a dna library;;

4 recombination between chromosomal dna of two different species;;

answer 3

Chromosome walking is a technique to clone a gene (e.g., a disease gene) from its known closest markers. The closest linked

marker (e.g., EST or a known gene) to the gene is used to probe a genomic library. A restriction fragment isolated from the

end of the positive clones is used to reprobe the genomic library for overlapping clones. This process is repeated several ti

to walk across the chromosome and reach the gene of interest.

Q79. the adverse event that limits the use of flutamide as a primary treatment of bph is ;;

1 breast tenderness;;

2 diarrhoea;;

3 erectile dysfunction;;

4 loss of libido;;

imp points from goodman n gillman

ANDROGEN RECEPTOR ANTAGONISTS

Flutamide, Bicalutamide, and Nilutamide These relatively potent AR antagonists have limited efficacy when used alone

because the increased LH secretion elevates serum testosterone concentrations. They primarily are used in conjunction with a

GnRH analog in the treatment of metastatic prostate cancer, where they block the action of adrenal androgens that are not

inhibited by GnRH analogs. AR antagonists used in this fashion include flutamide (EULEXIN), bicalutamide (CASODEX), or

nilutamide (NILANDRON). Bicalutamide is replacing flutamide for this purpose because it has less hepatotoxicity and is taken

once daily. Nilutamide apparently has worse side effects than flutamide and bicalutamide. Flutamide also has been used

effectively to treat hirsutism in women, but the association with hepatotoxicity argues against its routine use for this cosmetic

purpose.

Monotherapy : In clinical studies, the most frequently reported adverse reactions to DROGENIL Tablets are gynaecomastia

and/or breast tenderness, sometimes accompanied by galactorrhoea. These reactions disappear upon discontinuation of

treatment or reduction in dosage.

Q80. the commonest paritcles used in teletherapy are ;;

1 photons;;

2 neutrons;;

3 electrons;;

4 gamma radiation;;

X-rays consist of electromagnetic radiation of wavelengths shorter than those of ultraviolet rays and longer than those of

gamma rays. X-ray wavelengths fall roughly in the range from 0.01–10 nm (nanometers, or billionths of a meter).

X-rays are produced artificially when high-speed electrons collide with a tungsten or other such heavy metal target.

Q81. induction agent of choice for hyperthyroid patient ;;

1 propofol;;

2 thiopentone;;

3 ketamine;;

Page 30: DocumentQ1

4 etomidate;;

Therapeutic radiation is delivered in three ways:

(1) teletherapy, with beams of radiation generated at a distance and aimed at the tumor within the patient;

(2) brachytherapy, with encapsulated sources of radiation implanted directly into or adjacent to tumor tissues; and

(3) systemic therapy, with radionuclides targeted in some fashion to a site of tumor.

Teletherapy is the most commonly used form of radiation therapy.

Radiation from any source decreases in intensity as a function of the square of the distance from the source (inverse square

law). Thus, if the radiation source is 5 cm above the skin surface and the tumor is 5 cm below the skin surface, the intensity

of radiation in the tumor will be 52/102, or 25% of the intensity at the skin

. By contrast, if the radiation source is moved to 100 cm from the patient, the intensity of radiation in the tumor will be

1002/1052[these 2 r squares], or 91% of the intensity at the skin.

Teletherapy maintains intensity over a larger volume of target tissue by increasing the source-to-surface distance.

In brachytherapy, the source-to-surface distance is small; thus, the effective treatment volume is small.

X-rays and gamma rays are the forms of radiation most commonly used to treat cancer.

They are both electromagnetic, nonparticulate waves that cause the ejection of an orbital electron when absorbed. This orbital

electron ejection is called ionization.

X-rays are generated by linear accelerators;

gamma rays are generated from decay of atomic nuclei in radioisotopes such as cobalt and radium

These waves behave biologically as packets of energy, called photons

Particulate forms of radiation are also used in certain circumstances. Electron beams have a very low tissue penetrance and

are used to treat skin conditions such as mycosis fungoides

Neutron beams may be somewhat more effective than x-rays in treating salivary gland tumors.

However, aside from these specialized uses, particulate forms of radiation such as neutrons, protons, and negative mesons,

which should do more tissue damage because of their higher linear energy transfer and be less dependent on oxygen, have

not yet found wide applicability to cancer treatment.

Back to

top

Vinay. Guest

Posted: Sat Jul 03, 2010 12:22 pm Post subject:

Q82. most common site of atopic dermatitis in an infant is ;;

1 scalp;;

2 elbow;;

3 trunk;;

4 ante cubital fossa

The distribution and skin reaction pattern varies according to the patient's age and disease activity

During infancy, the AD is generally more acute and primarily involves the face, scalp, and the extensor surfaces of the

Page 31: DocumentQ1

extremities ( Fig. 122-7). The diaper area is usually spared.

In older children, and in those who have long-standing skin disease, the patient develops the chronic form of AD with

lichenification and localization of the rash to the flexural folds of the extremities

AD often subsides as the patient grows older, leaving an adult with skin that is prone to itching and inflammation when

exposed to exogenous irritants. Chronic hand eczema may be the primary manifestation of many adults with AD

Q83. FLUOPHER IS SEEN IN ;;

1 FLUROSIS;;

2 OSTEOPETROSIS;;

3 SCURVY;;

4 CARBON TETRAFLOURIDE POISONING;

Q84. PAUCI-immune glomerulonephritis isseen in ;;

1 RPGN;;

2 IGA NEPHROPATHY;;

3 MICROSCOPIC POLYANGITIS;;

4 FSGS;;

answer 3 microscopic polyangitis

group of patients with small-vessel vasculitis (arterioles, capillaries, and venules; rarely small arteries) and glomerulonephritis

have serum ANCA; the antibodies are of two types, anti-proteinase 3 (PR3) or anti-myeloperoxidase (MPO) (Chap. 319).

ANCA are produced with the help of T cells and activate leukocytes and monocytes, which together damage the walls of small

vessels. Endothelial injury also attracts more leukocytes and extends the inflammation. Wegener's granulomatosis,

microscopic polyangiitis, and Churg-Strauss syndrome belong to this group because they are ANCA-positive and have a pauci

immune glomerulonephritis with few immune complexes in small vessels and glomerular capillaries. Patients with any of these

three diseases can have any combination of the above serum antibodies, but anti-PR3 antibodies are more common in

Wegener's and anti-MPO antibodies are more common in microscopic polyangiitis or Churg-Strauss. While each of these

diseases have some unique clinical features, most features do not predict relapse or progression, and as a group they are

generally treated in the same way. Only the presence of upper-airway involvement, persistent pulmonary injury, and anti

antibodies suggests that the course of disease will be more difficult. Induction therapy usually includes some combination of

plasmapheresis, methylprednisolone, and cyclophosphamide. The benefit of plasmapheresis in this setting is uncertain. The

steroids are tapered soon after acute inflammation subsides, and patients are maintained on cyclophosphamide or

azathioprine for up to a year to minimize the risk of relapse.

Microscopic Polyangiitis

Clinically, these patients look somewhat similar to those with Wegener's granulomatosis, except they rarely have significant

lung disease or destructive sinusitis. The distinction is made on biopsy where the vasculitis in microscopic polyangiitis is

without granulomas. Some patients will also have injury limited to the capillaries and venules.

Q85. WHICH STAIN IS NOT USED FOR THE DIAGNOSIS OF ;;pneumocystis carinii infection;;

1 toluidine blue;;

2 giemsa;;

3 acridine orange;;

4 wright stain;;

Back to

top

vinay. Guest

Posted: Sun Jul 04, 2010 5:22 pm Post subject:

Q86. snowstorm knee is pathognomic of ;;

1synovial chondromatosis;;

Page 32: DocumentQ1

2osteochondroitis dissecans;;

3fat embolism;;

4chondromalacia paptellae

The aetiology of multiple loose bodies. Snow storm knee

PR Kay, AJ Freemont, and DR Davies

Park Hospital, Manchester, England.

We report four patients who showed hundreds of brilliant white loose bodies at arthroscopy of the knee after a short history of

pain and crepitus. Histological, historical and clinical evidence is presented which indicates that the aetiology of this con

is the culture of chondrocytes in synovial fluid. It is suggested that reversal of the usually accepted order of events in synovial

osteochondromatosis could provide a better and unified explanation for both that condition and multiple loose bodies. The

term 'snow storm knee' is proposed to describe the dramatic picture seen at arthroscopy.

Numerous white rice-grain size free bodies, presenting the so-called "snow storm appearance" were observed inside the

articular cavity of the left knee. Proliferation of the synovium on the ACL was identified and cartilaginous tissue was included.

Although there were numerous free bodies, no proliferation was found in the synovium except around the ACL, and no

damage was found in the meniscus and cartilage. The synovium that proliferated around the ACL was removed as much as

possible using a shaver and forceps, and free bodies that were assumed to be cartilaginous segments were washed out

Q87. skier thumb is due to ;;

1injury to ulnar collateral ligament;;

2fracture of base of 1st metacarpal;;

3fracture of trapezoid;;

4fracture of scaphoid tubercle;;

answer 1

The skier's thumb injury was described as an acute injury to the ulnar collateral ligament. When a skier falls with his or he

hand caught in a ski pole, the thumb can be pulled away from the hand. Because of the shape of the ski pole, the thumb

tends to get caught and significant stresses are placed on the ulnar collateral ligament. If the ulnar collateral ligament is

pulled far enough, it will tear. While there are many ways to injure the ulnar collateral ligament, a skier's thumb is the proper

eponym for an acute injury to the ligament.

The other injury is called a gamekeeper's thumb; this refers to a more chronic pattern of injury that leads to loosening of t

ulnar ligament over time. The name comes from the European gamekeepers who would kill their game by grasping the head

of the animal between their thumb and index finger to break its neck. Over time, the ulnar collateral ligament is stretched a

would eventually cause problems. Again, gamekeeper's thumb is the proper eponym to describe chronic injuries to the ulnar

collateral ligament.

Q88. the following are correct regarding omphalocele, except ;;

1is usually covered by a translucent membrane;;

2is frequently associated with other congenital malformations;;

3is lateral to the umbilical stump;;

4is within the umbilical ring

answer 3

it is gastrochisis which is lateral to umblical stump not omphalocele

Q89. which of the following is pathognomonic feature of t an abscess on ct ;;

1hypodense centre;;

2presence of air;;

3thick enhancing wall;;

4adjacent fat stranding;

Page 33: DocumentQ1

Q90. removal of proximal segments of the small intestine results in a decrease in ;;

1maximal acid output;;

2gastric emptying of liquids;;

3 gastric emptying of solids;;

4pancreatic enzyme secretion;;

with the removal of small intestine receptors which give information regarding composition of chyme to brain are removed

and reflex secretion of pancreatic and liver enzyme is affected

Inflammation or removal of the upper small intestine leads to a decrease in pancreatic and hepatobiliary function. The

proximal small intes¬tine contains a number of "receptors" that monitor the physical (volume) and chemical (pH, fat content,

caloric density, osmolality) composition of the chyme emptied from the stomach.

Stimulation of these receptors releases hormones and activates neural reflexes that initiate pancreatic enzyme and

bicarbonate secretion, stimulate gallbladder emptying, and provide feedback for inhibitory regulation of gastric function

(enterogastrone, enterogastric reflex).

Removal of these reflexes decreases pancreatic secretion and gall¬bladder emptying and increases gastric emptying and acid

output.

Q91. which one of the following is a cofactor and not a coenzyme;;

1biotin;;

2tetrahydrofolic acid;;

3copper;;

4 methycobalamine;;

A cofactor is a non-protein chemical compound that is bound to a protein and is required for the protein's biological activity.

loosely-bound cofactors termed coenzymes and tightly-bound cofactors termed prosthetic groups.

An inactive enzyme, without the cofactor is called an apoenzyme, while the complete enzyme with cofactor is the

holoenzyme.

cofactors: such as the metal ions Mg2+, Cu+, Mn2+ or iron-sulfur clusters.

Q92. DRUG CAUSING OLIGOSPERMIA;;

1LEFLUONAMIDE;;

2D-PENICILLAMINE;;

3METHOTREXATE;;

4CORTICOSTEROIDS;;

Back to

top

vinay. Guest

Posted: Mon Jul 05, 2010 1:49 pm Post subject:

Q93. RUBOXISTAURIN IS A NEW DRUG BEING USED FOR , WHICH OF THE FOLLOWING CONDITIONS;;

1prevention of diabetic retinopathy;;

2prevention of hypertensive retinpathy;;

3anti-asthmatic drug;;

4lipid lowering agent

The most common sites of mandibular fracture are at the body (21-40%), condyles (15-20%), and angle (20-31%). In

addition, 10-15% affect the parasymphysis; 3-9%, the ramus; 3-5%, the alveolar ridge; and 1-2%, the coronoid process.

Again, these numbers vary depending on the characteristics of a given community. Automobile accidents are associated with

higher percentages of condylar fractures, whereas motorcycle accidents are frequently associated with symphyseal and

Page 34: DocumentQ1

parasymphyseal fractures. When the patient's history is of assault, body and angle fractures are more common.

watz most cmnly fractured site of mandible.............?? above frm emedicine...

below wikipediaLocation

* Condyle – 30%

* Angle – 25%

* Body – 25%

* Symphesis – 15%

* Ramus – 3%

* Coronoid process – 2%

The mandible may be dislocated anteriorly (to the front) and inferiorly (downwards) but ve

Back to

top

vinay. Guest

Posted: Mon Jul 05, 2010 1:51 pm Post subject:

Q94. THE CALCIUM CHANNEL blocker , which can reverse the resistance of cancer cells to chemotherapeutic drugs is ;;

1verapamil;;

2diltiazem;;

3bepiridil;;

4nicardipine;;

a review of the pharmacological background and clinical activity of different Ca2+ channel blockers concerning reversal of

anticancer drug resistance in tumors suggests that verapamil and trifluoperazine may be the most immediate candidates.

Reversal of resistance to anthracyclines and vinca alkaloids originally observed in mouse experimental leukemia cell lines has

now been extended to other animal cell lines and human tumors as well-----------

Q95. oil- drop phenomenon is seen in ;;

1clubbing ;;

2psoriasis of nails;;

3lichen planus of nails;;

4tinea umguum;

A yellow “oil droplet” lesion in the nail is typical for psoriasis. Dermatophyte infection and lymphedema may also cause

yellowing of the lateral nail border. A “yellow nail syndrome,” with all nails yellow without cuticles, can be seen in chronic

chest infections or lymphedema. Yellow nails

Q96. JORDANS SIGN IS SEEN IN ;;

1.congenital dislocation of patella;;

2nail patella syndrome;;

3JRA;;

4PIGMENTED VILLONODULAR SYNOVITIS;;

Q97. which of the following indices is not an index of measurement of obesity;;

1body mass index;;

2ponderal index;;

3broca index;;

4sullivans index;

Q98 . tunica reaction is seen in ;;

1h.infuenja;;

Page 35: DocumentQ1

2plague;;

3r.mooseri;;

4b;;anthrax

Endemic typhus — there is swelling of scrotum and inflammation of tunica vaginalis in male guinea pig (Tunica reaction)

Rickettsia typhi, causes Endemic typhus

Back to

top

vinay. Guest

Posted: Mon Jul 05, 2010 1:54 pm Post subject:

Q99. GHENT CRITERIA IS USED IN DIAGNOSIS OF WHICH CONDITION ;;

1SCLERODERMA;;

2SLE;;

3MARFANS SYNDROME;;

4VASCULITIS;;

Marfan syndrome is currently diagnosed using criteria based on an evaluation of the family history, molecular data, and 6

organ systems. The diagnosis cannot be based on molecular analysis alone because molecular diagnosis is not generally

available, mutation detection is imperfect, and not all FBN1 mutations are associated with Marfan syndrome. With the

previous Berlin criteria, Marfan syndrome was diagnosed on the basis of involvement of the skeletal system and 2 other

systems, with the requirement of at least one major manifestation (ie, ectopia lentis, aortic dilatation or dissection, or du

ectasia).

Ghent criteria, they identify major and minor diagnostic findings, which are largely based on clinical observation of various

organ systems and on the family history. A major criterion is defined as one that carries high diagnostic precision because i

relatively infrequent in other conditions and in the general population. The Ghent criteria were intended to serve as an

international standard for clinical and molecular studies and for investigations of genetic heterogeneity and genotype-

phenotype correlations. The clinical diagnosis in adults should be made using the Ghent criteria, which are unreliable in

children.

The major criteria include the following:

A first-degree relative (parent, child, or sibling) who independently meets the diagnostic criteria

Presence of an FBN1 mutation known to cause Marfan syndrome

Inheritance of an FBN1 haplotype known to be associated with unequivocally diagnosed Marfan syndrome in the family

In family members, major involvement in one organ system and involvement in a second organ system

Q100. endosperules may be observed in the tissues or fluids of patients infected with;;

1blastomyces dermatitidis;;

2coccidioides inmitis;;

3cryptococcus neoformans;;

4sporothrix schenckii;;

Q101. THE CENTRE OF GRAVITY FOR THE HUMAN BODY IS ;;

1just ant to hip joint;;

2through the symphisis pubis;;

3just ant to s2;;

4lust posterior to l5;;

Q102. which of the following is an activator of LCAT;;

1APOB100;;

2APOB48;;

3APOE;;

Page 36: DocumentQ1

4APOA1;

answer apo a1

Lysolecithin (lysophosphatidylcholine) may be formed by an alternative route that involves lecithin: cholesterol

acyltransferase (LCAT). This enzyme found in plasma, catalyzes the transfer of a fatty acid residue from the 2 position of

lecithin to cholesterol to form cholesteryl ester and lysolecithin and is considered to be responsible for much of the choles

ester in plasma lipoproteins. Long-chain saturated fatty acids are found predominantly in the 1 position of phospholipids,

whereas the polyunsaturated acids (eg, the precursors of prostaglandins) are incorporated more into the 2 position. The

incorporation of fatty acids into lecithin occurs by complete synthesis of the phospholipid, by transacylation between

cholesteryl ester and lysolecithin, and by direct acylation of lysolecithin by acyl-CoA. Thus, a continuous exchange of the fatty

acids is possible, particularly with regard to introducing essential fatty acids into phospholipid molecules.

LCAT—and the LCAT activator apo A-I—

bind to the disk, and the surface phospholipid and free cholesterol are converted into cholesteryl esters and lysolecithin

(Chapter 24). The nonpolar cholesteryl esters move into the hydrophobic interior of the bilayer, whereas lysolecithin is

transferred to plasma albumin.

Thus, a nonpolar core is generated, forming a spherical, pseudomicellar HDL covered by a surface film of polar lipids and

apolipoproteins. In this way, the LCAT system is involved in the removal of excess unesterified cholesterol from lipoproteins

and tissues.

Back to

top

vinay. Guest

Posted: Wed Jul 07, 2010 12:24 pm Post subject:

Q103. which tendon is most commonly involved in impingement syndrome ;;

1.bicipital tendon;;

2subscapularis;;

3supraspinatus;;

4infraspinatus

supraspinatus tendon...

impingment syndrome = supraspinatus tendonitis+ sub acromial bursitis

Q104. complications of bronchial artery embolisation include all, except ;;

1cerebrovascular accident;;

2paraplegia;;

3renal failure;;

4pul monary infarcton

Q105. hoffman -rigler sign is used for the assessment of ;;

1left ventricular enlargement;;

2right ventricular enlargement;;

3right atrial enlargement;;

4left atrial enlargement;;

Hoffmann's sign

A tingling sign is a sensation triggered by a mechanical stimulus in the distal part of an injured nerve. This sensation radi

peripherally, from the point where it is triggered to the cutaneous distribution of the nerve. The tingling response can be

compared with that produced by a weak electric current, as in transcutaneous electrical nerve stimulation (TENS). This

unpleasant sensation is not a severe pain and does not persist.

Q106. which of the following alcoholic drink has the most purine content;;

1.whisky;;

Page 37: DocumentQ1

2vodka;;

3beer;;

4 rum;;

Q107. PSEUDOHYPERKALAEMIA , CAUSES INCLUDE ALL, EXCEPT ;;

1prolonged tourniquette application ;;

2leukocytosis;;

3thrombocytosis;;

4hyporeninaemia

Q108. which of the following statement is not true about chronic papilloedema;;

1collection of extracellular fluid;;

2stasis of axoplasmic transport;;

3disruption of neurofilament;;

4swelling of axon

Q109. limits of confidence of a hypothesis is determined by ;;

1power factor;;

2level of significance;;

3 1-power factor;;

4 1 -level of significance

Back to

top

jain. Guest

Posted: Thu Jul 08, 2010 12:44 pm Post subject:

107. PSEUDOHYPERKALAEMIA, CAUSES INCLUDE ALL, EXCEPT ;;

1prolonged tourniquette application ;;

2leukocytosis;;

3thrombocytosis;;

4hyporeninaemia--causes hyperlalaemia

Pseudohyperkalemia is a rise in the amount of potassium that occurs due to excessive leakage of potassium from cells, during

or after blood is drawn. It is a laboratory artifact rather than a biological abnormality and can be misleading to caregivers.

Pseudohyperkalemia is typically caused by hemolysis during venipuncture (by either excessive vacuum of the blood draw or

by a collection needle that is of too fine a gauge); excessive tourniquet time or fist clenching during phlebotomy (which

presumably leads to efflux of potassium from the muscle cells into the bloodstream); or by a delay in the processing of the

blood specimen. It can also occur in specimens from patients with abnormally high numbers of platelets (>500,000/mm³),

leukocytes (> 70 000/mm³), or erythrocytes (hematocrit > 55%). People with "leakier" cell membranes have been found,

whose blood must be separated immediately to avoid pseudohyperkalemia.

108. which of the following statement is not true about chronic papilloedema;;

1collection of extracellular fluid;;

2stasis of axoplasmic transport;;

3disruption of neurofilament;;

4swelling of axon

collection of extracellular fluid

"Electron microscopic studies indicate that most of the increase in tissue elevation that occurs in papilledema results from

intra-axonal swelling and not from extracellular edema. The intra axonal swelling is accompanied by an increase in

mitochondria, disorganization of neurofilaments, and the accumulation of dense intracellular membrane-enclosed

bodies."Walsh and Hoyt's Clinical Neuro-Ophthalmology, 2005

Answer is a, according to me

Page 38: DocumentQ1

Back to

top

vinay. Guest

Posted: Thu Jul 08, 2010 12:49 pm Post subject:

Q110. which of the following charecteristics of an axon is most dependant on its diameter;;

1the magnitude of its resting potential;;

2the duration of its refractory period;;

3the conduction velocity of its action potential;;

4the overshoot of its action potential

Answer 3

Comparison of the conduction velocities of myelinated and unmyelinated axons in the cat. The conduction velocity v of

myelinated neurons varies roughly linearly with axon diameter d (that is, v ∝ d), whereas the speed of unmyelinated neurons

varies roughly as the square root (v ∝√ d). The red and blue curves are fits of experimental data, whereas the dotted lines

are their theoretical extrapolations.

Q111. which of the following antihypertensive drug shows therapeutic window phenomenon''

1sodium nitroprusside;;

2clonidine;;

3methyldopa;;

4reserpine

Answer 2

Therapeutic window phenomenon This is an unusual feature seen with certain drugs Optimal therapeutic effect is exerted only

over a narrow range of plasma drug concentrations or drug doses Both below & above this range, the beneficial effects are

suboptimal. i.e; the effect declines if the dose is increased beyond a certain level E.g. tricyclics ( Imipramine), Clonidine

Glipizide

Q112. which of the following sugars is absorbed from the small intestine by facilitated diffusion;;

1glucose;;

2galactose;;

3fructose;;

4sucrose

Answer 3

Facilitated diffusion is a movement of nutrients against a concentration gradient and usually does not require energy, but do

require a carrier. Fructose absorption uses a carrier without expending energy pulling water with it upon entering the

intestines.

Passive diffusion is a movement of compounds across the cell membrane so as to equalize the concentration of the substrate

on both sides of the membrane. This process applies only to such small molecules as electrolytes, water, and small sugars. It

does not apply to such large molecules as starch or large proteins. Almost all the water-soluble nutrients are absorbed in this

manner or by osmosis (passive absorption).

Active transport is a process requiring energy (ATP) and a nutrient carrier to move an essential nutrient against a gradient.

Most carriers are nutrient-specific, and almost all required nutrients are actively transported. Minerals are the exception, as

they require both passive diffusion and carrier-mediated transport. Amino acids are actively transported, but also require a

carrier.

Pinocytosis or phagocytosis happens when the absorptive cell engulfs the material. This process is used for fat absorption.

Page 39: DocumentQ1

Back to top

Guest

Posted: Fri Jul 09, 2010 12:54 pm Post subject:

Luxury perfusion phenomenon in acute herpes simplex virus encephalitis.

Tanaka M, Uesugi M, Igeta Y, Kondo S, Sun X, Hirai S.

Department of Neurology, Gunma University School of Medicine, Japan.

Abstract

In a patient with acute herpes simplex virus (HSV) encephalitis, positron emission tomography (PET) demonstrated increased

cerebral blood flow in the affected temporal lobe accompanied by reduction in the cerebral oxygen extraction fraction and the

cerebral metabolic rate of oxygen, i.e., luxury perfusion. Follow-up PET studies showed reduction in cerebral perfusion until it

was more closely coupled with oxygen metabolism after the resolution of the acute inflammation. These findings support

previous single photon emission computed tomographic data and provide a pathophysiological background for the occurrence

of hyperperfusion in HSV encephalitis. This is an interesting example of the luxury perfusion phenomenon occurring in a

disease other than cerebral ischemia.

Back to

top

vinay. Guest

Posted: Fri Jul 09, 2010 12:56 pm Post subject:

Q113. ALL OF THE FOLLOWING are features of the foetal hydantoin syndrome, except ;;

1dysmorphic facies;;

2hypoplasia of distal phal;anges and nails;;

3iugr;;

4renal dysplasia

About one third of children whose mothers are taking this drug during pregnancy typically have children who have intrauterine

growth restriction with microcephaly and develop minor dysmorphic craniofacial features and limb defects including

hypoplastic nails and distal phalanges (birth defects). A smaller population will have growth problems and developmental

delay, or mental retardation. Rare side effects include methemoglobinemia.

Fetal hydantoin syndrome was first described in 1973 by Loughnan et al.

It consists of an array of anomalies, including

craniofacial anomalies,

distal digital hypoplasia,

epicanthal folds,

hypertelorism,

low-set ears, and

developmental delay.

The early descriptions of this syndrome were noted in 12 infants, 11 of whom had been exposed to other AEDs, notably

barbiturates. Of infants born to women who used phenytoin during pregnancy, 10-30% are reported to exhibit some of the

syndromic findings, most commonly distal digital hypoplasia. However, few infants exposed to monotherapy have the entire

constellation of findings. Of note, in a long-term neurodevelopmental study, 16 patients whose mothers received phenytoin

monotherapy during pregnancy demonstrated slightly delayed locomotor development compared with children whose mothe

took carbamazepine and with normal controls.

more on phenytoin

Varied Hand and foot defects include fingerlike thumbs, aplasia or hypoplasia of the distal phalanges, supernumerary

phalangeal epiphyses, and clubfoot.Dermatoglyphic abnormalities consist of abnormal palmar creases and nail hypoplasia or

aplasia.[b]

[b]General defects include growth retardation, atypical facial appearance, hirsutism, and low hairlines.Facial problems inclu

microcephaly, brachycephaly, midfacial hypoplasia, wide fontanels, metopic ridging, mild micrognathia, low-set deformed

Page 40: DocumentQ1

ears, blepharoptosis, mild hypertelorism, strabismus, short nose with a broad depressed bridge and epicanthal folds, cupid's

bow of the upper lip, and occasionally cleft lip and palate.

Reported torso abnormalities include short neck with mild webbing, widely spaced pages , umbilical or inguinal hernia, and rib

anomalies.Internal abnormalities include variable coarctation of the aorta, endocardial cushion defect, double-outlet right

ventricle, ventricular septal defect, atrial septal defect, bicuspid pulmonic valve, and intestinal malrotation.

Ambiguous genitalia are rarely associated with this syndrome

.A patient with the dysmorphic characteristics of fetal hydantoin syndrome presented with unusual hyperpigmentation of

several fingernails.

Another neonate presented with gum hypertrophy, digitalization of the thumbs, hypoplasia of the distal phalanges and nails,

epicanthal folds, pseudohypertelorism, epidermoid cyst, and geographic tongue.Onychopathy can be a monosymptomatic or

mild form of this syndrome. This syndrome may be associated with neonatal acne.

Studies: Phenytoin affects folate and vitamin K metabolism, which may increase fetal bleeding. Some investigators have

suggested that vitamin K be administered to mothers taking phenytoin in the third trimester to prevent hemorrhage.

However, the available evidence does not justify this policy of giving vitamin K throughout the last third of pregnancy to al

women being treated with anticonvulsants. When used in pregnancy, phenytoin can cause a syndrome of birth defects

referred to with various names, such as Dilantin congenital defects, fetal hydantoin syndrome (FHS), Meadow syndrome,

congenital hydantoin syndrome, Dilantin syndrome, fetal Dilantin syndrome, fetal phenytoin syndrome, and hydantoin

syndrome. Various malformations have been reported to occur because of phenytoin intake during pregnancy.

Q114. TURP SHOULD COMMENCE WITH ;;

1incision of the bladder neck;;

2resectionm of the middle lobe;;

3resection of tissue at 12 oclock;;

4resection of tissue at 3 to 9 oclock;;

1.Approximately 25% of all candidates for TURP present with urinary retention and require preoperative catheter drainage.

Some of these men may develop postobstructive diuresis and other electrolyte disturbances, which require appropriate

management. An attempt should be made to lower elevated BUN and creatinine levels in patients who are thought to be

azotemic due to urinary obstruction. For this, use continuous Foley catheter drainage for at least 2 weeks prior to any

anticipated surgery.

2.Prolonged catheter drainage before a TURP may also be helpful in patients with decompensated hypotonic bladders to help

them regain a more normal bladder capacity and muscle tone. Despite the severity of the obstruction, many of these patients

respond well to medical management with alpha-blockade, 5-alpha reductase inhibitors, or both.

3.Preoperative urodynamic studies are only indicated in patients considered to have underlying neurologic disease or those

who may have detrusor decompensation from chronic or recurrent urinary retention. Urodynamic testing results can help

predict whether the chronically distended bladder has the capability to regain adequate muscle tone and capacity. One

technique is to leave an indwelling catheter for a period of time (usually 2-4 wk) and then remeasure the bladder capacity and

voluntary voiding pressure. A reduction in bladder capacity or an increase in voluntary voiding pressure suggests the

for reasonably normal bladder function eventually

4.The use of preoperative finasteride (Proscar) and dutasteride (Avodart) has been demonstrated to help reduce bleeding

during and after TURP surgery, although the optimal timing is unclear. A recent prospective study found that 2-4 months of

pretreatment with finasteride significantly increased immediate postoperative hemoglobin levels, but only in patients whose

resected prostatic weight was more than 30 grams, suggesting this may be most helpful in patients with larger prostates.

Finasteride (Proscar) and dutasteride (Avodart) also reduce recurrent episodes of hematuria secondary to BPH. They may also

help control excessive or prolonged hematuria after prostatic needle biopsy.

These 5-alpha-reducatase inhibitor medications decrease suburethral prostatic microvessel density, inhibit prostate

Page 41: DocumentQ1

angiogenesis, and lower prostatic vascular endothelial growth factor, which are the likely mechanisms by which they reduce

prostatic bleeding.

5.Surgical alternatives to TURP are designed to decrease blood loss, inpatient hospitalization, and fluid absorption, while still

removing or destroying the obstructing prostatic tissue. These include the

vaporization TURP (VaporTrode),

bipolar TURP,

photoselective vaporization of the prostate (PVP), and

holmium laser enucleation.

PVP....

KTP laser energy at 532 nm is highly absorbed by oxyhemoglobin and only penetrates 1-2 mm deep into the prostatic tissue,

making it theoretically superior to other types of prostatic laser vaporization procedures. Modern KTP lasers produce 80 watts

or more of average power and 240 watts of peak power, which vaporizes the prostatic tissue fairly rapidly. The limited tissue

penetration, compared to Nd:YAG lasers, minimizes the adverse effects associated with other types of prostate laser

vaporization.

The PVP procedure has excellent hemostasis because the blood vessels are rapidly sealed by the KTP laser energy. No tissue

is available for analysis, but the procedure can be performed relatively quickly. It can safely be performed even in patients

who cannnot be taken off their anticoagulant medications, which is a major advantage over TURP. In the hands of skilled

operators, typical operating times are less than 1 hour for glands up to 100 grams.

General principles of transurethral prostate resection

Use of a plastic barrier sheath (eg, Lingeman sheath, O'Connor sheath) helps maintain sterility and protects the operative

field, while allowing digital manipulation of the prostate through the rectum.

Make sure the patient is positioned with the buttocks flush with the end of the cystoscopy table. Resection of the anterior

prostate opposite the bladder neck can be impaired when deflection of the resectoscope is restricted by the edge of the

cystoscopy table. Varying patient height to a comfortable level and using the Trendelenburg position appropriately should

make the resection easier and more comfortable and can facilitate visualization.

####procedure always begins with a careful cystoscopic inspection of the anterior urethra, external urinary sphincter,

verumontanum, prostatic urethra, bladder neck, prostatic median lobe, trigone, ureteral orifices, and the rest of the bladder

using a small-caliber cystoscope (see image below). This inspection is important not only to verify the absence of associated

pathologies (eg, bladder tumors, urethral strictures, vesical stones), but also to help the surgeon obtain a clear 3-dimensional

mental image of the patient's specific anatomical features and relationships.

always know exactly where the verumontanum is located and do not resect it. Without this anatomical landmark, one can

easily lose orientation and risk damaging the external sphincter muscle, causing the patient to be permanently incontinent. If,

during the case you are not absolutely certain of your exact location, orientation, or position relative to the verumontanum,

stop resecting immediately and reorient by finding a stable landmark such as the bladder neck or verumontanum.

Irrigating fluid should be kept at the lowest height (pressure) level possible to maintain an adequate flow. Raising the

irrigating fluid level from 60 cm to 70 cm height has been shown to dramatically affect fluid absorption. A starting fluid he

of 60 cm is suggested and is usually sufficient.

PERFORMING RESECTION

Resect tissue only when pulling or withdrawing the cutting loop toward the resectoscope, never when pushing it forward. This

cleanly separates the resected tissue from the rest of the prostate gland and prevents tunneling, perforation, bladder injury,

and extravasation.

Page 42: DocumentQ1

If a very large and obstructing median lobe is present, resect this first regardless of the method chosen for the rest of the

transurethral resection

When resecting at the bladder neck, make sure enough fluid is present in the bladder to keep the posterior bladder wall away

from the surgical area. Approximately 100 mL prevents accidental posterior bladder wall injury and perforation. Before

resecting at the 4 to 5-o'clock and 7 to 8-o'clock positions at the bladder neck, visually check the position of the ureteral

orifices to prevent their inadvertent resection

When cutting the anterior tissue at the 12-o'clock position, making relatively shallow straight cuts is better, which leaves the

surface fairly flat, rather than trying to make it concave or curved. The prostate is thinnest here; the external sphincter i

its most proximal position and the risk of perforation is relatively high, especially if this area is resected early in the case.

Try to resect all the prostatic tissue possible without perforating the capsule or unduly extending the operating time. Resid

tissue can regrow, foster infection, increase fluid absorption, and tends to bleed. An old urology axion states "It is not how

much is taken out that causes the postoperative problems, it is how much is left in."

Cautery involving the verumontanum is discouraged because it can result in painful ejaculation.

Do not attempt a TURP on a prostate that is too large for the procedure to be reasonably completed in 90 minutes of resecting

time. Either do the case via an open technique, send the patient to a colleague with more experience in TURP surgery, or

perform some alternative procedure. If in the middle of the case and unable to complete the entire resection in 90 minutes, at

least finish one lateral lobe, the median lobe (if enlarged), and the bladder neck. This often results in very good clinical

outcomes and offers essentially the same symptom relief as the completed TURP.

Leaving a small amount of nonobstructing prostate tissue near the verumontanum is much better than risking permanent

incontinence through an injury to the external sphincter muscle.

The most common area of injury to the external sphincter is at the 12-o'clock position, where direct visualization of the

verumontanum is not possible. Therefore, be extremely careful at the distal surgical margins, particularly when resecting at

the 12-o'clock position opposite the verumontanum, to avoid inadvertent damage to the external sphincter.

VARIOUS TECHNIQUES

The Nesbit technique is probably the best-known and most commonly performed TURP method. It was first described by Reed

M. Nesbit (see image below) of Michigan in his landmark 1943 book on transurethral prostatectomy and is currently

considered the standard approach to TURP surgery.15 As originally described by Nesbit, the procedure is divided into 3

stages: (1) intravesical or proximal, (2) extravesical, and (3) apical.

intravesical portion, the resectoscope is positioned with the tip between the bladder neck and the midpoint of the prostatic

urethra proximal to the verumontanum. This point is determined by the relative size of the intravesical prostate. Resection

begins by removing the intravesical portion of the prostate and bladder neck tissue. This is removed, along with the

immediately adjacent prostatic adenoma, starting at the 12-o'clock position and working clockwise

extravesical phase of the procedure, the resectoscope is repositioned just in front of the verumontanum, and the resection is

continued from the previous distal resected margin to just proximal to the verumontanum, starting again at the 12-o'clock

position. This channel is continued from the 12-o'clock to the 4-o'clock position on the left side and to the 8-o'clock position

on the right side. The intent is to create a channel between the surgical capsule and the bulk lateral lobe tissue (see secon

image below). This encircling maneuver, combined with the intravesical and bladder neck resection performed earlier, results

in the bulk of the 2 lateral lobes falling onto the floor of the prostatic fossa (see third image below). In addition, they a

essentially devascularized and can be resected easily with minimal bleeding. The bulk of the prostate tissue is resected during

this part of the procedure.

the final stage of the procedure is the apical stage, in which the remaining apical tissue around the verumontanum is careful

Page 43: DocumentQ1

removed (see image below). Again, Nesbit starts the apical portion of the procedure with an anterior resection (12-o'clock

position), using the verumontanum as the main landmark.

MILNER TECHNIQUE

initial resection groove is made at the 9-o'clock position and carried down to capsular fibers. This groove extends from the

bladder neck to a point parallel to the verumontanum at the level of the ejaculatory ducts. Coagulation is not used, except f

major bleeding sites when the resection has reached the surgical capsule. The groove is first extended upwards toward the

11-o'clock position and then downward toward the 7-o'clock position. No attempt is made to encircle the bulk tissue. Rather,

the intent is to resect the lateral lobe tissue from the inside out quickly and to reach the surgical capsule expeditiously, at

which point the perforating and bleeding vessels can be cauterized if necessary. The next section in line is resected until a

entire lobe is finished. The process is repeated on the opposite side

final area of resection is the anterior tissue between the 11-o'clock and the 1-o'clock positions. The anterior resected surface

is left relatively straight and flat to avoid perforation in this area. The risk of perforation when resecting the anterior t

relatively high because this is where the prostate is thinnest and no distal landmark is visible on the anterior surface, which

risks possible inadvertent injury to the external sphincter. Also, the anterior tissue is rarely obstructing; therefore, this

resected last in case the procedure needs to be terminated early for any reason.

CONCLUSION

if prostate is large more than 100mg.. first to b dissected median lobe

n accord to nesbit tech which is stil best n most common tech resction shud commence at 4 TO 8 ie intravesical>>>>

extravesical>>> apical

so answer here after seein so many books n disc i m in favour of choice 4..IE LATERAL LOBE REMOVED FIRST

nesbit is more common procedure

n for choice 4 is accord to milner tech

Milner personally instructed this author in his method, which differs slightly from the Nesbit technique described earlier. To

the author's knowledge, this is the first time his particular technique for transurethral prostatectomy has been published in

the modern era, and it is offered here as a tribute to this highly skilled surgeon and teacher.

Q115. 3 MILLION PLAN WAS GIVEN BY WHICH COMMITTEE;;

1bhore committee;;

2kartar singh committee;;

3mudaliar committee;;

4chadah committee;;

ANSWER bhore committe

his committee, known as the Health Survey & Development Committee, was appointed in 1943 with Sir Joseph Bhore as its

Chairman. It laid emphasis on integration of curative and preventive medicine at all levels. It made comprehensive

recommendations for remodeling of health services in India. The report, submitted in 1946, had some important

recommendations like :-

1.Integration of preventive and curative services of all administrative levels.

2. Development of Primary Health Centres in 2 stages :

a. Short-term measure – one primary health centre as suggested for a population of 40,000. Each PHC was to be manned by

2 doctors, one nurse, four public health nurses, four midwives, four trained dais, two sanitary inspectors, two health

Page 44: DocumentQ1

assistants, one pharmacist and fifteen other class IV employees. Secondary health centre was also envisaged to provide

support to PHC, and to coordinate and supervise their functioning.

b. A long-term programme (also called the 3 million plan) of setting up primary health units with 75 – bedded hospitals for

each 10,000 to 20,000 population and secondary units with 650 – bedded hospital, again regionalised around district hospitals

with 2500 beds.

3. Major changes in medical education which includes 3 - month training in preventive and social medicine to prepare “social

physicians”.

___________________________________________________________________________________________

MUDALIAR COMMITTEE. 1962

This committee known as the “Health Survey and Planning Committee”, headed by Dr. A.L. Mudaliar, was appointed to assess

the performance in health sector since the submission of Bhore Committee report. This committee found the conditions in

PHCs to be unsatisfactory and suggested that the PHC, already established should be strengthened before new ones are

opened.

Strengthening of sub divisional and district hospitals was also advised. It was emphasised that a PHC should not be made to

cater to more than 40,000 population and that the curative, preventive and promotive services should be all provided at th

PHC. The Mudaliar Committee also recommended that an All India Health service should be created to replace the erstwhile

Indian Medical service.

____________________________________________________________________________________________

CHADHA COMMITTEE, 1963.

This committee was appointed under chairmanship of Dr. M.S. Chadha, the then Director General of Health Services, to advise

about the necessary arrangements for the maintenance phase of National Malaria Eradication Programme. The committee

suggested that the vigilance activity in the NMEP should be carried out by basic health workers (one per 10,000 population),

who would function as multipurpose workers and would perform, in addition to malaria work, the duties of family planning and

vital statistics data collection under supervision of family planning health assistants.

_____________________________________________________________________________________

MUKHERJEE COMMITTEE. 1965.

The recommendations of the Chadha Committee, when implemented, were found to be impracticable because the basic health

workers, with their multiple functions could do justice neither to malaria work nor to family planning work

. The Mukherjee committee headed by the then Secretary of Health Shri Mukherjee, was appointed to review the performance

in the area of family planning. The committee recommended separate staff for the family planning programme. The family

planning assistants were to undertake family planning duties only. The basic health workers were to be utilised for purposes

other than family planning.

The committee also recommended to delink the malaria activities from family planning so that the latter would received

undivided attention of its staff.

___________________________________________________________________________________________

MUKHERJEE COMMITTEE. 1966.

Multiple activities of the mass programmes like family planning, small pox, leprosy, trachoma, NMEP (maintenance phase),

etc. were making it difficult for the states to undertake these effectively because of shortage of funds. A committee of state

health secretaries, headed by the Union Health Secretary, Shri Mukherjee, was set up to look into this problem. The

committee worked out the details of the Basic Health Service which should be provided at the Block level, and some

consequential strengthening required at higher levels of administration

___________________________________________________________________________________________

JUNGALWALLA COMMITTEE, 1967

This committee, known as the “Committee on Integration of Health Services” was set up in 1964 under the chairmanship of

Dr. N Jungalwalla, the then Director of National Institute of Health Administration and Education (currently NIHFW). It was

asked to look into various problems related to integration of health services, abolition of private practice by doctors in

government services, and the service conditions of Doctors. The committee defined “integrated health services” as :-

Page 45: DocumentQ1

a.

A service with a unified approach for all problems instead of a segmented approach for different problems.

b.

Medical care and public health programmes should be put under charge of a single administrator at all levels of hierarchy.

__________________________________________________________________________________

KARTAR SINGH COMMITTEE. 1973.

his committee, headed by the Additional Secretary of Health and titled the "Committee on multipurpose workers under Health

and Family Planning" was constituted to form a framework for integration of health and medical services at peripheral and

supervisory levels. Its main recommendations were :-

a. Various categories of peripheral workers should be amalgamated into a single cadre of multipurpose workers (male and

female). The erstwhile auxiliary nurse midwives were to be converted into MPW(F) and the basic health workers, malaria

surveillance workers etc. were to be converted to MPW(M). The work of 3-4 male and female MPWs was to be supervised by

one health supervisor (male or female respectively). The existing lady health visitors were to be converted into female health

supervisor.

b One Primary Health Centre should cover a population of 50,000. It should be divided into 16 subcentres (one for 3000 to

3500 population) each to be staffed by a male and a female health worker.

____________________________________________________________________________________________________

SHRIVASTAV COMMITTEE. 1975.

This committee was set up in 1974 as "Group on Medical Education and Support Manpower" to determine steps needed to (i)

reorient medical education in accordance with national needs & priorities and (ii) develop a curriculum for health assistants

who were to function as a link between medical officers and MPWs. It recommended immediate action for :

1. Creation of bonds of paraprofessional and semiprofessional health workers from within the community itself.

2. Establishment of 3 cadres of health workers namely – multipurpose health workers and health assistants between the

community level workers and doctors at PHC.

3. Development of a “Refferal Services Complex”

4. Establishment of a Medical and Health Education Commission for planning and implementing the reforms needed in health

and medical education on the lines of University Grants Commission.

Acceptance of the recommendations of the Shrivastava Committee in 1977 led to the launching of the Rural Health Service.

___________________________________________________________________________________

BAJAJ COMMITTEE, 1986.

"Expert Committee for Health Manpower Planning, Production and Management" was constituted in 1985 under Dr. J.S. Bajaj,

the then professor at AIIMS. Major recommendations are :-

1. Formulation of National Medical & Health Education Policy.

2. Formulation of National Health Manpower Policy.

3. Establishment of an Educational Commission for Health Sciences (ECHS) on the lines of UGC.

4. Establishment of Health Science Universities in various states and union territories.

5. Establishment of health manpower cells at centre and in the states.

6. Vocationalisation of education at 10+2 levels as regards health related fields with appropriate incentives, so that good

quality paramedical personnel may be available in adequate numbers.

Page 46: DocumentQ1

7. Carrying out a realistic health manpower survey.

Posted: Sat Jun 26, 2010 1:12 pm Post subject:

Q58. The most recently advocated thery of PEM pathogenesis is ;;

1 theory of adaptation;;

2 theory of free radicals;;

3 classical thery of pem;;

4 gopalans thery

Q59. TRACHYONYCHIA IS SEEN IN ALL, EXCEPT ;;

1. LICHEN PLANUS;;

2 PSORIASIS;;

3 ALOPECIA AREATA;;

4 PARONYCHIA

Twenty-nail dystrophy is also known as ‘trachyonychia’. It could be said that twenty-nail dystrophy is

widespread trachyonychia involving all 20 nails. The condition is characterised by longitudinal ridging

(alternating elevation and depression), pitting, loss of lustre, and roughening (similar to sandpaper) of

the nail surface.

Twenty-nail dystrophy most commonly occurs in childhood. In one study the condition was most

evident in young males in the 10-20 year old age group (52%). Slight nail abnormalities may be

evident at birth with the condition slowly progressing over the years to cause changes in the texture of

fingernails and toenails. Typical signs and symptoms include:

* Nails become grubby, rough and brittle

* Some nails may become distorted

* Nail examination shows longitudinal ridging, pitting, roughening and splitting

* Nails lose their lustre and may change to a muddy, greyish-white colour (sand-blasted appearance)

The cause of twenty-nail dystrophy is unknown but in some cases it appears to be associated with

other skin conditions such as lichen planus, eczema, psoriasis and alopecia areata. In some cases an

autosomal dominant pattern of inheritance has been found; in one report twenty-nail dystrophy

occurred in 4 males in 3 successive generations. Other cases of twenty-nail dystrophy are of unknown

origin and begin gradually in early childhood. These cases tend to be self-limiting and may resolve

slowly with age.

Q60. Biliary leak after liver transplantation is associated with, which of the following vascular

complications;;

1 portal vein thrombosis;;

2 hepatic vein thrombosis;;

3 hepatic artery thrombosis;;

4 mesenteric ischemia;;

Q61. 24 hrs refrigeration test is done to assess;;

1 hyperuricaemia;;

2 hyperlipoprotaenaemia;;

3 hypercalcaemia;;

4 determining etiology of jaundice

lipemia-refrigeration test

refrigeration of a lipemic blood sample may distinguish between triglyceride-rich lipoproteins, which

persist in the turbid sample, and chylomicra, which rise to form a flocculent top layer while the sample

clears

Page 47: DocumentQ1

Q62. PAS STAINS ALL THE FOLLOWING, EXCEPT ;;

1 GLYCOGEN;;

2 LIPIDS;;

3 FUNGAL CELL WALL;;

4 BASEMENT MEMBRANE OF BACTERIA

answer 2

Periodic Acid-Schiff (PAS)

This histology stain is particularly useful for staining glycogen and other carbohydrates, but is useful

for many things. It is often used to show glomeruli, basement membranes, and glycogen in the liver.

PAS stains glycogen, mucin, mucoprotein, and glycoproteins magenta. The nuclei will stain blue.

Collagen will stain pink.

Q63. Which of the following drug binds to bcl-2 messenger rna causing fragmentation of the protein

message ;;

1 oblimersen sodium;;

2 bevacizumab;;

3 endostatin;;

4 angiostatin

Genasense® (oblimersen sodium) Injection inhibits the production of a protein known as Bcl-2. By

reducing production of Bcl-2 in cancer cells, Genasense seeks to restore the biological process

whereby cancer cells are killed by standard anticancer treatments – a process known as apoptosis.

Genasense is an investigational anticancer agent undergoing testing to evaluate its safety and efficacy

in various forms of cancer when used in conjunction with standard therapies. Studies are currently

underway to examine the potential role of Genasense in a variety of clinical indications.

Posted: Sat Jul 10, 2010 12:38 pm Post subject:

Q116. tendency of colonic carcinoma to metastasize is best assessed by

a.size of tr

b.cea levels

c.depth off penetration of bowel wall

d.prop of bowel circumfrence involv

Q117. hair perforation test is done for ;;

1microsporum canis;;

2trichophyton mentagrophyte;;

3epidermophyton;;

4trichophyton rubrum

B. To distinguish between isolates of dermatophytes, particularly Trichophyton mentagrophytes

and its variants.

Ingredients:

Autoclaved blonde pre-pubital hair cut into short pieces (1cm)

Sterile distilled water 5 ml in a suitable vial.

Method:

1. Place hair in water in vial.

2. Inoculate with small fragments of the test fungus.

Page 48: DocumentQ1

3. Incubate at room temperature.

4. Individual hairs are removed at intervals up to 4 weeks and examined microscopically in

lactophenol cotton blue. Isolates of T. mentagrophytes produce marked localised areas of pitting

and marked erosion whereas those of T. rubrum do not.

Q118. septated uteri are most commonly surgically repaired using ;;

1straussman uteroplasty;;

2hysteroscopic resection ;;

3laser laparscopy;;

4tompkins metroplasty

Q119. which of the following primary tumors is most likely to metastasise to the ovary ;;

1breast cancer;;

2thyroid cancer;;

3astrocytoma;;

4osteosarcoma;;

answer breast

Metastatic tumours, accounting for about 5% of ovarian malignancies, and usually arise from

breast, colon, endometrium, stomach and cervical cancers

Q120. in mseleni joint disease -the joint commnly affected is ;;

1knee joint;;

2hip joint;;

3ankle joint;;

4wrist joint

Mseleni joint disease (MJD) is an unusual form of progressive and widespread degenerative

osteoarthropathy that has been identified in several hundred people in the remote Mseleni region

of northern Zululand. Affected individuals experience articular discomfort in childhood and may be

seriously handicapped as adults, often requiring prosthetic hip joint replacement

Q121. allopurinal is used in the treatment of ;;

1trypanosomiasis;;

2leishmaniasis;;

3amoebiasis;;

4trichomoniasis;

Q122. which is not a feature of CARNEY COMPLEX;;

1MYXOMA;;

2PIGMENTATION;;

3SCWANNOMA;;

4THYROID ADENOMA

ANSWER SCHWANNOMA

Carney complex (also known as "LAMB syndrome," and "NAME syndrome") is an autosomal

dominant condition comprising myxomas of the heart and skin, hyperpigmentation of the skin

(lentiginosis), and endocrine overactivity

DIFFERENTIATED FROM

carney triad...

Carney triad, as originally described in 1977, is the association of 3 tumors: gastric

gastrointestinal stromal tumor (gastric GIST) (ex-gastric epithelioid leiomyosarcoma), extra-

Page 49: DocumentQ1

adrenal paraganglioma, and pulmonary chondroma.

gastric stromal tumor in Carney triad showed distinctive features: female predilection, young

patient age, epithelioid cell predominance, multifocality, frequent lymph node metastasis, serial

tumor occurrence, and unpredictable behavior.

A123.

Back to top

vinay. Guest

Posted: Sun Jul 11, 2010 5:19 pm Post subject:

Q123. which of the following is a retinoid selective receptor;;

1tretinoin;;

2alitreinoin;;

3isotretinoin;;

4bexaratone;;

Bexarotene is a retinoid specifically selective for retinoid X

receptors, as opposed to the retinoic acid receptors.

Retinoids have been effective in vitro against many types of

cancer, including that of the breast . However, many breast

cancers become resistant to the chemotherapeutic effects of

these drugs . Recently, RXR-selective ligands were discovered

that inhibited proliferation of all-trans RA resistant breast cancer

cells in vitro and caused regression of the disease in animal

models. These RXR-selective ligands did not produce the side

effects that are normally associated with retinoid chemotherapy

. However, a phase II trial using one of these ligands

(bexarotene) showed limited response in patients with

metastatic breast cancer. Interestingly, a high-risk lesion for

invasive breast cancer, ductal carcinoma in situ, has been

shown to overexpress RXR-α

A retinoid X receptor (RXR)-selective retinoid reveals that RXR-

alpha is potentially a therapeutic target in breast cancer cell

lines, and that it potentiates antiproliferative and apoptotic

responses to peroxisome proliferator-activated receptor ligands.

Q124. systemic disorder associated with hyperprolactinaemia ,

are all , except ;;

1CRF;;

2CIRRHOSIS;;

3EPILEPSY;;

4CHF;;

Q125. WHICH OF THE FOLLOWING SITES CONTAINS -striated

muscle that is not under voluntary control;;

1bladder;;

2esophagus;;

3gallbladder;;

Page 50: DocumentQ1

4 stomach;

Esophageal skeletal muscle is under control of motor neurons

originating in the nucleus ambiguus in the brain stem,

The cervical esophagus is composed of striated muscle that is

directly innervated by lower motor neurons

Q126. all of the following are true about -FAP. EXCEPT ;;

1fundic polyps are not premalignant;;

2the gene associates is a tumor suppressor gene;;

3if left untreayed colonic cancer occurs in virtually 100 percent

affected;;

4is the most common hereditary colorectal cancer

Q127. MOST SENSITIVE METHOD FOR DIAGNOSING CHRONIC

PANCREATITIS;;

1CECT ABDOMEN;;

2MRI;;

3ERCP;;

4EUS;;

Q128. STEINSTRASSE OCCURRENCE CAN BE COUNTERED BY ;;

1doing a ureterostomy;;

2placing a ureteric stent;;

3doing multiple repeated basketting procedures;;

4none of the above;;

Percutaneous nephrostomy

tube placement to the patient under local anesthesia will provide

symptomatic relief. With proximal diversion, the steinstrasse

often passes spontaneously.

Back to

top

dishan. Guest

Posted: Mon Jul 12, 2010 12:58 pm Post subject:

Q126.

Explanation

Gastric polyps are usually an incidental finding on endoscopy, detected in 2% to 3% of gastroscopic evaluations. Fundic gland

polyps constitute 47% of all gastric polyps and have no malignant potential. Typically, they present as multiple 2- to 3-

sessile lesions in the body and fundus, most commonly in healthy gastric mucosa.

Most cases are sporadic, but gastric polyps can occur in 53% of patients with familial adenomatous polyposis or Gardner's

syndrome. Although the polyps themselves are non-neoplastic, retrospective studies have reported colorectal neoplasm

to 60% of patients with gastric fundic gland polyps

Tumor suppressor genes produce proteins that inhibit tumor formation by regulating mitotic activity and providing inhibitory

cycle control. Tumor formation occurs when these inhibitory controls are deregulated by mutation. Point mutations, loss of

heterozygosity (LOH), frame-shift mutations, and promoter hypermethylation are all types of genetic changes that can cause

failure of a tumor suppressor gene. These genes are often referred to as gatekeeper genes because they provide cell cycle

inhibition and regulatory control at specific checkpoints in cell division. The failure of regulation of normal cellular function by

tumor suppressor genes is appropriately described by the term loss of function. Both alleles of the gene must be nonfunctional

to initiate tumor formation.

Page 51: DocumentQ1

The adenomatous polyposis coli (APC) gene is a tumor suppressor gene located on chromosome 5q21.

Its product is 2843 amino acids in length and forms a cytoplasmic complex with GSK-3β (a serine-threonine kinase), β

and axin. β-Catenin, a multifunctional protein, is a structural component of the epithelial cell adherens junctions and the actin

cytoskeleton; it also binds in the cytoplasm to Tcf/LEF and is then transported into the nucleus, where it activates transcription

of genes like c-myc and others that regulate cellular growth and proliferation.

APC therefore participates in cell cycle control by regulating the intracytoplasmic pool of β-catenin.

Wnt signaling proteins are closely associated with the APC-β-catenin pathway. APC also influences cell cycle proliferation by

regulating Wnt expression. Wnt gene products are extracellular signaling molecules that help regulate tissue development

throughout the organism. The Wnt signaling proteins are closely associated with the APC-β-catenin pathway. Under normal

conditions, reduced intracytoplasmic β-catenin levels inhibit Wnt expression.

When APC is mutated however, β-catenin levels rise, and Wnt is activated. Overexpression of Wnt leads to activation of Wnt

target genes such as cyclin D1 and Myc, which drive cell proliferation and tumor formation

earliest mutations in the adenoma-carcinoma sequence occur in the APC gene.

###NEW MCQ###

The earliest phenotypic change present is known as aberrant crypt formation, and the most consistent genetic aberrations

within these cells are abnormally short proteins known as APC truncations. Most clinically relevant derangements in APC are

truncation mutations created by inappropriate transcription of premature termination codons.

A germline APC truncation mutation is responsible for the autosomal dominant–inherited disease, FAP.

Thirty percent of cases of FAP are de novo germline mutations, thus presenting without a family history of the disease. FAP is

rare, with an estimated incidence of 1 in 8000 of the U.S. population, occurring without gender predilection

. It is classically characterized by greater than 100 adenomatous polyps present in the colon and rectum. These polyps often

number in the thousands and are almost always manifest by the late second or early third decade of life

AIIMS MAY 2010

The variability of the FAP phenotype is also demonstrated by the presence or absence of extraintestinal manifestations

disease. In the past, the term Gardener's syndrome was used to describe the coexpression of profuse colonic adenomatous

polyps along with osteomas of the mandible and skull, desmoid tumors of the mesentery, and periampullary neoplasms.

Many other associated disorders have been subsequently described, including thyroid papillary tumors

medulloblastomas,

hypertrophic gastric fundic polyps, and

congenital hypertrophy of the retinal pigmented epithelium of the iris (CHRPE)

. The expression of extraintestinal manifestations of FAP is dependent on mutation location, with most of these signs seen only

when the truncation occurs in a very small area of the mutational cluster region

IMP MCQ POINTS

Larger adenomas are found to harbor cancers more often than smaller ones, and the larger the polyp, the higher the risk for

cancer.

Although the cellular characteristics of the polyp are important, with villous adenomas carrying a higher risk than tubular

adenomas, the size of either polyp is also important. The risk for cancer in a tubular adenoma smaller than 1 cm in diameter is

less than 5%, whereas the risk for cancer in a tubular adenoma larger than 2 cm is 35%. A villous adenoma larger than 2 cm in

size carries a 50% chance of containing a cancer

Page 52: DocumentQ1

Removal of polyps decreases the incidence of cancer. Patients with small adenomas have a 2.3 times increased risk for cancer

after the polyp is removed, compared with an 8-fold increased incidence of colorectal cancer in patients with polyps who do not

undergo polypectomy

Patients with FAP will develop colorectal cancer virtually 100% of the time in the absence of surgical intervention. ##choice

this mcq##

The adenomas that characterize this syndrome are histologically the same as sporadic adenomas.

The peak incidence for the discovery of benign colorectal polyps is 50 years of age. The peak incidence for the development of

colorectal cancer is 60 years of age. This suggests a 10-year time span for the progression of an adenomatous polyp to a

cancer. It has been estimated that a polyp larger than 1 cm has a cancer risk of 2.5% in 5 years, 8% in 10 years, and 24% in

20 years.

Rare extraintestinal malignancies in FAP patients include cancers of the extrahepatic bile ducts, gallbladder, pancreas, adre

thyroid, and liver.

An interesting marker for FAP is CHRPE, which can be detected by indirect ophthalmoscopy in about 75% of affected

individuals.

The gene is expressed in 100% of patients with the mutation. Autosomal dominance results in expression in 50% of offspring.

There is a negative family history in 10% to 20% of affected individuals, who apparently acquire the syndrome as the result o

a spontaneous mutation. All patients with the defective gene will develop cancer of the colon if left untreated.

The average age of discovery of a new patient with FAP is 29 years. The average age of a patient who is newly discovered to

have colorectal cancer related to FAP is 39 years.

Eponymous polyposis syndromes now recognized to belong to the general disorder of FAP include Gardner's syndrome (colonic

polyps, epidermal inclusion cysts, osteomas) and Turcot's syndrome (colonic polyps and brain tumors).

Osteomas usually present as visible and palpable prominences in the skull, mandible, and tibia of individuals with FAP. They are

virtually always benign. Radiographs of the maxilla and mandible may reveal bone cysts, supernumerary and impacted molars,

or congenitally absent teeth.

Desmoid tumors can present in the retroperitoneum and abdominal wall of affected patients, usually after surgery. These

tumors seldom metastasize but are often locally invasive, and direct invasion of the mesenteric vessels, ureters, or walls of

small intestine can result in death.

SURGERY

Surgical treatment of patients with FAP is directed at removal of all affected colonic and rectal mucosa. Restorative

proctocolectomy with IPAA has become the most commonly recommended operation. The procedure is usually accompanied by

a distal rectal mucosectomy to ensure that all premalignant colonic mucosa is removed, and the IPAA is fashioned between the

ileal pouch and the dentate line of the anal canal. Patients who undergo this procedure for FAP have a better functional resu

than patients similarly treated for ulcerative colitis, in that the incidence of inflammation in the ileal pouch (pouchitis) is much

lower in patients with FAP than in patients with ulcerative colitis.

less spreaded disease ..... abdominal colectomy and ileorectal anastomosis is the observation that sulindac and celecoxib have

been observed to cause the regression of adenomatous polyps in some patients with FAP. The disadvantages are that the

rectum remains at high risk for the formation of new precancerous polyps, a proctoscopic examination is required every 6

months to detect and destroy any new polyps, and there is a definite increased risk for cancer arising in the rectum with the

Page 53: DocumentQ1

passage of time

There have been sporadic reports of regression of desmoid tumors after treatment with sulindac, tamoxifen, low-dose

methotrexate, radiation, and various types of chemotherapy. The initial treatment is usually with sulindac or tamoxifen.

#########HNPCC is the most frequently occurring hereditary colorectal cancer syndrome in the United States and Western

Europe. It accounts for about 3% of all cases of colorectal cancer and for about 15% of such cancers in patients with a family

history of colorectal cancer###########

ANSWER IS CHOICE D

Back to

top

dishan. Guest

Posted: Mon Jul 12, 2010 12:58 pm Post subject:

Q127.

Explanation

Radiographs or CT scans showing pancreatic calcifications are diagnostic of chronic pancreatitis . Those calcifications refle

deposition of calcium carbonate in the intraductal protein plugs that frequently, but not invariably, occur in chronic pancreatitis.

Thus, the absence of pancreatic calcifications does not rule out a diagnosis of chronic pancreatitis.

Perhaps the most sensitive methods for diagnosing chronic pancreatitis are those that provide images of the pancreatic ductal

system. ERCP, CT cholangiopancreatography, or magnetic resonance cholangiopancreatography may be particularly valuable in

the diagnosis of chronic pancreatitis

chronic pancreatitis can be shown to have major ducts that have the appearance of a “chain of lakes” or a “string of pearls” that

is the result of segments of dilated duct separated by areas of ductal stricture. Transcutaneous and endoscopic ultrasound ca

also be used to diagnose chronic pancreatitis if duct dilation, calcifications, pseudocysts, or parenchymal fibrosis are seen.

Ultrasound examination is more operator dependent and perhaps less sensitive than either CT or MRI

Back to

top

vinay. Guest

Posted: Mon Jul 12, 2010 1:04 pm Post subject:

Q129. which of the following artery shows maximum frequency of arteriographic abnormalities in takayasu arteritis;;

1subclavian artery;;

2aortic arch;;

3common carotid artery;;

4 coronary artery;

answer def subclavian artery

Takayasu's arteritis is an inflammatory and stenotic disease of medium- and large-sized arteries characterized by a strong

predilection for the aortic arch and its branches. For this reason, it is often referred to as the aortic arch syndrome.

most prevalent in adolescent girls and young women. Although it is more common in Asia, it is neither racially nor

geographically restricted.

The disease involves medium- and large-sized arteries, with a strong predilection for the aortic arch and its branches; the

pulmonary artery may also be involved. The most commonly affected arteries seen by angiography are listed in Table 319

The involvement of the major branches of the aorta is much more marked at their origin than distally

Page 54: DocumentQ1

The disease is a panarteritis with inflammatory mononuclear cell infiltrates and occasionally giant cells. There are marked

intimal proliferation and fibrosis, scarring and vascularization of the media, and disruption and degeneration of the elastic

lamina. Narrowing of the lumen occurs with or without thrombosis. The vasa vasorum are frequently involved. Pathologic

changes in various organs reflect the compromise of blood flow through the involved vessels.

Disease-related mortality most often occurs from congestive heart failure, cerebrovascular events, myocardial infarction,

aneurysm rupture, or renal failure. Even in the absence of life-threatening disease Takayasu's arteritis can be associated with

significant morbidity. The course of the disease is variable, and although spontaneous remissions may occur, Takayasu's

arteritis is most often chronic and relapsing. Although glucocorticoid therapy in doses of 40–60 mg prednisone per day

alleviates symptoms, there are no convincing studies that indicate that they increase survival. The combination of glucocorti

therapy for acute signs and symptoms and an aggressive surgical and/or angioplastic approach to stenosed vessels has

markedly improved outcome and decreased morbidity by lessening the risk of stroke, correcting hypertension due to renal

artery stenosis, and improving blood flow to ischemic viscera and limbs. Unless it is urgently required, surgical correction of

stenosed arteries should be undertaken only when the vascular inflammatory process is well controlled with medical therapy. I

individuals who are refractory to or unable to taper glucocorticoids, methotrexate in doses up to 25 mg per week has yielded

encouraging results

Takayasu's arteritis (also known as "Aortic arch syndrome," and "Pulseless disease":841) is an inflammatory disease with an

unknown cause. It affects the aorta, the main blood vessel from the heart, as well as the blood vessels that attach to it.

Although it has been reported worldwide, it is more common in young Asian women. Females are about 8-9 times more likely to

get it than males. People usually get the disease between 15 and 30 years of age. In the Western world, atherosclerosis is a

more frequent cause of obstruction of the aortic arch vessels than is Takayasu's arteritis

Q130. the nucleus involved in papez circuit is ;;

1pulvinar;;

2intralaminar;;

3vpl nucleus;;

4ant nucleus;;

Q131. structure that cross the midline is ;;

1arteirathyroidea ima;;

2median sacral artery;;

3zenkers diverticulum;;

4left gonadal vein;;

median sacral artery (or middle sacral artery) is a small vessel, which arises from the back of the aorta, a little above its

bifurcation.

It descends in the middle line in front of the fourth and fifth lumbar vertebræ, the sacrum and coccyx, and ends in the glomu

coccygeum (coccygeal gland).

From it, minute branches are said to pass to the posterior surface of the rectum.

On the last lumbar vertebra it anastomoses with the lumbar branch of the iliolumbar artery; in front of the sacrum it

anastomoses with the lateral sacral arteries, and sends offsets into the anterior sacral foramina.

Q132. DALBAVANCIN - IS MAINLY BEING USED FOR WHICH INFECTION;;

1PSEUDOMONAS;;

2MRSA;;

3KLEBSIELLA;;

4ANAEROBE INF

Dalbavancin is a novel second-generation lipoglycopeptide antibiotic. It belongs to the same class as vancomycin, the most

widely-used and one of the few treatments available to patients infected with methicillin-resistant Staphylococcus aureus

(MRSA).

It possesses in vitro activity against a variety of Gram-positive pathogens including MRSA and MRSE It is a once-weekly, two

dose antibiotic .

Page 55: DocumentQ1

Q133. COMPARED TO the other antidepressant drugs , mirtazapine has distinct ability to act as an antagonist of ;;

1beta receptors;;

2d2 receptors;;

3alpha 2 receptors;;

45-ht receptors;;

answer 3

mitrazapine tetracyclic antidepressant (TeCA) used primarily in the treatment of depression. It is also sometimes used as an

anxiolytic, hypnotic, antiemetic, appetite stimulant, and antihistamine, among other indications. Along with its close analog

mianserin and setiptiline, mirtazapine is one of the few noradrenergic and specific serotonergic antidepressants (NaSSAs)

no significant act on dopamine receptor

Q134. TERMINAL PHALANX RESORPTION IS NOT SEEN IN ;;

1HYPERPARATHYROIDISM;;

2REITERS SYNDROME;;

3SCLERODERMA;;

4PSORIASIS;;

Q135. ALL OF THE FOLLOWING CAN BE DONE FOR FEMORAL HERNIA REPAIR, EXCEPT ;;

1lockwood repair;;

2mc evedy repair;;

3lothessions repair;;

4beclard repair;

Lockwood’s infra-inguinal approach

Lotheissen‘s trans-inguinal approach

McEvedy’s high approach

answer is obviously beclard repair..

beclard is hernia thru saphenous opening

Lockwood is remembered for his surgical work with femoral and inguinal hernias. He developed an infra-inguinal approach for

femoral hernia operations that is known today as the low approach or Lockwood's operation

Classically three approaches are described to open femoral hernia repair: Lockwood's infra-inguinal,

Lotheissen's trans-inguinal and

McEvedy's high approach.

Each approach describes a separate skin incision and dissection to access the femoral sac.

lockwood repair...

The infra-inguinal approach is the preferred method for elective repair, approaching the femoral canal from below through an

oblique incision 1 cm below and parallel to the inguinal ligament. This approach however offers little scope for resecting any

compromised bowel.

lotheissen ...

The trans-inguinal approach involves a skin incision 2 cm above the inguinal ligament, dissecting through the inguinal canal and

thus weakening this important structure. The danger with this, particularly in the presence of wound infection, is that a hernia

may form later which would be difficult to repair. In addition, if necrotic bowel is encountered the risk of infection may pr

the use of synthetic mesh to repair the inguinal canal and predispose to inguinal hernia occurrence.

mc evedy....

high approach involves an oblique skin incision 3 cm above the pubic tubercle running laterally to cross the lateral border o

Page 56: DocumentQ1

rectus muscle, that is divided allowing preperitoneal dissection of the sac. This approach is preferred in the emergency setting

when strangulation is suspected allowing better access to and visualisation of bowel for possible resection

Q136. A 40 yr old female p3+0 is observed to have CIN grade 3 on colposcopic biopsy .which is best management? (AI

09/AIIMS 10)

a)cryosurgery

b)conization

c)LEEP

d)Hysterectomy

Back to

top

vinay. Guest

Posted: Tue Jul 13, 2010 1:07 pm Post subject:

Q136. glycoproteins are degraded by ;;

1ubiquitins;;

2cathepsins;;

3lysosomes;;

4peroxisomes;;

Q137. master slave system is related to ;;

1SILS;;

2NOTES;;

3HALS;;

4ROBOTS;;

Q138. which of the following 5ht 3 receptor blockers has highest receptor affinity;;

1ondansetron;;

2granisetron;;

3dolaserton;;

4palonosetron;;

Q139. which is not correct with ref to histopathology of psoriasis;;

1hyperkeratosis;;

2hypergranulosis;;

3parakeratosis;;

4microabscess;;

Psoriasis is a chronic relapsing disease of the skin characterized by variable clinical features. The cutaneous lesions are u

so distinct that a clinical diagnosis is easy to make. The lesions are classified as erythrosquamous, which indicates that bo

vasculature (erythema) and the epidermis (increased scale formation) are involved.

Inverse psoriasis . The skin in the intertriginous areas is highly erythematous, and typical scaling is lacking.

A total of 2147 patients suffering from various types of psoriasis were examined and the age of onset was recorded. Most

patients of both sexes had an early onset [age 16 (F) and 22 (M)], while nearly one-quarter had a late onset (age 56).

Psoriasis also may present in a pustular form. There is a generalized form, usually referred to as pustular psoriasis (von

Zumbusch), and a localized variant, confined to the palms and soles, known as pustulosis palmaris et plantaris (PPP; see Chap.

70). In rare instances in psoriasis of the plaque type or guttate psoriasis, pustules may develop after acute relapses (psori

with pustules)

###new mcq###

First signs appear in males at a mean age of 29 and in females at age 27. A study of the onset of psoriasis in 2400 patients

showed a peak incidence at 22.5 years of age; a second peak of onset around age 55 was found in 11.8 percent of the patients

Page 57: DocumentQ1

4 (see Fig. 42-5A).

In a study of psoriasis in 245 children, the mean age of onset was 8.1 years, 5 and in a census study from the Faeroe Islands,

the mean age of onset was 12.5 years.

An early onset (before age 15) predicts more severe disease relative to the percentage of body surface involved with psoriasis

and the response to therapy. Also, the earlier the onset, the greater is the probability of a positive family history of psor

The HLA types most frequently reported to be associated with psoriasis are HLA-B13, HLA-Bw57, HLA-Cw6, and HLA-DR7.

hla cw6 newly discoverd and mc in india ref iams

CLINICAL POINTS

four prominent features:

(1) they are sharply demarcated with clear-cut borders,

(2) the surface consists of noncoherent silvery scales,

(3) under the scale the skin has a glossy, homogeneous erythema,

(4) there is an Auspitz sign.

The size of a single lesion varies from a pinpoint to plaques that cover large areas of the body. The clinical presentation o

psoriasis is better understood when it is realized that disease activity can range from a chronic stationary phase, to a resolving

process, to flares of disease that may be associated with numerous sterile pustules. The Auspitz sign is a specific feature o

erythrosquamous lesion of psoriasis. It is noted when the hyperkeratotic scales are mechanically removed from a psoriatic

plaque by scraping. Within a few seconds after mechanical removal of the scale, small blood droplets appear on the shiny

erythematous surface ). The Auspitz sign has diagnostic value; it is not present in inverse or pustular psoriasis and may help to

differentiate psoriasis from other skin conditions with a similar morphology.

The Auspitz phenomenon in its three phases: . Native psoriatic lesion.

1 Scratching generates silvery-opaque scale.

2. Further scratching leads to removal of the scale, and a glossy area is visible.

3 Further scratching produces blood droplets

1.

Pits are evident within the nail plate. This morphologic pattern apparently is due to defective keratinization of the dorsal side of

the proximal nail fold

2.

Yellowish macules beneath the nail plate often extend distally toward the hyponychium. This morphologic pattern appears to be

caused by psoriatic processes located in the nail bed

3.

Severe onychodystrophy results in yellowish keratinous material. This morphologic pattern is believed to be secondary to

psoriasis involving the nail matrix

Common Laboratory Abnormalities in Psoriasis

Serum uric acid is elevated in up to 50 percent of patients and is mainly correlated with the extent of lesions and the activity of

disease. There is an increased risk of developing gouty arthritis. Elevated serum uric acid levels usually cease after therap

In severe psoriasis vulgaris, generalized pustular psoriasis, and erythroderma, a negative nitrogen balance can be detected,

mainly consisting in a decrease of serum albumin.

C-reactive protein, a 2-macroglobulin, as well as sedimentation rate, can be increased in psoriasis related to disease activity

and body involvement

Page 58: DocumentQ1

. Increased serum IgA levels and IgA immune complexes have been observed in psoriasis, the role of this phenomenon still

being unclear

mild anemia

Histopathology of psoriasis.

A. The earliest changes in psoriasis consist of papillary edema with a marked infiltrate of mononuclear cells, which tend to

invade the spongiotic epidermis. Later, neutrophils follow.

B. Pustular psoriasis with large spongiform abscesses in the subcorneal zone without marked epidermal hyperplasia.

C. Neutrophils migrating into the stratum corneum form so-called Munro's microabscesses.

D. In plaque-type psoriasis, large numbers of CD3+ T cells are present and tend to migrate into the affected epidermal

portions

characteristic feature of involved skin of psoriatic subjects is hyperproliferation, first defined by Van Scott and Ekel.

Currently, there is evidence of more than an eightfold shortening of the epidermal cell cycle (36 h versus 311 h for normal)

involved skin of patients with psoriasis. Further, there is a twofold increase in the proliferative cell population, and 100 percent

of the germinative cells of the epidermis appear to enter the growth fraction compared with 60 to 70 percent for normal

subjects. These alterations result in a hyperplastic epidermis generating 35,000 cells/mm per day from a proliferative

compartment containing approximately 52,000 cells/mm of skin surface. Normal skin produces only 1218 cells/mm per day

from a proliferative compartment of 26,000 cells/mm

****** Clinical Classification of Psoriasis******

PSORIASIS VULGARIS, CHRONIC STATIONARY PSORIASIS, PLAQUE-TYPE PSORIASIS This clinical pattern is the most frequent.

Red scaly lesions, as described earlier, persist for months to years. There is constant production of large amounts of sc

little alteration in shape or distribution of individual plaques. Areas of predilection are the

elbows ,

the knees,

the scalp and, in particular,

the retroauricular region,

the lumbar area

umbilicus.

Single small lesions may become confluent, forming plaques in which the borders resemble a land map (psoriasis geographica)

(see Fig. 42-3B). Lesions may extend laterally and become circinate because of the confluence of several plaques (psoriasis

gyrata). Occasionally there is partial central clearing, resulting in ringlike lesions (annular psoriasis)

Eruptive (guttate) psoriasis

Typically, this pattern presents as small (0.5–1.5 cm in diameter) lesions over the upper trunk and proximal extremities. This

form is characteristic of psoriasis of an early age of onset and as such is found frequently in young adults. As noted below

streptococcal throat infection frequently precedes the onset or flare of guttate psoriasis. Occasionally, a disseminated macu

drug eruption may precede this pattern of psoriasis. Very active lesions of psoriasis of many types can have pustules that are 1

to 2 mm in diameter and are surrounded by an intensive wall of erythema. This process usually signals an acute exacerbation

of disease.

Predisposing factors for such an event are bacterial infection, aggressive local therapy, or withdrawal of systemic

glucocorticoids

Psoriatic erythroderma Psoriatic erythroderma

Page 59: DocumentQ1

represents the generalized form of the disease that affects all body sites, including the face, hands, feet, nails, trunk, and

extremities. [/b ]Although all the symptoms of psoriasis are present, erythema is the most prominent feature, and scaling

usually is less severe compared with chronic stationary psoriasis. Psoriatic erythroderma may have different degrees of disease

activity, presenting suddenly as a generalized erythema or evolving gradually from chronic plaque psoriasis into a generalize

exfoliative phase. In the latter phase there are usually some areas of uninvolved skin.

Annular pustular psoriasis

A rare variant of pustular psoriasis is an annular, or circinate, form of the disease occurring during episodes of pustular

eruptions. Lesions may appear at the onset of pustular psoriasis, with a [b]tendency to spread and form enlarged rings, or

may develop during the course of generalized pustular psoriasis.

The main features are pustules on a ringlike erythema that sometimes resembles erythema annulare centrifugum.

Histologically, there is mild acanthosis and a neutrophil accumulation with formation of microabscesses.

Identical lesions are found in patients with impetigo herpetiformis, a pustular form of psoriasis associated with pregnancy

Generalized pustular psoriasis (von Zumbusch)

Pustular psoriasis of the von Zumbusch type ( Fig. 42-11) appears as a distinctive acute variant of psoriasis. It is unusual to see

other forms of psoriasis on the skin at the same time. Attacks of pustular psoriasis are characterized by fever that lasts se

days. A sudden generalized eruption of sterile pustules 2 to 3 mm in diameter parallels the onset of fever. The pustules are

disseminated over the trunk and extremities, including the nail beds, palms, and soles. The pustules usually arise on highly

erythematous skin, first as patches and then becoming confluent as the disease becomes more severe.

In addition to the pustule formation of the nail matrix and loss of the entire nail, the fingertips may become atrophic in pa

with prolonged disease. As with other forms of psoriasis, the face usually remains free of lesions. The erythema that surrounds

the pustules often spreads and becomes confluent, leading to erythroderma

Topical Treatment

ANTHRALIN Anthralin (1,8-dihydroxyanthrone; cignolin, dithranol) was introduced by Galewsky and Unna in 1916. This

compound is still a widely used remedy for psoriasis in different vehicles and application modes. A major advantage of anthra

is the lack of any long-term side effects,

Mode of action Anthralin possesses antiproliferative activity on human keratinocytes. In recent years it has become clear that

this compound also exerts strong anti-inflammatory effects mainly on cells of the inflammatory infiltrate. Inhibition of neutrophil

and monocyte functions and production and ?-oxidation of leukotriene B 4 from neutrophils have been observed. It was found

recently that anthralin induces nuclear transcription factor NF-? B in murine keratinocytes. Since NF-?B is involved in the

transcription of proinflammatory cytokines such as IL-6, IL-8, and TNF-a, these findings may be helpful in explaining the irritant

properties of anthralin.

VITAMIN D 3 AND ANALOGUES Since the first report about the beneficial effects of vitamin D 3 in psoriasis by Morimoto et al.,

new analogues have been developed to decrease hormonal effects on calcium/phosphate homeostasis and to maintain effects

on keratinocyte proliferation and differentiation. Soon after the introduction of the first vitamin D 3 analogue, calcipotrio

called calcipotriene in some countries), to topical therapy, it became a widely used remedy for plaque-type psoriasis. Another

vitamin D 3 analogue, tacalcitol, is available in some countries. Recently, active hormone 1,25-dihydroxyvitamin D 3 (calcitriol)

was registered for topical psoriasis treatment in a number of countries.

Mode of action Vitamin D 3 and its analogues inhibit keratinocyte proliferation and induce terminal differentiation. Anti-

inflammatory properties of these compounds include inhibition of nuclear factor NF-?B protein in lymphocytes, leading to a

reduced transcription of IL-2. Calcitriol and calcipotriol can inhibit production of IL-6 from cytokine-stimulated human dermal

microvascular endothelial cells and reduce the antigen-presenting function of Langerhans cells. Calcitriol is a potent inhibitor of

dendritic cell differentiation

TAZAROTENE Tazarotene is a retinoid for topical use that reduces mainly scaling and plaque thickness, with limited

Page 60: DocumentQ1

effectiveness on erythema. In comparison with other topical drugs for psoriasis, tazarotene has a lower efficacy. However,

efficacy can be enhanced by combination with UVB therapy.

TAR The use of tar either as coal tar or wood tar (birch, pine, beech) has a long history in antipsoriatic therapy. These tar

contain a great variety of compounds, most of which are not well defined. Very little is known about how tars act in skin.

Preparations of 2% to 5% tar in various bases have been shown to be preferentially effective in chronic plaque-type psoriasis.

These preparations are nonirritant, and serious side effects are not seen even after long periods of treatment.

BALNEOPHOTOTHERAPY Empirically, it has been known that the combination of salt-water bathing and sunlight exposure is an

effective treatment for psoriasis

SELECTIVE UVB THERAPY Treatment with UVB without UVA, also known as selective UVB phototherapy (SUP), can be

performed as monotherapy or preferably in combination with topical treatments such as glucocorticoids, vitamin D 3 and

analogues, tazarotene, or anthralin. SUP is very effective in guttate psoriasis and also improves lesions of the plaque type.

Today, narrow-band UVB treatment (311 nm, Philips TL01 bulbs) has become a standard therapy for plaque-type and guttate

psoriasis.

systemic therapy mtx,cyclosporine.glucocorticoids,

retinoids lik.. Etretinate and acitretin are clinically effective in pustular forms of psoriasis

FUMARIC ACID ESTERS A mixture of fumaric acid monoethyl and dimethyl esters is approved in Germany for systemic

treatment of severe psoriasis. Clinical experience exists since 1959 when fuamrates were introduced.

Mode of action Inhibition of TNF-a–induced keratinocyte ICAM-1 expression by dimethylfumarate has been demonstrated.

Monomethylfumarate, the main metabolite of dimethylfumarate, was shown to stimulate release of T H2 cytokines IL-4 and IL

5 from human peripheral blood T cells without changing production of T H1 cytokines IL-2 and interferon-?.

Dimethylfumarate potently inhibits dendritic cell differentiation. Recently, dimethylfumarate was shown to induce apoptosis is a

number of cells, including dendritic cells. Furthermore, this compound inhibits cytokine production by blocking NF?B signalin

BATH PUVA

Another way to deliver the photosensitizer (8-MOP or 5-MOP) to the skin is by addition of these compounds to bath water, first

described by Fischer and Alsins in 1976. Major advantages of bath PUVA are the lack of systemic effects, such as

gastrointestinal complaints (nausea is present in about 13 percent of patients taking 8-MOP orally), and the overall reduction of

UV dose down to one-quarter of that required to obtain therapeutic results similar to those of conventional PUVA, thus reducing

the risk of nonmelanoma skin cancer. Furthermore, erythema is less frequent in bath PUVA patients, and eye protection by

sunglasses is not required.

Q140. WHICH OF THE FOLLOWING cytokine downregulates the immunological response;;

1il-1;;

2il-2;;

3il-6;;

4 il-10

Interleukin-10 (IL-10 or IL10), also known as human cytokine synthesis inhibitory factor (CSIF), is an anti-inflammatory

cytokine. In humans IL-10 is encoded by the IL10 gene.

This cytokine is produced primarily by monocytes and to a lesser extent by lymphocytes. This cytokine has pleiotropic effects

immunoregulation and inflammation. It down-regulates the expression of Th1 cytokines, MHC class II antigens, and

costimulatory molecules on macrophages.

Q141. ORNITHINE CITRULLINE TRANSPORTER DEFECT CAUSES ALL, EXCEPT ;;

1HYPERAMMONAEMIA;;

2HYPERORNITHINEMIA;;

Page 61: DocumentQ1

3HYPERHOMOCITRULLINEMIA;;

4HYPERCITRULLINAEMIA;;

Q142. THE WORLD HEALTH DAY THEME FOR THE YEAR 2010 IS ;;

1HEALTH FACILITIES IN EMERGENCIES;;

2PROTECT HEALTH FROM CLIMATE CHANGES;;

3URBANISATION AND HEALTH;;

4 NONE OF THE ABOVE;;

Q143. THE POSTEROMEDIAL corner of the knee joint is very vital to its rotatory stability ;;it includes all, except ;;

1post oblique ligament;;

2multiple insertions of semimembranosus;;

3oblique popliteal ligament;;

4the arcuate ligament;;

Q144. the first response of osmoreceptors to hyperosmolality is

1renin secretion;;

2 thirst;;

3vasopressin secretion;;

4 none;;

Back to

top

vinay. Guest

Posted: Wed Jul 14, 2010 12:59 pm Post subject:

Q145. treatment for diabetic retinopathy is ;;

1primordial prevention;;

2primary prevention;;

3sec prevention;;

4tertiary prevention

Q146. carisoprodol activation forms -which metabolite ;;

1amphetamine;;

2meprobomate;;

3doxylamine;;

4dimethadione;;

meprobomate

Carisoprodol is a centrally-acting skeletal muscle relaxant. It is a colorless, crystalline powder, having a mild characteristic odor

and a bitter taste. Carisoprodol is slightly soluble in water and freely soluble in alcohol, chloroform and acetone. The drug

solubility is practically independent of pH.

carisoprodol has abuse potential as a product of meprobamate and/or potentiator of hydrocodone, dihydrocodeine, codeine and

similar drugs, it continues to be prescribed in North America, alongside orphenadrine and cyclobenzaprine. In Europe, doctors

favor cyclobenzaprine. In the United Kingdom, benzodiazepines are preferred instead. All of the above plus chlorzoxazone are

used in Canada.

As of November 2007, carisoprodol (Somadril, Somadril comp.) has been taken off the market in Sweden due to problems with

dependence and side effects. The agency overseeing pharmaceuticals has considered other drugs used with the same

indications as carisoprodol to have the same or better effects without the risks of the drug. In May 2008 it was taken off th

market in Norway as well

Q147. finger in glove appearance on cect chest is found in ;;

1sarcoidosis;;

2emphysema;;

Page 62: DocumentQ1

3allergic bronchopulmonary aspergillosis;;

4interstitial lung diseases;;

The finger in glove sign seen on CXR and CT chest and refers to the characteristic sign of a bronchocoele, as seen in alle

bronchopulmonary aspergillosis (ABPA). Rarely a similar appearance can occur with bronchial atresia.

The same appearance has also been referred to as:

rabbit ear appearance

Mickey Mouse appearance

toothpaste shaped opacities

Y-shaped opacities

V-shaped opacities

Q148. maddery score is used as prognostic indicator in ;;

1chronic hepatitis b;;

2chronic hepatitis c;;

3alcoholic hepatitis;;

4primary biliary cirrhosis;;

Maddrey’s Discriminant Function for Alcoholic Hepatitis

Pt's Prothrombin Time (PT) sec

Lab Control Prothrombin Time (Pt) sec

Pt's Serum Total Bilirubin

Discriminant Function = 4.6 * (Pt's PT - Control PT) + TBili

>32 points indicates poor prognosis and pt may benefit from glucocorticoid therapy.

the modified Maddrey's discriminant function) was originally described by Maddrey and Boitnott to predict prognosis in alcoholic

hepatitis. It is calculated by a simple formula:

Prospective studies have shown that, it is useful in predicting short term prognosis especially mortality within 30 days. A value

more than 32 implies poor outcome with one month mortality ranging between 35% to 45%.

Back to

top

vinay. Guest

Posted: Thu Jul 15, 2010 12:48 pm Post subject:

Q149. which one of the GAGS plays an imp role in the develpment of atherosclerotic plaque;;

1hyaluronic acid;;

2chondroitin sulphate;;

3dermatan sulphate;;

4heparin

The International HapMap Project is an organization whose goal is to develop a haplotype map (HapMap) of the human

genome, which will describe the common patterns of human genetic variation. The HapMap is expected to be a key resource for

researchers to find genetic variants affecting health, disease and responses to drugs and environmental factors

Entrez Gene is NCBI's repository for gene-specific information

Blastx Use the BLAST algorithm to compare the six-frame conceptual translation products of a nucleotide query sequence (both

strands) against a protein sequence database. The BLAST (Basic Local Alignment Search Tool) programs have been designed

for speed to find high scoring local alignments. BLAST uses a heuristic algorithm which seeks local as opposed to global

alignments and is therefore able to detect relationships among sequences which share only isolated regions of similarity

(Altschul et al., 1990). Because of its design for speed, there may be a minimal loss of sensitivity to distant sequence

relationships.

Page 63: DocumentQ1

Hyaluronic acid may be important in permitting tumor cells to migrate through the ECM. Tumor cells can induce fibroblasts to

synthesize greatly increased amounts of this GAG, thereby perhaps facilitating their own spread. Some tumor cells have less

heparan sulfate at their surfaces, and this may play a role in the lack of adhesiveness that these cells display. The intima

arterial wall contains hyaluronic acid and chondroitin sulfate, dermatan sulfate, and heparan sulfate proteoglycans. Of these

proteoglycans, dermatan sulfate binds plasma low-density lipoproteins. In addition, dermatan sulfate appears to be the major

GAG synthesized by arterial smooth muscle cells. Because it is these cells that proliferate in atherosclerotic lesions in arteries,

dermatan sulfate may play an important role in development of the atherosclerotic plaque.

Q150. the genetic factors that contribute to susceptibility to inf are elucidated by ;;

1HAPMAP;;

2ENTREZ GENE;;

3BLASTX;;

4RFLP;;

Q151. dr milind has ansered q no 150 correctly;;

which of the following diseaqse worsens during pregnancy;;

1ulcerative colitis;;

2thyrotoxicosis;;

3rheumatoid arthritis;;

4bronchial asthma

Rheumatoid arthritis: RA may begin during pregnancy or, even more often, during the postpartum period. Preexisting RA

generally abates temporarily during pregnancy. The fetus is not specifically affected, but delivery may be difficult if the

woman's hip joints or lumbar spine is affected. If a woman develops an RA flare during pregnancy, first-line treatment usually

begins with prednisone

other immunosuppressive agents may be required

****.Myasthenia gravis: Myasthenia gravis varies in its course during pregnancy. Frequent acute myasthenic episodes may

require increasing doses of anticholinesterase drugs (eg, neostigmine ), which may cause symptoms of cholinergic excess (eg,

abdominal pain, diarrhea, vomiting, increasing weakness); atropine may then be required. Sometimes myasthenia becomes

refractory to standard therapy and requires corticosteroids or immunosuppressants. During labor, women may need assisted

ventilation and are extremely sensitive to drugs that depress respiration (eg, sedatives, opioids, Mg sulfate). Because the IgG

responsible for myasthenia crosses the placenta, transient myasthenia occurs in 20% of neonates, even more if mothers have

not had a thymectomy

Immune thrombocytopenic purpura (ITP): ITP , mediated by maternal antiplatelet IgG, tends to worsen during pregnancy and

increases risk of maternal morbidity. Corticosteroids reduce IgG levels and cause remission in most women, but improvement is

sustained in only 50%. Immunosuppressive therapy and plasmapheresis further reduce IgG, increasing platelet counts. Rarely,

splenectomy is required for refractory cases; it is best done during the 2nd trimester, when it causes sustained remission in

about 80%

Q152. the following are genetic risk factors for rheumatic fever, except ;;

1HLA CLASS 1 ALLELES;;

2HIGH LEVEL EXPRESSION OF D8/17''

3HIGH LEVELS OF MANNOSE BINDING LECTIN;;

4POLYMORPHISM OF THE TGF BETA ONE GENE

Genetic host risk factors include the D8/17 B-cell antigen and certain class II histocompatibility antigens

N-Acetylglucosamine (GlcNAc) is the major immunoepitope of group A streptococcal cell wall carbohydrates. Antistreptococcal

antibodies cross-reactive with anti-GlcNAc and laminin are present in sera of patients with rheumatic fever. The cross-reactivity

of these antibodies with human heart valvular endothelium and the underlying basement membrane has been suggested to be

a possible cause of immune-mediated valve lesion. Mannose-binding lectin (MBL) encoded by the MBL2 gene, a soluble

Page 64: DocumentQ1

pathogen recognition receptor, has high affinity for GlcNAc. We postulated that mutations in exon 1 of the MBL2 gene

associated with a deficient serum level of MBL may contribute to chronic severe aortic regurgitation (AR) of rheumatic etiolo

Molecular mimicry between streptococcal and human proteins is considered as the triggering factor leading to autoimmunity in

rheumatic fever (RF) and rheumatic heart disease (RHD). Here, we present a review of the genetic susceptibility markers

involved in the development of RF/RHD and the major immunopathological events underlying the pathogenesis of RF and RHD.

Several human leucocyte antigen (HLA) class II alleles are associated with the disease. Among these alleles, HLA-DR7 is

predominantly observed in different ethnicities and is associated with the development of valvular lesions in RHD patients.

Cardiac myosin is one of the major autoantigens involved in rheumatic heart lesions and several peptides from the LMM (light

meromyosin) region were recognized by peripheral and intralesional T-cell clones from RF and RHD patients. The production of

TNF-α and IFN-γ from heart-infiltrating mononuclear cells suggests that Th-1 type cytokines are the mediators of RHD heart

lesions while the presence of few interleukin-4 producing cells in the valve tissue contributes to the maintenance and

progression of the valvular lesions.

POLYMORPHISM OF THE TGF BETA ONE GENE...is associated wth rheumatoid arthritis

Association of transforming growth factor-ß1 gene polymorphism in the development of Epstein-Barr virus-related hematologic

diseases

Back to

top

vinay. Guest

Posted: Fri Jul 16, 2010 12:13 pm Post subject:

Q153. the most common organism implicated in interleukin -1 receptor associated kinase defect is ;;

1n.meningitidis;;

2n.gonorrhoea;;

3s;pneumoniae;;

4proteus vulgaris

susceptibility to pyogenic bacteria such as staphylococci, streptococci, clostridia; resistant to mycobacteria; autosomal

recessive

Q154. the following drug has maximum propensity to cause -peripheral neuropathy;;

1.DIDANOSINE;;

2ZIDOVUDENE;;

3STAVUDINE;;

4LAMIVUDINE

STAVUDINE;;

The main severe adverse effect is peripheral neuropathy, which can be corrected by reducing dosage. Stavudine has been

shown in laboratory test to be genotoxic, but with clinical doses its carcinogenic effects are non-existent. It is also one of the

most likely antiviral drugs to cause lipodystrophy, and for this reason it is no longer considered an appropriate treatment f

most patients in developed countries.

It is still used as first choice in first line therapy in resource poor settings such as in India. Only in case of development of

peripheral neuropathy or pregnancy is it changed to the next choice - Zidovudine.

DIDANOSINE;

The most common adverse events with didanosine are diarrhea, nausea, vomiting, abdominal pain, fever, headache and rash.

Peripheral neuropathy occurred in 21-26% of participants in key didanosine trials.

Pancreatitis is rarely observed but has caused occasional fatalities, and has black box warning status. Other reported serious

adverse events are retinal changes, optic neuritis and alterations of liver functions. The risk of some of these serious adve

events is increased by drinking alcohol.

In February of 2010, the United States Food and Drug Administration issued a statement that patients using Didanosine (Videx)

Page 65: DocumentQ1

are at risk for a rare but potentially fatal liver disorder, non-cirrhotic portal hypertension

Didanosine (ddI; 2',3'-dideoxyinosine) was the second drug licensed for the treatment of HIV infection, followed shortly

thereafter by zalcitabine. Didanosine is metabolized to dideoxyadenosine in vivo. It is best absorbed on an empty stomach at

high pH. The toxicity profile of didanosine is quite different from that of zidovudine. The most common toxicity is a painful

sensory peripheral neuropathy that occurs in ~30% of patients receiving >400 mg/d. 'It generally resolves with discontinuatio

of the drug and may not recur if the drug is resumed at a reduced dose. At higher doses than are currently used one may see

pancreatitis in ~10% of patients. Pancreatitis associated with didanosine therapy can be fatal. Didanosine should be

discontinued if a patient experiences abdominal pain consistent with pancreatitis or if an elevated serum amylase or lipa

is found in association with an edematous pancreas on ultrasound.

Zalcitabine (ddC; 2',3'-dideoxycytidine) is rarely used today in the management of patients with HIV infectionand was

discontinued from the US market in 2006. Among the nucleoside analogues licensed for the treatment of HIV infection, it is

probably the weakest. The main toxicities of ddC are peripheral neuropathy and pancreatitis.

no % given

Stavudine (d4T; 2',3'-didehydro-3'-deoxythymidine) was the fourth drug licensed for the treatment of HIV infection and was

discontinued from the US market in 2006. Like zidovudine, stavudine is a thymidine analogue. These two drugs are antagonistic

in vitro and in vivo and should not be given together. Stavudine has been associated with a higher incidence of mitochondrial

toxicity than the other licensed nucleoside analogues. Peripheral neuropathy, lipoatrophy, lactic acidosis, and hepatic steat

are the main toxicities of stavudine.

DIDANOSINE...

The most serious toxicities associated with didanosine include peripheral neuropathy and pancreatitis, both of which are thought

to be a consequence of mitochondrial toxicity. Up to 20% of patients reported peripheral neuropathy in early clinical trials

(Perry and Noble, 1999). As with other dideoxynucleosides, peripheral neuropathy is more common with higher doses or

concentrations of didanosine and is more prevalent in patients with underlying HIV-related neuropathy or in those receiving

other neurotoxic drugs. Typically, this is a symmetrical distal sensory neuropathy that begins in the feet and lower extremities

but may involve the hands as it progresses (stocking/glove distribution). Patients complain of pain, numbness, and tingling i

the affected extremities. If the drug is stopped as soon as symptoms appear, the neuropathy will stabilize and should improve

or resolve. Retinal changes and optic neuritis also have been reported with didanosine, and patients should undergo periodic

retinal examinations.

****STAVUDINE

The most common serious toxicity of stavudine is peripheral neuropathy. Neuropathy occurred in up to 71% of patients in initial

monotherapy trials with a dose of 4 mg/kg per day. With the current recommended dose of 40 mg twice daily, the neuropathy

incidence is about 12% (Hurst and Noble, 1999). Although this is thought to reflect mitochondrial toxicity, stavudine is a less

potent inhibitor of DNA polymerase-g than either didanosine or zalcitabine, suggesting that other mechanisms may be involved

(Cui et al., 1997). As with other dideoxynucleosides, peripheral neuropathy is more common with higher doses or

concentrations of stavudine and is more prevalent in patients with underlying HIV-related neuropathy or in those receiving

other neurotoxic drugs. Stavudine is also associated with a progressive motor neuropathy characterized by weakness and in

some cases respiratory failure, similar to Guillain-Barre syndrome (HIV Neuromuscular Syndrome Study Group, 2004).

ZALCITABINE

Zalcitabine toxicities are similar to those of the other dideoxynucleoside analogs didanosine and stavudine.

Severe peripheral neuropathy has been reported in up to 15% of patients. Peripheral neuropathy is dose-related and more

common with preexisting HIV-associated neuropathy and advanced HIV disease. This is a symmetrical distal sensory

neuropathy that begins in the feet but may progress to a stocking/glove distribution. Other specific risk factors for neuropa

include alcohol consumption, diabetes, and low vitamin B12 concentrations (Fichtenbaum et al., 1995). If the drug is stopped

as soon as symptoms appear, the neuropathy usually stabilizes and should improve or resolve. Pancreatitis occurs rarely with

zalcitabine therapy and appears to be less frequent than with didanosine (Saravolatz et al., 1996).

Page 66: DocumentQ1

One toxicity unique to zalcitabine is oral ulceration and stomatitis, suggesting that this drug may have toxicity in rapidly

dividing mucosal cells. Ulcerations of the buccal mucosa, soft palate, tongue, or pharynx occur in up to 4% of patients (Adki

et al., 1997) but may resolve with continued therapy. An erythematous maculopapular rash is reported commonly during the

first 14 days of therapy but generally is self-limited and mild. Other reported toxicities include cardiomyopathy, arthralgias,

myalgias, and elevated hepatic transaminases.

CONCLUSION-------

DIDANOSINE(20%)>ZALCITABINE(15%)>STAVUDINE(12%)

STAVUDINE IN VERY HIGH DOSE CAN PER NEUROPATHY UPTO 71% MAY B TATS Y SOME INSTITUTE HAS GIVEN THATS AS

ANSWER BUT I FEEL IN AIIMS THIS WILL B THE SEQUENCE

Q155. non-invasive diarrhoea is caused by all, except ;;

1shigella;;

2typhoid;;

3bacillus cereus;;

4y.enterocolitica;

ANSWER SHIGELLA

ITS JUST A STRAIGHT FOWARD QUESTION AS IT CAUSE BLOODY DIARRHOEA,,OUGHT TO B INVASIVE.....

Q156. PELLINGRI-STEIDA LESION IN KNEE JOINT SIGNIFIES;;

1patellofemoral pathology;;

2acute acl tear;;

3chronic acl tear;;

4chronic mcl injury

A Pellegrini-Stieda lesion refers to a post traumatic ossification in or near the medial collateral ligament near the margin of the

medial femoral condyle. One presumed mechanism of injury is a Stieda fracture (avulsion injury of the medial collateral

ligament at the medial femoral condyle).

Etymology

Q157. thimble pitting of nails is seen in ;;

1alopecia areata;;

2lichen planus;;

3pityriasis lubra pilaris;;

4pityryais rosea;;

Q158. all of the following are branches of the basilar artery, except ;;

1labyrhinthine;;

2ant inf cerebellar;;

3post nf cerebellar;;

4sup cerebellar;;

Back to

top

vinay. Guest

Posted: Sat Jul 17, 2010 12:43 pm Post subject:

Q159. haemodialysis amyloid is deposited in ;;

1knee joint;;

2large intestine;;

Page 67: DocumentQ1

3liver;;

4 tongue;

Nuffield Department of Pathology, University of Oxford John Radcliffe Hospital, Oxford *Nuffield Department of Orthopaedi

Surgery, Nuffield Orthopaedic Centre Oxford Renal Unit, Churchill Hospital Oxford

Accepted for publication 24 July 1990.

Deposition of ß2-microglobulin amyloid in the joints of dialysis patients is common and begins early in the course of treatment,

but its pathogenic significance in the production of dialysis arthropathy is uncertain.

The joints (hip, knee, shoulder, elbow, wrist, cervical and lumbar spine, sacroiliac joint) and systemic tissues of 19 autops

patients who had undergone haemodialysis for between 6 and 231 months were examined histopathologically for the presence

of ß2-microglobulin amyloid;

it was present in all joints examined, including those unassociated with radiological changes and those of patients who had b

on haemodialysis alone for only 24 months. Osteoarticular ß2-microglobulin amyloid deposits were also found in patients who

had been treated mainly by continuous ambulatory peritoneal dialysis. Systemic amyloid deposition was only seen in patients

who had been haemodialysed for more than 13 years and consisted of sparse tiny deposits in blood vessel walls.

Q160. charge syndrome is associated with ;;

1cystic fibrosis;;

2oesophageal atresia;;

3choanall atresia;;

4cleft palate;;

Although genetic testing positively identifies nearly 2/3 of the total number of children with CHARGE, diagnosis is still largely

clinical. The acronym CHARGE was coined in 1981 to describe a cluster of features identified in a number of children. The

following are the signs that were originally identified in children with this syndrome, but these features alone are no longer used

in official diagnosis.

* C - Coloboma of the eye, central nervous system anomalies

* H - Heart defects

* A - Atresia of the choanae

* R - Retardation of growth and/or development

* G - Genital and/or urinary defects (Hypogonadism)

* E - Ear anomalies and/or deafness

Q161. the excimer laser operates in which of the following spectral wavelenghths;;

1infrared;;

2microwave;;

3near ultraviolet;;

4far ultraviolet;;

An excimer laser (sometimes, and more correctly, called an exciplex laser) is a form of ultraviolet laser which is commonly u

in eye surgery and semiconductor manufacturing. The term excimer is short for 'excited dimer', while exciplex is short for

'excited complex'. An excimer laser typically uses a combination of an inert gas (argon, krypton, or xenon) and a reactive ga

(fluorine or chlorine). Under the appropriate conditions of electrical stimulation, a pseudo-molecule called an excimer (or

of noble gas halides, exciplex) is created, which can only exist in an energized state and can give rise to laser light in th

ultraviolet range.

The wavelength of an excimer laser depends on the molecules used, and is usually in the ultraviolet.

The high-power ultraviolet output of excimer lasers makes them useful for surgery (particularly eye surgery), for lithography for

semiconductor manufacturing, and for dermatological treatment. Excimer laser light is typically absorbed within the first

Page 68: DocumentQ1

billionth of a meter (nanometer) of tissue.

Q162. welcome dr palm;;pl put the q no, while you anser;;

polyprenoid, which canot be synthesises by human body is ;;

1dolichol;;

2ubiquinone;;

3retinoids;;

4menaquinone;

Chain-like structures are also synthesized from isoprenoid units and include ubiquinone, dolichol and the fat-soluble vitamins.

Ubiquinone is an electron carrier in the mitochondrial respiratory chain while dolichol is a long-chain alcohol which helps in

glycoprotein synthesis. V itamins A, D, E and K are absorbed in the intestine and transported in the blood in the lipoprotein

complexes. Vitamin A activity is present in retinoids and carotenoids. They function as visual pigment and as regulators of

tissue growth and development. Vit E is represented by a group of closely related, hydrophobic lipids called the tocopherols.

They are powerful antioxidants which prevent oxidative damage to membrane lipids. Vit K is a blood-clotting co-factor.

Plant-derived isoprenoid compounds include

rubber, camphor, the fat-soluble vitamins A, D,

E, and K, and β-carotene (provitamin A) pg 118 harper old

Originary from vitamin A or synthesized by human body, retinoids can modulate cell proliferation, differentiation, and apopto

decreasing carcinogenesis, effects controlled by six different human nuclear retinoid receptors

answer menoquinone

Q163. which of the following is not part of a plan;;

1schedules;;

2policies;;

3programmes;;

4manpower;

Q164. time take for any project is estimated by ;;

1work sampling;;

2input-output analysis;;

3network analysis;;

4 system analysis

Back to

top

vinay. Guest

Posted: Mon Jul 19, 2010 12:46 pm Post subject:

Q165. xemphysema aquosum s seen in ;;

1wet drowning;;

2dry drowning;;

3immersion syndrome;;

4sec drowning;;

Q166. THURSTON-HOLLAND SIGN IS SEEN IN ;;

1OBLIQUE FRACTURE OF LOWER 1/3 OF HUMERUS;;

2REVERSE OBLIQUE FRACTURE OF INTERTROCHANTERIC FEMUR;;

3SALTER -HARRIS TYPE 2 FRACTURE;;

4CORONAL FRACTURE F FEMORAL CONDYLES;;

Q167. what is the mechanism of action of levosimendon;;

1it is a potassium channel pener;;

Page 69: DocumentQ1

2it inhibits sodium entry into the cell;;

3it is a calcium sensitiser;;

4it inhibits na-k atp pump;;

answer 3

Levosimendan is a calcium sensitiser – it increases the sensitivity of the heart to calcium, thus increasing cardiac contra

without a rise in intracellular calcium. Levosimendan exerts its positive inotropic effect by increasing calcium sensitivity

myocytes by binding to cardiac troponin C in a calcium-dependent manner. It also has a vasodilatory effect, by opening

adenosine triphosphate (ATP)-sensitive potassium channels in vascular smooth muscle to cause smooth muscle relaxation. The

combined inotropic and vasodilatory actions result in an increased force of contraction, decreased preload and decreased

afterload. Moreover, by opening also the mitochondrial (ATP)-sensitive potassium channels in cardiomyocytes, the drug exerts a

cardioprotective effect.

Q168. maculae cerulae are caused by ;;

1pubic lice;;

2head lice;;

3body lice;;

4mites;;

pubic lice

small gray-blue macules found on the chest, abdomen, pages , or upper arms in pediculosis pubis, which are especially

noticeable in light-skinned individuals.

Q169. the type of filter, which has silver catalyst is ;;

1chamberland filter;;

2candle filter;;

3berkefield;;

4katadyn filter;;

answer 4

In the Katadyn filter, the surface of the filter is coated with a silver catalyst so that bacteria coming in contact with the

are killed by the 'oligodynamic' action of the silver ions, which are liberated into the water.

silver is unique in its behavior with oxygen. For example,when pure silver is melted in ambient air it absorbs about ten time

volume, or 0.3% of its weight of oxygen. On cooling to a few degrees above solidification, it abruptly releases most of its

oxygen in a dramatic phenomenon which the industry calls a “spit."

Q170. which of the following color code is not used for classifying the patient under the imnci strategy;;

1pink;;

2 red;;

3green;;

4yellow;;

classifications are colour coded: “pink” suggests hospital referral or admission,

“yellow” indicates initiation of treatment, and “green” calls for home treatment.

imnci

During the mid-1990s, the World Health Organization (WHO), in collaboration with UNICEF and many other agencies,

institutions and individuals, responded to this challenge by developing a strategy known as the Integrated Management of

Childhood Illness (IMCI). Although the major reason for developing the IMCI strategy stemmed from the needs of curative care

the strategy also addresses aspects of nutrition, immunization, and other important elements of disease prevention and health

Page 70: DocumentQ1

promotion. The objectives of the strategy are to reduce death and the frequency and severity of illness and disability, and t

contribute to improved growth and development. This strategy has been adapted for India as Integrated Management of

Neonatal and Childhood Illness (IMNCI).

The IMNCI clinical guidelines target children less than 5 years old — the age group that bears the highest burden of deaths

from common childhood diseases. The guidelines take an evidence-based, syndromic approach to case management that

supports the rational, effective and affordable use of drugs and diagnostic tools

An evidence-based syndromic approach can be used to determine the:

• Health problem(s) the child may have;

• Severity of the child’s condition;

• Actions that can be taken to care for the child (e.g. refer the child immediately, manage with available resources, or mana

at home).

In addition, IMNCI promotes:

• Adjustment of interventions to the capacity and functions of the health system; and

• Active involvement of family members and the community in the health care process.

Q171. association is best implicated by ;;

1case control study;;

2prospective study;;

3cross sectional study;;

4experimental epidemiology;;

Q172. partial lipodystrophy is seen with ;;

1MGN;;

2MPGN;;

3FSGS;;

4MCD;;

Q173. ALL OF THE FOLLOWING DRUGS ARE RECOMMENDED IN LIFE threatening fungal infections in neutropaenic pts ,

receiving chemotherapy, except ;;

1voricanazole;;

2caspofungin;;

3amphotericin;;

4itraconazole;;

ANSWER ITRACONAZOLE

FOR CASPOFUNGINS In mainly HIV-positive patients with oesophageal and oropharyngeal candidiasis, the new antifungal agent

caspofungin in a dosage of 35 or 50 mg/day showed superior efficacy compared with low-dose D-AmB (0.5 mg/kg/day).11

Remarkably, this compound, representing the first clinically studied agent from a new class of antifungals inhibiting the

synthesis of ß-(1,3)-d-glucan in the fungal cell wall, was extremely well tolerated. A dose-limiting toxicity has not yet been

defined. Beyond that, resistance to this and other echinocandin antifungals has so far been experimentally induced in Candida

species only at extremely low rates in vitro.12 Caspofungin has also shown significant efficacy as salvage antifungal in 63

patients with documented invasive aspergillosis refractory or intolerant to conventional antifungals.13 Based upon the clearl

documented response to caspofungin in 41% of these patients, the drug was approved in early 2001 by the Food and Drug

Administration (FDA) for the treatment of patients with aspergillosis refractory or intolerant to other licensed antifungals

Itraconazole, the only triazole presently licensed for the treatment of aspergillosis, has recently become available for

intravenous administration. In neutropenic patients with haematological malignancies and refractory fever, intravenous

itraconazole has shown at least equivalent efficacy (47% versus 38%) to D-AmB. Not surprisingly, it was significantly better

tolerated (withdrawal due to toxicity 19% versus 38%) and showed less nephrotoxicity than D-AmB. Beyond that, about one

third of patients given intravenous itraconazole could be switched to the oral formulation after a median of 9 days of treatment.

Page 71: DocumentQ1

As with voriconazole, an important potential for interactions with a large number of other drugs must be considered for

itraconazole.

Q174. which one is a robovirus;;

1buniya virus;;

2phlebovirus;;

3nairovirus;;

4hantavirus;;

Q175. WHICH OF THE FOLLOWING ARE NOT CONTINUOUS CELL LINES;;

1hela;;

2hep-2''

3mc coy;;

4wl-38

Back to top

vinay. Guest

Posted: Tue Jul 20, 2010 12:48 pm Post subject:

Q176. this disease is not associated with endogenous intracellular accumulations;;

1steatosis;;

2ochronosis;;

3anthracosis;;

4melanoma;;

ans anthracosis?

it is an exogenous pigment deposition disease...

Black lung disease, also known as Coal workers' pneumoconiosis (CWP), is caused by long exposure to coal dust. It is a

common affliction of coal miners and others who work with coal, similar to both silicosis from inhaling silica dust, and to t

long-term effects of tobacco smoking. Inhaled coal dust progressively builds up in the lungs and is unable to be removed by the

body; that leads to inflammation, fibrosis, and in the worst case, necrosis.

Coal workers' pneumoconiosis, in its most severe state, develops after the initial, milder form of the disease known as

anthracosis (anthrac - coal, carbon). This is often asymptomatic and is found to at least some extent in all urban dwellers[1]

due to air pollution. Prolonged exposure to large amounts of coal dust can result in more serious forms of the disease, simpl

coal workers' pneumoconiosis and complicated coal workers' pneumoconiosis (or Progressive massive fibrosis,)

All the other options are causes of endogenous pigment depositions...

steatosis-lipid deposit

ochranosis-HOMOGENTISIC ACID OXI DEFI in children with homogentisic acid deposition in joints cartilages

melanosis-inc melanin deposi in skin eyes etc...

Q178. in leucocyte adhesion def type 1 -the defect lies in the synthesis of ;;

1alpha interferon;;

2beta 2 microglobulin;;

3beta2 integrins;;

4 leukotrienes;;

ans B2 integrins...

Leukocyte adhesion deficiency type I (LAD I) is a failure to express CD18, which composes the common ß2 subunit of LFA1

family (ß2 integrins). CD11a/CD18 (LFA-1) expressed on lymphocytes is known to play an important role in lymphocyte

trafficking (adhesion to vascular endothelium), as well as interactions to antigen presenting cells (APC). LFA-1 also plays a role

of cytotoxic killing by T cells. Another member of this family is CD11bCD18 (MacA or CR3) and CD11cCD18(CR4). These 2

members mediate leukocyte adhesions to endothelial cells but they also serve as receptors for iC3b (inactivated C3b). These

patients succumb to life-threatening infection, usually within 2 years of life in severe cases of leukocyte adhesion deficiency I

(<1% expression of CD18). In milder forms of leukocyte adhesion deficiency I (1-30% expression of CD8), patients may

survive to adulthood.

Page 72: DocumentQ1

Leukocyte adhesion deficiency type II is extremely rare; only a handful cases have been reported and most of them are of

Middle Eastern decent. It is a defect in the expression of ligands for selectins due to lack of enzymes required for expression of

selectin ligands. Patients have leukocytosis, recurrent infections (more prominent in infants and toddlers), and severe growt

and mental retardation. This disease is a defect in fucose metabolism (lack of fucosylation of the carbohydrate selectin ligands)

that results in failure to express the ligand for E and P selectin, sialyl Lewis-X (CD15s) expressed on leukocytes and endothelial

cells. The patients are unable to fucosylate other glycoproteins, including the H blood group polysaccharide.

Patients with leukocyte adhesion deficiency II manifest the Bombay phenotype (ie, negative for O and H blood group antigens

with potential production of anti-H antibody). The immunoglobulin M (IgM) and immunoglobulin G (IgG) heavy chains are also

not fucosylated. However, IgM and IgG serum levels are within the reference range in patients with leukocyte adhesion

deficiency II.

Q179. PRINGLE MANOVEUR REFERS TO ;;

1clamo over ivc;;

2clamp over hepatic vein;;

3clamp across of foramen of winslow;;

4clamp across splenic artery

ans-clamp across of foramen of winslow ref. BAILEY 25TH,SCHWARTZ'S 9TH

The Pringle manoeuvre is a surgical manoeuvre used in for controlling traumatic liver injury n elective hepatic resection to

minimize blood loss during surgery.Pringle maneuver (occlu- sion of hepatic artery, portal vein, and bile duct)ie hepatic pedicle

is accomplished by tourniquet or vascular clamp through the foramen of Winslow.

Q180. which one of the following activities is simultaneously stimulated by epinephrine in muscle and and inhibited by

epinephrine in the liver;;

1fatty acid oxidation;;

2glycogenolysis;;

3cyclic amp synthesis;;

4glycolysis;;

Q181. all the following are risk factors for thromboembolic events in the perioperative period, except ;;

1morbid obesity;;

2adenocarcinoma;;

3nephrotic syndrome;;

4 chronic renal failure;

Back to

top

vinay. Guest

Posted: Wed Jul 21, 2010 12:46 pm Post subject:

Q183. anencephaly is the result of a defect in which of the following;;

1closure of the caudal neuropore;;

2closure of the rostral neuropore;;

3formation of the first bbranchial arch;;

4formation of the somites;;

Q184. CASTELLANIS PAINT contains all of the following except ;;

1basic fuchsin;;

2phenol;;

3benzene hexachloride;;

4boric acid;

ans-benzene hexachloride ref.internet

Page 73: DocumentQ1

CASTELLANI'S PAINT

Basic fuchsin 0.3

Ethyl alcohol 95% 10.0

Boric acid 1.0

Phenol liquef. 4.0

Acetone 5.0

Resorcinol 10.0

Water to 100.0

Castellani's paint was perfected in 1905 by Aldo Castellani (1878-1971), an Italian physician and a specialist in tropical

diseases. Castellani's paint is an excellent preparation for tinea cruris and moniliasis of intertriginous areas.It is effective in

pustular dermatoses of the hands and feet, and has also been recommended for pruritus ani and pruritus vulvae. Colorless

Castellani's paint may be used to reduce secondary bacterial contamination in onycholysis and in chronic paronychia.

Q185. LAURINS/ MERCHANTS VIEW is the x-ray view of ;;

1talus;;

2spine in a patient with scoliosis;;

3hip joint;;

4patella;;

answer 4 patella

Merchant Technique

mainly for subluxation

- Discussion:

- for evaluation of both the patella and trochlea surfaces of femur;

- disadvantages:

- this view does not necessarily predict which patients will show functional improvements

w/ surgical realignment;

Laurin technique:

- lateral patellofemoral angle is index of tilt but not of subluxation;

- patellar alignment assessed using lateral patellofemoral angle on axial views, made w/ knee in 20 deg of flexion;

- angle formed by lateral patellar facet & line drawn across most prominent aspects of anterior

portion of femoral trochlea should be open laterally in normal patellofemoral joint;

other views

Stress Axilla View:

- in some cases, there will be impressive differences between static and dynamic axilla knee views;

- made with the knee flexed 35 deg off the end of the x-ray table;

- a constant lateral pressure is exerted on to the patella in an attempt to displace the patella laterally;

- comparisons should be made between the symptomatic and asymptomatic knees;

Sunrise View:

- used to image a tangential view of the patella;

Page 74: DocumentQ1

- the patient is prone with the knee flexed 115 deg;

- central beam is directed toward the patella with 15 deg cephalic tilt;

Q186. which structure most effectively prevents toxic molecules from penetrating an epithelium by passing between ad

epithelial cells;;

1desmosome;;

2gap iunction;;

3 hemidesmosome;;

4tight junction;;

ans tight junction..?

An important property of epithelial and endothelial cells is their assembly into a physical and ion- and size-selective barrier

separating tissues. Intercellular junctions, such as adherens and tight junctions, play a crucial role in the formation and

maintenance of epithelial and endothelial barriers. Adherens and tight junctions were first identified on the ultrastructural

as part of the terminal bar, a tripartite junctional complex bordering the apico-basolateral membrane in a variety of polarized

simple epithelia and implicated in barrier function . Desmosomes form the third structure of this complex.. Tight junctions a

the most apical structure of the apical complex demarcating the border between apical and basolateral membrane domains. The

intercellular membrane space of tight junctions is almost completely obliterated, hence their alternative name zonulae

occludens. Adherens junctions are positioned immediately below tight junctions and characterized by two apposing membranes,

which are separated by 20 nm, that run parallel over a distance of 0.2–0.5 m. Both adherens and tight junctions are closely

associated with a circumferential belt of actin.

Tight junctions do provide epithelia with a semipermeable size- and ion-specific barrier, which varies depending on their exact

molecular composition (reviewed in Anderson et al., 2004). They also restrict the basolateral n memb components, so called

"fence functions"....

HEMIdeSMOSOMES connect cell to ECM..

GAP junc - free mobility of moleculecus/ions cross cell- functional syncitium..

DESMOSOMES ...??they too are intrecellular transmemb connections but prvide structural support mainly by attaching

themselves to intermediate filaments in cytoplasm..AB against them l/t bullous skin disorders.....?

Q187. the systematic distorsion of retrospective studies that can be eliminated by a prospective design is ;;

1confounding;;

2effect modification;;

3recall bias;;

4measurement bias;;

ans recall bias

or memory bias..

since in cohort studies we're proceeding from cause to effect there's no need for cohorts to recall events of past...

Q188. LEVI-LORAIN DWARFISM IS DUE TO A LACK OF WHICH HORMONE;;

1SOMATOSTATIN;;

2SOMATOTROPIN;;

3 SOMATOMEDINC;;

4TRANSFORMING GROWTH FACTOR -ALPHA;;

answer somatomedin c

Dwarfism can also result from a lack of somatomedin C (also called insulin like growth factor, IGF-1) production. Somatomedin

C is a hormone produced in the liver that increases bone growth when growth hormone is present. The African pygmy and the

Levi-Lorain dwarfs lack the ability to produce somatomedin C in response to growth hormone. All causes of dwarfism lead to a

proportionate little person.

Q189. THE VISUAL FIELD DEFECT SEEN IN VIT B 12 DEFICIENCY;;

1arcuate;;

Page 75: DocumentQ1

2 paracentral scotoma;;

3nasal step;;

4centrocecal scotoma

Back to

top

vinay. Guest

Posted: Thu Jul 22, 2010 1:21 pm Post subject:

Q190. all the following uraemic manifestations improve with dialysis, except ;;

1metabolic acidosis;;

2osteodystrophy;;

3asterixis;;

4bleeding;

ANS-OSTEODYSTROPHY REF.HARRISON 17,CMDT 10

READ TABLE 274-3

METABOLIC ACIDOSIS N ASTERIXIS IMPROVES WITH DIALYSIS

BLEEDING TIME IMPROVES WITH DIALYSIS ALTHOUGH IT DOESNT BECOME NORMAL-REQUIRES CRYO PPT TRANSFUSION

RENAL OSTEODYSTROPHY CAN IMPROVE OR WORSEN WITH DIALYSIS..OFTEN REQUIRES OTHER MEASURES LIKE VIT D

SUPPL. ,PARATHYROIDECTOMY OR RENAL TRANSPLANT.

Q191. which culture method is used for the preparation of bacterial antigens and vaccines;;

1lawn culture;;

2stroke culture;;

3pour plate culture;;

4liquid cultyre;;

Q192. which area of the mandible is most commonly fractured;;

1 angle;;

2condyle;;

3molar ;;

4 mental;

Q193. during lumbar synpathectomy-open - all can be confused for the lumbar sypathetic chain, except ;;

1genitofemoral nerve;;

2iliohypogastric nerve;;

3lymph nodes and lymphatics;;

4tendinous strip of psoas minor;;

ANS-ILIOHYPOGASTRIC NERVE REF-DIRECT PICK FROM BAILEY 24 PG 953 NOT GIVEN IN 25TH EDITION AS SURGERY HAS

BECOME OBSELETE.

''care should b taken not to mistake genitofemoral nerve,small lymph nodes and lymphatics or occasional tendinous strip of

psoas minorfor sympathetic chain"

Lumbar sympathectomy (LS) has been used in the treatment of various vascular and neurological disorders of lower extremities

for close to 80 years. LS is acknowledged to have a role in the treatment of patients with reflex symptomatic dystrophy

(causalgia), vasospastic disorders like acrocyanosis and Raynaud's syndrome, hyperhydrosis of the feet, symptomatic

vasospasm, and nonreconstructable arterial occlusive disease. A literature survey finds its miscellaneous uses for frostbites

desiccation of chronically moist ulcerations between the toes, chronic renal pain, rectal tenesmus, and sympathetically

maintained intractable pain due to malignant reasons. Although it is the most commonly performed operation in the developing

Page 76: DocumentQ1

countries for peripheral ischemic diseases (PID), there is considerable controversy about its usefulness

Q194. perforin molecules are found in ;;

1primary granules of neutrophils;;

2sec granules of neutrophils;;

3cytotoxic t cells;;

4macrophages;;

ANS-3)CYTOTOXIC T CELLS REF-ROBBINS 8TH PG 31 -GIVEN IN MECH OF APOPTOSIS VERY IMP TOPIC

PERFORINS -A TRANSMEMBRANE PORE FORMING MOLECULE WHICH PROMOTES ENTRY OF GRANZYMES

Q195. MEYERS -KOUVENAAR bodies are found in ;;

1filariasis;;

2falciparum malaria;;

3schistosomiasis;;

4l epromatous leprosy;;

ANS-FILARIASIS REF ROBBINS 8TH PG 395

dead microfilariae surrounded by stellate, hyaline, eosinophilic precipitates embedded in small epithelioid granulomas (Meyer

Kouvenaar bodies)

Back to top

Guest

Posted: Fri Jul 23, 2010 12:50 pm Post subject:

A study was made of the Meyers-Kouwenaar (MK) body in the livers of experimentally infected ferrets. Meyers-Kouwenaar

bodies, the carcasses of microfilariae (mff) covered by deposits of Splendore-Hoeppli (SH) material, were found in small

abscesses of eosinophils and in granulomas. The SH deposits varied from an eosinophilic, hyaline fringe around intact mff to

multilayered deposits surrounding an unrecognizable granular remnant. In abscesses, peroxidase activity was intense in SH

deposits and the surrounding eosinophils. The presence and localization of IgG were variable in MK bodies, as detected by an

enzyme-linked immunohistologic assay; and antigens of mff were not detected in the SH deposits. Electron microscopy of the

MK body demonstrated a layered, radial deposition of amorphous and granular material on the mff and a structural

heterogeneity which apparently included leukocyte granules and other cell organelles. Leukocytes surrounding MK bodies in

abscesses were often degranulated and degenerate; incorporation of lysosomes of eosinophils and cellular debris into the SH

deposits at the periphery of the MK bodies was indicated.

Back to

top

vinay. Guest

Posted: Fri Jul 23, 2010 12:53 pm Post subject:

Q196. all are actions of endothelin 1 except ;;

1bronchodilatation;;

2vasoconstriction;;

3devreased gfr;;

4vasodilatation;;

Endothelial cells also produce endothelin-1, one of the most potent vasoconstrictor agents yet isolated. Endothelin-1 (ET

endothelin-2 (ET-2), and endothelin-3 (ET-3) are the members of a family of three similar 21-amino-acid polypeptides

this ques is lil tricky in sense ,vasoconstriction is well known feature of endothelin 1..so we tend to think vasodil will no

but vasodil is due to increased NO..

Page 77: DocumentQ1

decreased gfr too seen it becoz contraction of mesangial cell n capillaries...

answer here is bronchodilation

as wth endothelin 1 broncho constriction which is commonly seen in asthma..

Q197. splenenunculi can be found in all of the following areas, except ;;

1splenic hilum;;

2post abdominal wall;;

3tail of pancreas;;

4left spermatic cord;;

Splenunculi are small nodules of spleen that are detached from the rest of the organ. They are common, seen in up to 16% of

CTs of the abdomen and up to 30% of autopsies. They are benign and asymptomatic, their importance mainly related to the

need to distinguish them from more sinister pathology.

Splenunculi are typically a few centimetres in diameter when identified, well circumscribed rounded or ovoid nodules. Althoug

most are located near the spleen, they have been identified elsewhere in the abdominal cavity including 2,3.

near the spleen (most common)

1.gastrosplenic ligament

2.splenorenal ligament

3.pancreatic tail

4.greater omentum

5.small bowel mesentery

6.stomach or bowel wall

7.scrotum

CT

They have density and enhancing characteristics similar to the rest of the spleen. Small (<1cm) may appear hypodense c.f. to

spleen, most likely as an artefact of partial voluming 2.

The converse of that is that if a mass is seen at the splenic hilum, which does not enhance in the same fashion as the adjacent

spleen, then alternative diagnoses should be entertained.

Scintigraphy

Tc 99m sulphur colloid scan

In rare instances the diagnosis can be confirmed with Tc99m sulphur colloid scan which will demontrate increased uptake as

long as the splenunculus is at [b]least 2cm/b] in diameter.

splenunculi adherent to anterior abdominal wall not posterior

apart from that its also seen in spermatic cord n scrotum

answer here choice 2 post abd wall

Q198. which one is the least commn type of high arch foot;;

1calaneocavus;;

2cavovarus;;

3plantaris;;

4calaeaneus;

CAVOVARUS

Neuromuscular imbalance due to weak intrinsics

Page 78: DocumentQ1

Conditions associated with cavus or cavo-varus feet :

Central nervous system

Brain

Cerebral palsy

Friedreich's Ataxia

Spinal Cord

Spinal cord tumour

Spinal dysraphism

Tethered cord

Spinal bifida

Diastematomyelia

Poliomyelitis

Spinal muscular atrophy

Peripheral Nervous system

Hereditary peripheral neuropathies, e.g. CMT

Traumatic peripheral nerve lesions

Muscle Disease

Duchenne's muscular dystrophy

Post-ischaemic (Volkmans contracture)

Common component of residual club feet

Idiopathic

Clinically

Often complains of fatigue and discomfort in the foot or ankle with the severity of symptoms paralleling the degree and rigidity

of the deformity

The presenting abnormality may be representing the early signs of a progressive neurologic degenerative disease

The idiopathic variety usually noticed at about 10 years of age and it is usually bilateral

Ankle usually has reduced dorsiflexion

Hindfoot : There is inversion of the heel (calcaneo varus) and inversion of the subtalar joint

Forefoot is plantar flexed (plantaris deformity) with pronation of the forefoot in relation to the hind foot

MTP joints are hyperextended

Interphalangeal joints are flexed

Weight bearing is via a tripod of the heel, the first and fifth metatarsal heads and associated with hind foot varus which

becomes fixed

Pressure is taken over a small area under the metatarsal heads, causing pain and callosities

Callosities also develop over interphalangeal joints due to pressure from shoe-wear

Coleman's block test

Establishes whether the hind foot is fixed or flexible and demonstrates the type of surgical correction required

Place the foot obliquely on a 1" - 1.5" block so that the first metatarsal hangs free to allow correction of the hind foot during

weight bearing

If the hind foot returns to neutral or valgus the deformity is flexible

Treatment

Is based on the age of the patient and the flexibility of the deformity

Tendon lengthenings and transfers are used for flexible feet

Bony procedures added for fixed deformities

Young children

Strengthening exercises and stretching

Ensure footwear has enough room for the toes

Operative

Page 79: DocumentQ1

Flexor to extensor transfers and interphalangeal joint arthrodesis will straighten the toes, if there is much cavus

Steindlers and short flexor / abductor hallucis release may also be needed to release plantar fascia, followed by a BK POP with

the foot held in valgus and supination for 6 weeks with frequent changes of POP to correct deformity (children aged 4 -

years)

Transfer of Tibialis Posterior to the calcaneum may result in correction of calcaneus deformity

Adolescents

If toe deformities become fixed, arthrodese all the PIP joints; so the long flexors no longer bunch the toes up and the long

extensors are reinserted into the metatarsal necks to elevate the forefoot

If hind foot varus is fixed in Coleman's test and the first ray deformity is severe, calcaneal osteotomy may be indicated

Dwyer type medial opening wedge or modified Dwyer with a lateral closing wedge osteotomy of the calcaneum

For children aged 10 or older

Can be combined with Steindler operation

Wedge excision of the tarso-metatarsal joints may be needed for severe deformity in the absence of neurological abnormality.

Done after maturity

Adults

Increased arch support and contact area in appropriate fitting shoes is the best treatment

Severe painful deformities may require operation

CALCANEOCAVUS

Due to major muscle imbalance, usually weak or absent gastrocnemius/soleus

Usually tibialis anterior, tibialis posterior and peronei are preserved

No conservative treatment

Progressive

Clinically

Complains of shoe fitting problems

Pain over prominences

Prominent heel

Upward pitch of calcaneus

High arch (pistol grip deformity)

Treatment

Before 5 years old

No treatment in genera,l as no bony deformity is usually present

Good muscle testing in this age group is difficult

Patient is not likely to be disabled by the problem

5 - 12 years old

Stabilise subtalar joint by extra-articular arthrodesis

Management of bony deformity of calcaneus and ankle

Release of plantar soft tissue contractures with or without calcaneal posterior displacement osteotomy

Tendon transfers to improve plantar flexion of os calcis

Page 80: DocumentQ1

Older than 12 years old

These patients have fixed deformity

Triple arthrodese to stabilise and correct hindfoot +/- tendon transfer to improve plantar flexion

wheelers ortho says

- cavus deformity of the foot (elevated longitudinal arch) due to fixed plantar flexion of the forefoot;

- main type is the cavovarus and less frequent is the cavovarus;

- associated with spinal cerebellar degenerative dz;

Q199. the nerve fibres mostresistant to mechanical trauma ;;

1motor;;

2sensory;;

3sympathetic;;

4parasympathetic;;

Q200. lipedema is not associated with ;;

1woman predominance;;

2no pitting;;

3involvement of trunk;;

4stammers sign;;

Lipedema, which is also known as painful fat syndrome, is abnormal symmetrical swelling due to accumulations of fat and fluid

located in the tissues just under the skin of the hips and legs. It extends downward toward the ankles but does not affect the

feet. There are also fatty bulges on the outer surfaces of the pages . [1]

This condition is inherited, occurs almost exclusively in women, and most cases gradually develop during puberty.

Lipedema involves the extra deposit and expansion of fat cells in a distinct pattern on the body from just below the waist to just

above the ankles.

Unlike the fat associated with obesity, the fat associated with lipedema cannot be exercised away.

Also, this fat tissue does not respond or reduce with diet.

The Symptoms of Lipedema

Pain is present particularly along the shin. This is a major problem

.

In the early stages of lipedema the upper part of the body may be slender.

Despite having a slim upper body, fat accumulates from the tops of the hips to the ankles.

When weight is gained, it accumulates in the areas of the hips and legs.

When weight is lost, the fat decrease occurs in areas other than those affected by lipedema. Weight loss does not occur in th

area between the waist and ankles.

Fat that extends down the legs creates a ring of fatty tissue that overlaps the tops of the feet.

Swelling develops in the legs during the second half of the day; however, this swelling decreases during sleep.

In the early stages, nodules develop. Nodules are small fatty lymps within the tissues.

In the later stages, lobules develop. Lobules are rounded fat deposits that aer largr than nodules.

diff b/w lipedema n lymphedema

1.Lipedema is symmetrical because the excess fat involves both legs equally.

Lymphedema is not symmetrical and often occurs in only one leg.

2.Lipedema swelling has a characteristic ring at the base of the ankle where the swelling stops.

Lymphedema swelling often begins at the foot and there is not ring at the base of the ankle.

Page 81: DocumentQ1

3.Lipedema does not include swelling of the foot.

Lymphedema swelling affects the foot, ankle, and leg.

4.Lipedema patients often complain of pain when touched.

Lymphedema pain is rarely this severe.

5.Lipedema often causes bruising and subcutaneous bleeding.

Lymphedema symptoms do not include bruising and subcutaneous bleeding.

6.Lipedema does not have pitting edema as a symptom. See How Lymphedema is Diagnosed

Lymphedema does have pitting edea as a diagnostic indicator.

7.Lipedema does not have Stemmer’s sign as a symptom.

Lymphedema does have Stemmer’s sign as a diagnostic indicator.

answer choce d stemmer sign

Back to

top

vinay. Guest

Posted: Sat Jul 24, 2010 12:47 pm Post subject:

Q202. in a neuron, graded electrogenesis occurs at ;;

1dendritic zone;;

2initial segment;;

3axon;;

4nerve ending

Q203. middle cardiac vein opening into coronary sinus is related to ;;

11 ant interventricular groove;;

2inferior interventricular groove;;

3lower border of the heart;;

4post atrioventricular groove

ans-posterior(inferior) interventricular septum REF.GRAYS 40,BDC VIMP TOPIC BLOOD SUPPLY OF HEART

Q204. BLOODLESS FOLD OF TREVES IS RELATED TO ;;

1meckles diverticulum;;

2ischiorectal fossae;;

3dononvilliers fascia;;

4iliocaecal fold

ans-ileocaecal fold ref -grays 40,BDC

Cecal Fossæ (pericecal folds or fossæ).—There are three principal pouches or recesses in the neighborhood of the cecum: (a)

The superior ileocecal fossa is formed by a fold of peritoneum, arching over the branch of the ileocolic artery which supplies the

ileocolic junction. The fossa is a narrow chink situated between the mesentery of the small intestine, the ileum, and the sma

portion of the cecum behind. (b) The inferior ileocecal fossa is situated behind the angle of junction of the ileum and cecum. It

is formed by the ileocecal fold of peritoneum (bloodless fold of Treves), the upper border of which is fixed to the ileum, op

its mesenteric attachment, while the lower border, passing over the ileocecal junction, joins the mesenteriole of the vermiform

process, and sometimes the process itself. Between this fold and the mesenteriole of the vermiform process is the inferior

ileocecal fossa. It is bounded above by the posterior surface of the ileum and the mesentery; in front and below by the ileocecal

fold, and behind by the upper part of the mesenteriole of the vermiform process. (c) The cecal fossa is situated immediately

behind the cecum, which has to be raised to bring it into view. It varies much in size and extent. In some cases it is sufficiently

large to admit the index finger, and extends upward behind the ascending colon in the direction of the kidney; in others it i

merely a shallow depression. It is bounded on the right by the cecal fold, which is attached by one edge to the abdominal wall

Page 82: DocumentQ1

from the lower border of the kidney to the iliac

Q205. the most widespread rickettsial infection is ;;

1epidemic typhus;;

2indian tick typhus;;

3murine typhus;;

4scrub typhus;;

ans-scrub typhus ref-parks 20 th-imp topic in psm

ALTHOUGH EPIDEMIC TYPHUS DISTRIBUTED THROUGH OUT THE WORLD PREVALENCE HAS DRASTICALLY REDUCED..NOW

SCRUB TYPHUS MC RICKETTSIAL DISEASE-PARKS

Tsutsugamushi disease (ie, scrub typhus): Cases are usually seen in rural south and southeast Asia, limited to the geographical

area bound by Japan, the Solomon Islands, and Pakistan. It is estimated that 1 million cases occur each year.

Rickettsialpox: This may be more prevalent worldwide than is reported. It has been identified in large cities in Russia, South

Africa, and Korea.

mc in india is scrub typhus {bhatia micro prof}

Q206. human live vaccine is available against which species of brucella;;

1b.melitensis;;

2b.abortus;;

3b.suis;;

4b canis ;;

ans-b.abortus REF.HARRISONS PG 976,ANANTH 7TH

In many developing nations, immunizations derived from the S19 vaccine have been evaluated in humans. In the former Soviet

Union, the administration of live S19 preparations were immunogenic, and protection was achieved and considered to last 1

year but caused a modest but notable incidence of clinical cases, as well as a hypersensitivity reaction. As such, S19 is not

safe vaccine candidate for human use

Q207. anaesthetic agent with least effect on myocardial contractility is ;;

1halothane;;

2trilene;;

3isoflurane;;

4ether;;

Q208. HEERFORDTS DISEAQSE consists of all, except ;;

1facial paralysis;;

2parotid lymphoma;;

3iridocyclitis;;

4 fever;;

ans-parotid lymphoma ref- harr 16 not given in 17

A peculiar course of sarcoidosis with enlargement of the parotid glands, mild fever, and uveitis, and facial nerve palsy. There

may be associated hyperalgesia, papilloedema, meningism and pleocytosis on the cerebrospinal fluid due to sarcoidosis. Other

clinical features may include hilar adenopathy, generalized lymphadenopathy, visceral lesion and skin rash. Young adults are

usually affected, males more often than females

Also known as:

Page 83: DocumentQ1

Heerfordt-Mylius syndrome

Heerfordt-Waldenström syndrome

Waldenström’s uveoparotitis

Q209. the drug of chice for POSNER-SCHLAUSUMANN SYNDROME IS ;;

1TIMOLOL;;

2TOPICAL STEROIDS;;

3PILOCARPINE;;

4BRIMONIDINE;

ANS-TOPICAL STEROIDS REF KANSKI 6TH

POSNER-Schlossman Syndrome (Glaucomatocyclitic crisis) is a condition with self-limited recurrent episodes of markedly

elevated intraocular pressure (IOP) with mild idiopathic anterior chamber inflammation. It is most often classified as second

inflammatory glaucoma

Pathophysiology

Episodic changes in the trabecular meshwork lead to impairment of outflow facility and result in an elevation of IOP. These

changes are accompanied by mild intraocular inflammation. In the acute phase of PSS, optic nerve head parameters and retinal

flow rates were altered; however, all returned to normal without any permanent damage after resolution of the elevated IOP.

Electroretinogram studies in the acute phase demonstrate a selective reduction in the S-cone b-wave.

Associations with immunogenetic factors also exist; in one study, the presence of human leukocyte antigen Bw54 (HLA

was found in 41% of patients

# Treatment recommendations include the following:

* Topical steroids - Prednisolone acetate 1% 1 gtt qid, followed by taper

* Topical antiglaucoma drops - Timolol 0.25-0.5% 1 gtt bid or equivalent, or dorzolamide 2% 1 gtt bid/tid or equivalent (Beta

blockers should be avoided in patients with asthma.)

* Systemic carbonic anhydrase inhibitors - Acetazolamide 250 mg PO qid

* Topical NSAIDs - Diclofenac 0.1% 1 gtt tid/qid or equivalent

* Oral NSAIDs - Indomethacin 75-150 mg/d PO

# Miotics and mydriatic agents seldom are used because they may have further deleterious effects on the blood-aqueous

barrier, and long-acting periocular steroids are frowned upon because of lingering IOP effects.

# NSAIDs reduce the inflammatory component by inhibiting the production of prostaglandins, and antiglaucoma medications

reduce the influx of new aqueous; both these effects rapidly control the IOP. This combination also avoids potential IOP

elevations caused by steroids in steroid-responsive patients.

Q210. the main action of trabeculectomy is ;;

1external drainage of aqueous;;

2hyposecretion of aqueous;;

3cyclodialysis;;

4increased uveoscleral outflow;;

During a trabeculectomy, the patient's eye is held open with a speculum. The outer layer, or conjunctiva, and the white of the

eye, or sclera, are cut open (A). A superficial scleral flap is created and a plug of sclera and underlying trabecular networ

removed (B). This allows the fluid in the eye to circulate, relieving pressure. The scleral flap is closed and sutured (C). The

conjunctiva is closed (D)

Intraocular pressure may be lowered by allowing drainage of aqueous humor from within the eye to the following routes: (1)

filtration through the sclerostomy around the margins of the scleral flap into the filtering bleb that forms underneath the

conjunctiva, (2) filtration through outlet channels in the scleral flap to underneath the conjunctiva, (3) filtration through

Page 84: DocumentQ1

connective tissue of the scleral flap to underneath the conjunctiva. into cut ends of Schlemm's canal, (4) aqueous flow into cut

ends of Schlemm's canal into collector channels and episcleral veins and (5) into a cyclodialysis cleft between the ciliary b

and the sclera if tissue is dissected posterior to the scleral spur

Back to

top

kollin. Guest

Posted: Mon Jul 26, 2010 12:05 pm Post subject:

Q211. CHASSAIGNAC TUBERCLE IS IS A LAND MARK FOR ;;

1stellate ganglion block;;

2trigeminal ganglion block;;

3celiac plexus block;;

4brachial plexus block;;

The Chassaignac tubercle is used as a landmark by anesthesiologists for local anesthesia of the brachial plexus and the cervi

plexus using the supraclavicular approach.

it is the name given to the anterior tubercle of the transverse process of the sixth cervical vertebra against which the carotid

artery may be compressed by the finger.

The patient is placed in the supine position with the neck slightly extended, the head rotated slightly to the side opposite

block, and the jaw open. The point of needle puncture is located between the trachea and the carotid sheath at the level of the

cricoid cartilage and Chassaignac's tubercle. Although the ganglion lies at the level of the C7 vertebral body, the needle is

inserted at the level of C6 to avoid the piercing the pleura. Cutaneous anaesthesia is obtained with a skin wheal of local

anaesthetic.

The sternocleidomastoid and carotid artery are retracted laterally as the index and middle fingers palpate Chassaignac's

tubercle. The skin and subcutaneous tissue are pressed firmly onto the tubercle to reduce the distance between the skin surface

and bone, and in an attempt to push the dome of the lung out of the path of the needle. When properly performed, this

manoeuvre is uncomfortable for the patient.

The needle is directed onto the tubercle, and then redirected medially and inferiorly toward the body of C6. After the body i

contacted, the needle is withdrawn 1-2 mm. This brings the needle out of the belly of the longus colli muscle, which sits

posterior to the ganglion and runs along the anterolateral surface of the cervical vertebral bodies. The needle is then held

immobile.

Needle position is confirmed by fluoroscopy. Spread of radiocontrast is confirmed by both anteroposterior and lateral vi

Failure of the solution to spread cephalad and caudad between tissue planes suggests intramuscular injection into the longus

colli muscle. Immediate dissipation of the solution indicates intravascular placement of the needle orifice.

A 10 ml control syringe charged with local anaesthetic is attached to the needle and aspiration is performed to rule out

intravascular placement. A 0.5 ml test dose is performed to rule out intravascular injection into the vertebral artery. The

usefulness of this test dose to provide early warning of intra-arterial injection is questionable, however, since seizures can occur

immediately, even with very small volumes of local anaesthetic. This test dose is followed by a 3 ml epinephrine-containing test

dose to rule out intravenous placement. The remainder of the anaesthetic (10-15 ml) is injected in divided doses of 3 ml with

intermittent aspiration. The patient is placed in the sitting position to facilitate the spread of anaesthesia inferiorly to the

stellate ganglion.

The onset of Horner's syndrome indicates a successful block.

Q212. ect is least effective in ;;

1obsessions;;

2neuroleptic malignant syndrome;;

3depression;;

4delirium;;

Page 85: DocumentQ1

the problem in psych is any damn thing which is not responding to drug therapy they will give ECT...so ect will b mentioned for

most of the psych conditions..as GMAX SAID approach by exclusion is best ..NMS N DEPRESSION definitely used..now

obsession severe enough to cause severe depression ECT USED WHEN ALL OTHER THERAPY FAILS...in delirium -an acute

confusional state occuring in various medicl,surgical,metabolic ,toxic or post op conditions-ECT IS HARDLY OF USE CAUSES

MORE HARM THAN BENIFIT EVEN IN RESISTANT STOPOUROUS CASES...

SO IT VL GO WITH GMAX'S ANS-DELIRIUM >OBSESSION

Q213. ALEXITHYMIA IS ;;

1.a feeling of intense rapture;;

2pathological sadness;;

3affective flattening;;

4inability to recognise and desribe feelings;;

Alexithymia is a relatively new term which means the inability to express feelings with words.

Q214. which of the following skin tuberculosis show -highest immunity;;

1tbvc;;

2lupus vulgaris;;

3lupus miliary;;

4tb chancre;;

Primary Inoculation Tuberculosis (Tuberculous Chancre; Tuberculous Primary Complex)

Primary inoculation tuberculosis results from the inoculation of mycobacteria into the skin of a host not previously infected with

tuberculosis. The tuberculous chancre and the affected regional lymph nodes constitute the tuberculous primary complex of the

skin.

INCIDENCE

Primary inoculation tuberculosis is estimated to constitute a very small proportion of all primary tuberculous lesions; however, it

may be not quite as rare as generally believed. In some regions, particularly in Asia where the incidence of tuberculosis is

high and where living conditions and hygiene are poor, primary inoculation tuberculosis of the skin is not unusual. Most affected

individuals are children, but the lesions may also occur in adolescents and young adults. All parts of the body may be affect

but sites of predilection are the face, hands, and lower extremities, which are readily injured. One-third of the lesions are found

on the mucous membranes of the conjunctiva and oral cavity.

PATHOGENESIS

Because tubercle bacilli cannot actively penetrate intact skin, they are introduced into the tissue at the site of minor abrasions

or wounds. Rarely, infection may occur in healthy individuals after sexual contact with patients suffering from genitourinary

tuberculosis. Oral lesions may be due to bovine bacilli from nonpasteurized milk and occur after mucosal trauma or tooth

extraction. The skin lesion appears 2 to 4 weeks after inoculation. Infection spreads to the regional lymph nodes, producing

tuberculous lymphadenitis; with increasing acquired immunity, the process is localized to the particular region involved. Primary

inoculation tuberculosis is initially a multibacillary type of skin tuberculosis that becomes paucibacillary as immunity deve

CLINICAL MANIFESTATIONS

The tuberculous chancre initially presents as a small papule, scab, or wound with little tendency to heal. A painless ulcer

develops, which may be quite insignificant or may enlarge to a diameter >5 cm ( Fig. 200-1). The lesion is shallow with a

granular or hemorrhagic base studded with miliary abscesses or covered by necrotic tissue. The ragged edges are undermined

and of a reddish-blue hue; as the lesions grow older, they become more indurated, with thick adherent crusts.

Wounds inoculated with tubercle bacilli may heal temporarily but break down later, giving rise to granulating ulcers. Mucosal

infections result in painless ulcers or fungating granulomas.

Inoculation tuberculosis of the finger may present as a painless paronychia; inoculations of mycobacteria in puncture wounds

Page 86: DocumentQ1

have resulted in subcutaneous abscesses. Slowly progressing, regional lymphadenopathy develops 3 to 8 weeks after the

infection (see Fig. 200-1) and may rarely be the only clinical symptom. After weeks or months, cold abscesses may develop

that perforate to the surface of the skin and form sinuses; the lymph nodes draining the primary glands may also be involved.

Body temperature may be slightly elevated; occasionally, lymph node enlargement, abscess formation, and perforation may

take a more acute course. Fever, pain, and inflammatory swelling of the surrounding tissues simulating a pyogenic infection are

present in half the cases.

HISTOPATHOLOGY

In the early phase, there is an acute nonspecific inflammatory reaction in both skin and lymph nodes, and mycobacteria are

easily detected. After 3 to 6 weeks, the infiltrate and the regional lymph nodes acquire a tuberculoid appearance and caseati

may occur.

DIAGNOSIS AND DIFFERENTIAL DIAGNOSIS

Lack of awareness of the condition is probably the most common reason for diagnostic error. Any ulcer with little or no

tendency to heal and unilateral regional lymphadenopathy in a child should always arouse suspicion. Acid-fast organisms can be

demonstrated in histologic sections or in smears obtained from the primary ulcer and draining nodes in the initial stages of the

disease, but may be difficult to find in older lesions. The diagnosis is verified by bacterial culture. The reaction to intradermal

PPD is negative initially and later converts to positive (see Fig. 200-1). A previous tuberculous infection should be excluded. The

differential diagnosis includes the primary complexes of syphilis, tularemia, cat-scratch fever, sporotrichosis, and ulcerative

lesions of other mycobacterioses as well as other forms of skin tuberculosis.

COURSE

Without treatment, the condition may last up to 12 months. Lesions heal by scarring, but in rare cases, lupus vulgaris develo

at the site of a healed tuberculous chancre; in more than 50 percent of patients the regional lymph nodes calcify. Usually, the

primary tuberculous complex yields satisfactory immunity, but reactivation of the disease may occur. Hematogenous spread

may give rise to tuberculosis of other organs, particularly of the bones and joints. Depending on the size of the inoculum and

the age and resistance of the host, primary inoculation tuberculosis may progress to acute miliary disease with fatal outcome

Erythema nodosum is a feature in approximately 10 percent of the cases.

Tuberculosis Verrucosa Cutis (Warty Tuberculosis)

This disorder is verrucous skin tuberculosis caused by exogenous reinfection in previously sensitized individuals.

INCIDENCE

In Western countries, tuberculosis verrucosa cutis is rare. It is the most frequent form of tuberculous skin disease in Hong

Kong, accounting for more than 40 percent of cases.

PATHOGENESIS

Tuberculosis verrucosa cutis is inoculation tuberculosis occurring in persons who have acquired a certain degree of immunity;

thus, skin tests are highly positive and it is a paucibacillary type of skin tuberculosis. Inoculation occurs at sites of minor

wounds or abrasions or, rarely, from the patient's own sputum. In the past, certain professional groups, particularly physici

pathologists, medical students, and laboratory attendants, were most liable to acquire warty tuberculosis from tuberculous

patients or from autopsy material (“verruca necrogenica,” “anatomist's wart,” “postmortem wart”). Farmers, butchers, and

knackers contracted the disease from tuberculous cattle, and in these cases, M. bovis was responsible. Children can become

infected by playing on contaminated ground, a frequent occurrence in underdeveloped countries.

CLINICAL MANIFESTATIONS

Tuberculosis verrucosa cutis usually occurs on the hands. In children, the sites of predilection are the lower extremities. The

lesions are asymptomatic and start as a small papule or papulopustule with a purple inflammatory halo; they become

hyperkeratotic and are often mistaken for a common wart. Slow growth and peripheral expansion lead to the development of a

verrucous plaque with an irregular outline and a papillomatous horny surface. Clefts and fissures discharging pus extend into

Page 87: DocumentQ1

the underlying brownish-red to purplish infiltrated base. The lesion usually is solitary, but multiple lesions may occur. The

regional lymph nodes are rarely affected but may become enlarged from secondary bacterial infection.

HISTOPATHOLOGY

The most prominent histopathologic features are pseudoepitheliomatous hyperplasia with marked hyperkeratosis, a dense

inflammatory infiltrate, and abscesses in the superficial dermis or within the pseudoepitheliomatous rete pegs. Epithelioid c

and giant cells are found in the upper and middle dermis; typical tubercles are uncommon. Mycobacteria can occasionally be

seen. At times, the dermal infiltrate may be nonspecific.

DIAGNOSIS AND DIFFERENTIAL DIAGNOSIS

Early lesions resemble warts or keratoses. Hyperkeratotic lupus vulgaris exhibits “apple-jelly” nodules at the periphery and

occurs in sites where tuberculosis verrucosa cutis is rare (see below). Blastomycosis, chromomycosis, and bromoderma may be

similar clinically and histopathologically but can be excluded by special stains and by culture. Negative fungal cultures and

tuberculoid foci are diagnostic aids. Chronic vegetating pyoderma and hyperkeratotic lesions due to other mycobacteria may be

difficult to exclude. Hypertrophic lichen planus is pruritic and more disseminated, and usually other cutaneous and mucosal

lesions are found. Tertiary syphilis is not quite as verrucous and is accompanied by diagnostic serology.

COURSE

The lesions progress slowly, and, if untreated, persist for many years. Secondary pyogenic infection may temporarily lead to

more acute inflammatory changes, with lymphangitis and regional lymphadenitis. Spontaneous involution does occur and

results in sunken atrophic scars. Occasionally, ulcerative and sclerotic lesions or fungating granulomas are seen.

Tuberculous Cellulitis

Two cases of “cellulitis” caused by M. tuberculosis were recently described in patients on long-term systemic glucocorticoid

therapy. 12 A mode of infection was not determined, although one of the patients had been treated previously for pulmonary

tuberculosis. Histopathologically, multinucleated giant cells, granulomas, and acid-fast bacilli were present. Both patients

responded to antituberculous antibiotics

Lupus Vulgaris

Lupus vulgaris is an extremely chronic and progressive form of tuberculosis of the skin occurring in individuals with moderate

immunity and a high degree of tuberculin sensitivity.

INCIDENCE

Although the incidence of lupus vulgaris has steadily declined during the past decades, it was estimated in 1960 that some

50,000 new cases occur throughout the world every year. It has always been less common in the United States than in Europe.

Females appear to be affected two to three times as often as males; all age groups are affected equally.

PATHOGENESIS

Lupus vulgaris is a postprimary, paucibacillary form of skin tuberculosis arising in previously sensitized individuals with only

moderate immunity. I

t originates by hematogenous, lymphatic, or contiguous spread from tuberculosis elsewhere in the body, most often from

cervical adenitis or pulmonary tuberculosis, but sometimes from an old, apparently quiescent primary complex.

Rarely, it follows primary inoculation tuberculosis or BCG vaccination 13 (see below). The lesions progress steadily.

Spontaneous involution may occur and new lesions may arise within old scars. Complete healing occurs only rarely without

therapy.

CLINICAL MANIFESTATIONS

The lesions are usually solitary, but two or more sites may be involved simultaneously; in patients with active pulmonary

tuberculosis, multiple foci may develop. In approximately 90 percent of patients, the head and neck are involved. Lupus

vulgaris usually starts on the nose or cheek and slowly extends onto adjacent areas. The earlobes are often affected, and

Page 88: DocumentQ1

solitary patches may be encountered on the scalp. Only a small percentage of the lesions occur on the extremities, and, except

for patients with disseminated lupus vulgaris, the trunk is rarely involved.

In general, lupus vulgaris is asymptomatic. The initial lesion is a brownish-red, soft or friable lupus macule or papule with a

smooth surface or covered by a scale. On diascopy the infiltrate exhibits a typical apple-jelly color. Progression is characterized

by elevation, a deeper brownish color ( Fig. 200-4), and formation of a plaque. Involution in one area and simultaneous

expansion in another result in a gyrate outline.

Lesions may become flat plaques with a serpiginous or polycyclic outline and a smooth surface or psoriasiform scaling; there

may be erosions, ulceration, and scarring. Hypertrophic forms appear as a soft mass with a nodular, hyperkeratotic surface.

Edema, lymphatic stasis, recurrent erysipelas, elephantiasic thickening, and vascular dilatation may lead to gross deformity.

ulcerative forms, the underlying tissue may be affected by progressive necrosis; involvement of the nasal or auricular cartilage

results in extensive destruction and disfigurement.

Atrophic scarring, with or without prior ulceration, is a prominent feature of lupus vulgaris. Recurrence within a scar is

characteristic. Sometimes, fibrosis is very pronounced and leads to deformations, mutilations, and contractures.

Lupus vulgaris of mucous membranes:-

The mucosae may be primarily involved or become affected by the extension of skin lesions. They show small, soft, gray or pin

papules, ulcers, or granulating masses that bleed easily. A dry rhinitis is often the only symptom of early nasal lupus, but

progressive lesions destroy the cartilage of the nasal septum. Scarring of the soft palate and stenosis of the larynx may als

result.

Lupus postexanthematicus:-

After a transient impairment of immunity, particularly after measles (thus the term lupus postexanthematicus), multiple

disseminated lesions may arise simultaneously in different regions of the body as a consequence of hematogenous spread from

a latent tuberculous focus. During and after the eruption, a previously positive tuberculin test may become negative but will

usually revert to positive as the general condition of the patient improves. Clinically and histopathologically, the lesions of

postexanthematic lupus are typical for lupus vulgaris, and their appearance distinguishes the condition from acute miliary

tuberculosis of the skin.

HISTOPATHOLOGY

The most prominent histopathologic feature is the formation of typical tubercles ( Fig. 200-9). Secondary changes may be

superimposed: epidermal thinning and atrophy or acanthosis with excessive hyperkeratosis or pseudoepitheliomatous

hyperplasia; acid-fast bacilli are usually not found. Nonspecific inflammatory reactions may partially conceal the tuberculous

structures. Old lesions are composed chiefly of epithelioid cells and may be impossible to distinguish from sarcoidal infiltr

DIAGNOSIS AND DIFFERENTIAL DIAGNOSIS

Typical lupus vulgaris plaques do not present diagnostic problems; they have to be distinguished from lesions of sarcoidosis,

lymphocytoma, discoid lupus erythematosus, tertiary syphilis, leprosy, blastomycosis or other deep mycotic infections, lupoid

leishmaniasis, and chronic vegetating pyodermas. Criteria helpful in the diagnosis are the softness of the lesions, the brownish

red color, and the slow evolution. The apple-jelly nodules revealed by diascopy are highly characteristic; finding them may be

decisive, especially in ulcerated, crusted, or hyperkeratotic lesions. Histologic examination and a positive culture for M.

tuberculosis/bovis confirm the diagnosis. The tuberculin test is strongly positive except for the early phases of

postexanthematic lupus.

COURSE

Lupus vulgaris is extremely chronic, and without therapy its course usually extends over many years or even decades. Thus,

adults or older patients tend to have more extensive lesions than children. The disease is progressive and leads to considera

impairment of function and to disfiguration. Contractions result in a reduction of joint mobility, and ulceration and destruction of

the cartilages of the face and scarring lead to disfigurement (see Fig. 200- , to ectropion with its complications, to

microstomia with impairment of speech and food intake, and to other severe mutilations.

****The most serious complication of long-standing lupus vulgaris is the development of carcinoma (see Fig. 200- . In the

Page 89: DocumentQ1

early 1900s, carcinoma was estimated to occur in almost 10 percent of patients. Squamous cell carcinomas outnumber basal

cell carcinomas by far, and the incidence of metastases is surprisingly high.

RELATIONSHIP OF LUPUS VULGARIS TO TUBERCULOSIS OF OTHER ORGANS In 40 percent of patients there is associated

tuberculous lymphadenitis, and 10 to 20 percent have pulmonary tuberculosis or tuberculosis of the bones and joints. The rate

of occurrence of pulmonary tuberculosis is 4 to 10 times higher in patients with lupus vulgaris than in the general populatio

some cases, lupus vulgaris may be regarded as a symptom of another tuberculous disease running a more serious course.

Scrofuloderma (Tuberculosis Colliquativa Cutis)

Scrofuloderma is a subcutaneous tuberculosis leading to cold abscess formation and a secondary breakdown of the overlying

skin.

PATHOGENESIS Scrofuloderma results from contiguous involvement of the skin overlying another tuberculous process, most

commonly tuberculous lymphadenitis, tuberculosis of bones and joints, or tuberculous epididymitis. It may affect all age

groups, although there is a higher prevalence among children, adolescents, and the aged.

It may be either a multibacillary or a paucibacillary form of skin tuberculosis.

CLINICAL MANIFESTATIONS

Tuberculosis colliquativa most often occurs in the parotidal, submandibular, and supraclavicular regions, as well as on the

lateral aspects of the neck; lesions may be bilateral. Lesions on the extremities or on the trunk accompany tuberculous disea

of the phalangeal bones, the joints, the sternum, and the ribs. Skin lesions first present as firm, subcutaneous nodules or as a

well-defined, freely movable asymptomatic infiltrate. As the infiltrate enlarges it softens, but it may take months before there i

liquefaction with subsequent perforation ( Fig. 200-10). Ulcers and sinuses develop and discharge watery and purulent or

caseous material. The ulcers are linear or serpiginous with undermined, inverted, bluish edges and uneven, soft, and

granulating floors. Sinusoidal tracts undermine the skin, and clefts and dissecting subcutaneous pockets alternate with soft

nodules; scar tracts develop and bridge ulcerative areas or even stretches of normal skin. Tuberculin sensitivity is usually

pronounced.

other types

tub gumma : poorly nourished children, hgous spread ,firm subcut nodules,undermined ulcer

oroficial tub:

immunocompromised

lips tongue genitalia reddish painful ulcer

due to auto inoculation from advanced internal git tb

remaining are tuberculids

lichen scruflosorum

papulonecrotic tuberculid

eryhthema nodosum

Q215. which of the following is least likely to present as an eccentric osteolytic lesion;;

1aneurysmal bone cyst;;

2giant cell tumor;;

3fibrous cortical defect;;

4simple bone cyst;;

ANS-SIMPLE BONE CYST REF-APLEY 8TH PG 178

READ THE TABLE V.IMP TOPIC

ECCENTRIC EXPANSILE LESION

1)aneurysmal bone cyst

Page 90: DocumentQ1

2)fibrous cortical defect

3)giant cell tumor

4)non ossifying fibroma

5)chondromyxoid fibroma

CENTRAL NON EXPANSILE

1)simple (unicameral bone cyst

2)enchondroma

3)chondroblastoma

Q216. MERCEDEZ BENZ SIGN IS SEEN IN ;;

1porcelain gallbladder;;

2limely bile;;

3gall stones;;

4emphysematous cholecystitis;

MERCEDEZ BENZ SIGN *

+ "Mercedes Benz" sign is a triradiate collection of nitrogen gas

# Fills crevices created by shrinkage of cholesterol crystals in the stone

# Radiolucent fissures usually widest centrally radiating like points of star

ANS-GALL STONE REF-BAILEY 25 PG 1115

The Mercedes-Benz sign describes a star-shaped pattern of gas-fissuring within gallstones initially described on an abdominal

radiograph . Fissures, usually fluid-filled, are present in close to 50% of gallstones. Less than half of these fissured gallstones

contain some amount of gas . The radiolucency caused by the gas usually appears in a triradiate pattern, mimicking the

Mercedes-Benz logo.

IT IS ALSO CALLED SEA-GULL SIGN

Q217. the term lead time is used in ;;

1pollution studies;;

2inventory management;;

3entomology;;

4rehabilitation;;

answer inventory management

goin logically answer v wud tempt to go for pollution studies as pollution will hav effect over a period of time nt immediately...

there s some time taken leadin to its effect... n it cud hav been answer if inventory management nt in choice.....

because its specifically use with it...

Q218. which of the following is not an example of non-random sampling;;

1quota sampling;;

2purposive sampling;;

3convenient sampling;;

4cluster sampling;;

Q219. which of the following is not a charecteristic of indian census;;

1international simultaneity;;

2delimited territory;;

3universality;;

Page 91: DocumentQ1

4dejure nature;;

Q220. which of the folowing is not an example of latent infection;;

1yaws;;

2plasmodium vivax;;

3tuberculosis;;

4brill-zinser disease;;

A lingering infection that may lie dormant in the body for a period of time but may become active under certain conditions.

Back to

top

dilshan. Guest

Posted: Tue Jul 27, 2010 12:23 pm Post subject:

Q219.

answer is dejure nature

int simultaneity... that its along wth int census every 1o years>> census movement..starts wth sunrise of 1st march

delimited territory..no territorial limit ..reacf far flung places... every area covered

indian census is de facto..not de jure... de jure census tallies ppl accord to their regular or legal residence...

where as de facto census allocates them to place ehere they prsnt at time of enumration,,,,,,,,,

FROM CENSUS OF INDIA WEB SITE

Some of the essential features of the census are as follows:

Sponsorship:

To conduct a census, vast organisation and considerable resources are needed. The census organisation has to mobilise

extensive administrative machinery with adequate legislative authority. This can be done only by National Government with the

cooperation of States and Local Governments.

Defined Territory:

The population figures have no meaning unless they refer to a well defined territory. The territory covered, alongwith any

changes in its area in successive censuses should be clearly and explicitly stated.

Universality

:Each person present and/or residing within its scope, without omission or duplication should be included in the census to

ensure completeness and accuracy of census data.

Simultaneity:The total population enumerated should refer to one well defined point of time and the data collected should also

refer to a well defined point or period of time. This is essential to an accurate count of the total population and relations

facts about the population to a specified period of time. As a general rule, a day is fixed for the census and also a particular

moment which is called the "census moment". In India, the "Census moment" now is the sunrise of 1st March, of the census

year. The fixing of the "census moment" helps the enumerators to decide which persons are to be included in the census. The

persons born after the census moment or the persons dying before the census moment are to be excluded from the census.

Some of the characteristics of the population like age, marital status, occupation, literacy and birthplace etc., are referre

period of time usually from 10th February to 28th February of the census year to ensure simultaneity.

Page 92: DocumentQ1

Defined Periodicity: Censuses should be taken at regular intervals so that comparable information is made available in a fixe

sequence. A series of censuses makes it possible to appraise the past, accurately describe the present and estimate the future.

Individual Units: Census data must be collected separately for each individual so that detailed classifications may be provid

all the required combinations. A procedure of "group enumeration" is not a census in the strict sense of the term because the

recording of aggregated or summarized information on the characteristics of a group of person usually precludes the cross

tabulation of data on several characteristics. Even though a well-designed "group enumeration" can produce cross-

classifications of certain individual characteristics, such as sex and age, the possibilities in this respect are so limited that the

procedure is not recommended for general use, particularly since it tends to result in under-enumeration of the population.

Individual enumeration does not preclude the use of sampling techniques for obtaining data on specified characteristics,

provided that the sample design is consistent with the size of the areas for which the data are to be tabulated and the degree of

detail in the cross-tabulations to be made.

Compilation and Publications: No census is complete unless the data collected are compiled and published by geographic

and by basic demographic variables. The unpublished data is of no use to the potential users of the census data. That is why

the census should be limited to very important items which can be tabulated and published in time.

International Simultaneity: The census of any country is of greater value nationally, regionally and internationally if it can be

compared with the censuses of other countries which are taken at approximately the same time. Censuses in most countries of

the world are conducted in years ending in 0 or 1. In India, the decennial censuses are conducted in years ending in 1.

International comparability is thus maintained. The Indian census accompanies all the above features.

Back to

top

kollin. Guest

Posted: Tue Jul 27, 2010 12:27 pm Post subject:

Q221. osteomalacia and painful bone fractures are commonly seen in chronic poisoning by ;;

1arsenic;;

2lead;;

3cadmium;;

4mercury;;

answer cadmium

Cadmium has no constructive purpose in the human body. Cadmium is extremely toxic even in low concentrations, and will

bioaccumulate in organisms and ecosystems.

Acute exposure to cadmium fumes may cause flu like symptoms including chills, fever, and muscle ache sometimes referred to

as "the cadmium blues." Symptoms may resolve after a week if there is no respiratory damage. More severe exposures can

cause tracheo-bronchitis, pneumonitis, and pulmonary edema. Symptoms of inflammation may start hours after the exposure

and include cough, dryness and irritation of the nose and throat, headache, dizziness, weakness, fever, chills, and chest pain.

Inhaling cadmium-laden dust quickly leads to respiratory tract and kidney problems which can be fatal (often from renal

failure). Ingestion of any significant amount of cadmium causes immediate poisoning and damage to the liver and the kidneys.

Compounds containing cadmium are also carcinogenic.

The bones become soft (osteomalacia), lose bone mineral density (osteoporosis) and become weaker. This causes the pain in

the joints and the back, and also increases the risk of fractures. In extreme cases of cadmium poisoning, mere body weight

causes a fracture.

Q222. a pregnant 35 year old patient isat highest risk for the concurrent development of which of the following malignancies;

Page 93: DocumentQ1

1cervix;;

2ovary;;

3breast;;

4vagina;;

answer cervix

ques direct pick from pre test seroes of obg ..ques no. 281..pg 231 answer given

Cervical cancer is a more common gynecologic malignancy in pregnancy than ovarian or breast cancer due to the fact that it is

a disease of younger women. Management of cervical intraepithelial lesions is complicated in pregnancy because of increased

vascularity of the cervix and because of the concern that manipulation of and trauma to the cervix can compromise continuatio

of the pregnancy. A traditional cone biopsy is only indicated in the presence of apparent microinvasive disease on a

colposcopically directed cervical biopsy. Otherwise, more limited procedures such as shallow coin biopsies are more appropria

If invasive cancer is diagnosed, the decision to treat immediately or wait until fetal viability depends in part on the gestational

age at which the diagnosis is made and the severity of the disorder. Survival is decreased for malignancies discovered later

pregnancy. Radiation therapy almost always results in spontaneous abortion, in part because the fetus is particularly

radiosensitive. Chemotherapy is associated with higher than expected rates of fetal malformations consistent with the

antimetabolite effects of agents used. Specific malformations depend on the agent used and the time in pregnancy at which the

exposure occurs.

* Late pregnancy or never being pregnant: Because estrogen levels are lower during pregnancy, breast tissue is exposed to

more estrogen in women who become pregnant for the first time after age 35 or who never become pregnant.

Q223. RETRACTILE MESENTERY MAY BE SEEN IN ;;

1ormonds disease;;

2gardners syndrome;;

3turners syndrome;;

4down syndrome;;

answer ormonds disease

also known as sclerosing retroperitonitis,characterised by diffuse fibrous overgrowth and chronic inflammation.may be ass with

similar process in the mediastinum,sclerosing cholangitis and riedel's thyroiditis and termed multifocal fibrosclerosis.thoug

idiopathic,the etiologic role of ergot derivative drugs and autoimmune reaction has been suggested

Circulating antibodies to ceroid, a lipoproteinaceous by-product of vascular atheromatous plaque oxidation, are present in more

than 90% of patients with retroperitoneal fibrosis. The relationship of this finding to the occurrence of fibrosis remains

uncertain. The early inflammatory reaction is predominated by T-helper cells, plasma cells, and macrophages, but is

subsequently replaced by collagen-synthesizing fibroblasts.

Medical therapy is initiated with prednisone (60 mg every other day, for 2 months). Following this initial therapy, prednisone is

gradually tapered off over the next 2 months. Therapeutic efficacy is assessed on the basis of patient symptomatology,

erythrocyte sedimentation rate, and diagnostic imaging. Cyclosporine, tamoxifen, or azathioprine may be used to treat patients

who are recalcitrant to the above regimen.

Q224. two parents are both affected with albinism;;but have a normal child;;which of the following terms best applies to this

situation;;

1allelic heterogeneity;;

2locus heterogeneity;;

3variable expressivity;;

4incomplete penetrance;;

Q225. agent of choice for obese patients;;

1desflurane;;

Page 94: DocumentQ1

2isoflurane;;

3sevoflurane;;

4halothane

Desflurane, sevoflurane,

and isoflurane are minimally metabolized and are therefore useful agents in the obese patient, with desflurane possibly

providing better hemodynamic stability. Sevoflurane provides rapid recovery, good hemodynamic control, infrequent incidence

of nausea and vomiting, and prompt regaining of psychological and physical functioning when compared to isoflurane.

Metabolism of volatile anesthetics is greater in obese than in normal-weight patients, which is reflected by a greater increase in

serum inorganic fluoride, including with sevoflurane, whose biotransformation has not been shown to result in significant

differences in plasma fluoride levels nor differences in pre- and postoperative liver function and renal function tests between

obese and nonobese patients. A potentially hepatotoxic reductive pathway metabolizes halothane in obese patients, resulting in

an increased incidence of “halothane hepatitis.” Fortunately, halothane is rarely FIGURE 36-8. A. The preformed head elevation

pillow. B. Proper head-elevated laryngoscopy position (HELP) placement with the head elevation pillow combined with a

standard intubating pillow as described in the text. needed in modern-day practice. Evidence does not support the suggestion

that significant delayed recovery and awakening from volatile anesthetic agents occurs in obese patients when compared to the

nonobese. Rapid elimination and analgesic properties make nitrous oxide an attractive choice for anesthesia in obese patients

but high oxygen demand in this patient population limits its use. Short-acting opioids at the lowest possible dose, combined

with a low solubility inhalation anesthetic, facilitate a more rapid emergence without increasing opioid-related side effects.

Cisatracurium possesses an organ-independent elimination profile and is a favorable nondepolarizing muscle relaxant for use

during maintenance of anesthesia. Vecuronium and rocuronium are also useful choices.

Dexmedetomidine, an alpha-2 agonist with sedative and analgesic properties, provides hemodynamic stability without

myocardial depression. It has no clinically significant adverse effects on respiration, which makes it an attractive agent fo

as an anesthetic adjunct in obese patients.55 Furthermore, it reduces the postoperative opioid analgesic requirements and their

subsequent detrimental respiratory depressant effects.

Ventilatory tidal volumes greater than 13 mL/kg offer no added advantages during ventilation of anesthetized morbidly obese

patients. Increasing tidal volumes further only increases the peak inspiratory airway pressure, end-expiratory (plateau) airway

pressure, and lung compliance without significantly improving arterial oxygen tension.57 Arterial oxygenation during

laparoscopy in morbidly obese patients is affected mainly by body weight and not body position, pneumoperitoneum, or mode

of ventilation, and oxygenation is not significantly improved by increasing either the respiratory rate or tidal volume.58 Po

end-expiratory pressure is the only ventilatory parameter that has consistently been shown to improve respiratory function in

obese subjects.59 PEEP may, however, decrease venous return and cardiac output.

Back to

top

Posted: Wed Jul 28, 2010 12:30 pm Post subject:

Q224.

ANS-incomplete penetrance ref-ROBBINS8, HARRISON 17 PG 396 VERY nicely explained

1)allelic heterogenity-I different mutations occurring within the same gene produce disorders, it is said to manifest allelic heterogeneity. This

term has been used when a number of different alleles cause a similar phenotype or different phenotypes.

Eg- Different FBN1 mutations causing Marfan's syndrome

Cystic fibrosis is caused by mutations in 136 different alleles

2)LOCUS HETEROGENITY -If mutations in unrelated gene loci cause a single disorder, it is referred to as locus heterogeneity

3)VARIABLE EXPRESSIVITY-This differs from penetrance, which refers to the likelihood of the gene generating any phenotype at all, while

expressivity refers to the influence of an expressed gene in individuals. Variable expressivity occurs when a phenotype is expressed to a

different degree among individuals with the same genotype. For example, individuals with the same allele for gene involved in a quantitative

trait like body height might have large variance some are taller than others, making prediction of the phenotype from a particular genotype

Page 95: DocumentQ1

alone difficult

4)incomplete penetrance or reduced penetrance. Penetrance is said to be reduced or incomplete when some individuals fail to express the

trait, even though they carry the allele.

Q226. MAX PROPENSITY TO FOR RELEASING HISTAMINE;;

1dtc;;

2pancuronium;;

3gallamine;;

4atrracurium;;

ans-dtc ref-MORGANS 4 TH,LEE12

MC CAUSE-Tubocurarine, the first muscle relaxant used clinically, often produced hypotension and tachycardia through histamine release; its

ability to block autonomic ganglia was of secondary importance. Histamine release could also produce or exacerbate bronchospa

its no more used now

OTHER LIKELY CAUSES -Histamine release from mast cells can result in bronchospasm, skin flushing, and hypotension from peripheral

vasodilation. Both atracurium and mivacurium are capable of triggering histamine release, particularly at higher doses. Slow injection rates

and H1 and H2 antihistamine pretreatment ameliorate these side effects

also mus.relax. least likely to cause histamin release-vecuronium

Q227. epidural blood patch is used in the treatment of ;;

1spinal headache;;

2cauda equino syndrome;;

3chronic back ache;;

4 none of the above;;

ans-spinal headache

Any breach of the dura may result in a postdural puncture headache (PDPH) also called SPINAL HEADACHE. This may follow a diagnost

lumbar puncture, a myelogram, a spinal anesthetic, or an epidural "wet tap" in which the epidural needle passed through the epi

and entered the subarachnoid space. Similarly, an epidural catheter might puncture the dura at any time and result in PDPH. An epidural wet

tap is usually immediately recognized as CSF dripping from the needle or aspirated from an epidural catheter. .

Typically, PDPH is bilateral, frontal or retroorbital, and occipital and extends into the neck. It may be throbbing or constant and associated

with photophobia and nausea. The hallmark of PDPH is its association with body position. The pain is aggravated by sitting or standing and

relieved or decreased by lying down flat. The onset of headache is usually 12–72 h following the procedure; however, it may be seen almost

immediately. Untreated, the pain may last weeks, and in rare instances has required surgical repair

An epidural blood patch is a very effective treatment for PDPH. It involves injecting 15–20 mL of autologous blood into the epidural space at,

or one interspace below, the level of the dural puncture. It is believed to stop further leakage of CSF by either mass effect or coagulation.

The effects may be immediate or may take some hours as CSF production slowly builds intracranial pressure. Approximately 90%

will respond to a single blood patch, and 90% of initial nonresponders will obtain relief from a second injection. Prophylactic blood patching

has been advocated by injecting blood through an epidural catheter that was placed after a wet tap. However, not all patients will develop

PDPH, and the tip of the catheter may be many levels away from the dural defect. Alternatively, a saline bolus can be injected through the

epidural catheter but does not appear to be as effective as blood patching. Most practitioners either offer the epidural blood patch when

PDPH becomes apparent or allow conservative therapy a trial of 12–24 h.

Q228. what is a straddle fracture;;

1fracture of the skull;;

2fracture of the c5 vertebra;;

3multiple rib fractures both sides of midline;;

4pelvic fractures;;

Page 96: DocumentQ1

ans-pelvic # ref-APLEYS 8TH,MAHESHWARI

straddle fracture-Bilateral fractures of the superior and inferior public rami

List of fracture eponyms

Aviator’s astragalus:Implies a variety of fractures of the talus; described after World War I as rudder bar is driven into foot during plane

crash.

Barton’s fracture:Displaced articular lip fracture of the distal radius; may be associated with carpal subluxatioin. Fracture configuration may

be in a dorsal or volar direction.

Bennett’s fracture: Oblique fracture of the first metacarpal base separating a small triangular fragment of the volar lip from the proximally

displaced metacarpal shaft.

Bosworth fracture: Fracture of the distal fibula with fixed displacement of the proximal fragment posteriorly behind the posterolateral tibial

ridge.

Boxer’s fracture: Fracture of the fifth metacarpalneck with volar displacement of the metacarpal head.

Burst fracture: Fracture of the vertebral body from axial load, usually with outward displacement of the fragments. May occur in the cervical,

thoracic, or lumbar spine.

Chance fracture: Distraction fracture of the thoracolumbar vertebral body with horizontal disruption of the spinous process, neural arch, and

vertebral body.

Chauffeur’s fracture (Hutchinson’s fracture): Oblique fracture of the radial styloid, initially attributed to the starting crank of an engine being

forcibly reversed by a backfire.

Chopart’s fracture and dislocation: Fracture and/or dislocation involving Chopart’s joints (talonavicular nd calcaneocuboid) of the foot.

Clay-shoveler’s (coal-shoveler’s) fracture: Spinous process fracture of the lower cervical or upper thoracic vertebrae. Injury initially

attributed to workers attempting to throw upward a full shovel of clay, but the clay, adhering to the shovel, would cause a sudde

force opposite to the neck musculature.

Colles’fracture: General term for fractures of the distal radius with dorsal displacememnt, with or without an ulnar styloid fracture.

Cotton’s fractures: Trimalleolar ankle fracture with fractures of both malleoli and the posterior lip of the tibia.

Die-punch fracture: Intraarticular fracture of the distal radius with impaction of the dorsal aspect of the lunate fossa.

Dupuytren’s fracture: Fracture of the distal fibula with rupture of the distal tibiofibular ligaments and lateral displacement of the talus.

Duverney’s fracture: Fracture of the iliac wing without disruption of the pelvic ring.

Essex-Lopresti’s fracture: Fracture of the radial head with associated dislocation of the distal radioulnar joint.

Galezzi’s fracture: Fracture of the radius in the distal third associated with subluxation of the distal ulna.

Greenstick fracture: Incompletely fractured bone in a child, with a portion of the cortex and periosteum remaining intact on the comp

side of the fracture.

Hangman’s fracture: Fracture through the neural arch of the second cervical vertebra (axis).

Hill-Sachs fracture: Posterolateral humeral head compression fracture caused by anterior glenohumeral dislocation and impaction o

humeral head against the anterior glenoid rim.

Page 97: DocumentQ1

Holstein-Lewis fracture: Fracture of the distal third of the humerus with entrapment of the radial nerve.

Jefferson’s fracture: Comminuted fracture of the ring of the atlas due to axial compressive forces. Fractures usually occur anterior and

posterior to the lateral facet joints.

Jones fracture: Diaphyseal fracture of the base of the fifth metatarsal.

Lisfranc’s fracture dislocation: Fracture and/or dislocation involving Lisfranc’s (tarsometatarsal) joint of the foot. Lisfranc was one of

Napoleon’s surgeons and described traumatic foot amputation through the level of the tarsometatarsal joint.

Maisonneuve’s fracture: Fracture of the proximal fibula with syndesmosis rupture and associated fracture of the medial malleolus or rupture

of the deltoid ligament.

Malgaigne’s fracture: Unstable pelvic fracture with vertical fractures anterior and posterior to the hip joint.

Mallet finger: Flexion deformity of the distal interphalangeal joint caused by separation of the extensor tendon from the distal phalanx. The

deformity may be secondary to direct injury of the extensor tendon or an avulsion fracture from the dorsum of the distal phalanx, where the

tendon inserts.

Monteggia’s fracture: Fracture of the proximal third of the ulna with associated dislocation of the radial head.

Nightstick fracture: Isolated fracture of the ulna secondary to direct trauma.

Posadas’ fracture: Transcondylar humeral fracture with displacement of the distal fragment anteriorly and dislocation of the radius and ulna

from the bicondylar fragment.

Pott’s fracture: Fracture of the fibula 2 to 3 in above the lateral malleolus with rupture of hte deltoid ligament and lateral subluxation of the

talus.

Rolando’s fracture: Y-shaped intraarticular fracture of the thumb metacarpal.

Segond’s fracture: Avulsion fracture of the lateral tibial condyle from the bony insertion of the iliotibial band.

Shepherd’s fracture: Fracture of the lateral tubercle of hte posterior talar process.

Smith’s fracture: Fracture of the distal radius with palmar displacement of the distal fragment. Also referred to as a reverse Colle’s fracture.

Stieda’s fracture: Avulsion fracture of the medial femoral condyle at the origin of the medial collateral ligament.

Straddle fracture: Bilateral fractures of the superior and inferior public rami.

Teardrop fracture: Flexion fracture/dislocation of the cervical spine with associated triangular anterior fragment of the involved vertebrae.

Injury complex in unstable, with posterior ligamentous disruption.

Tillaux’s fracture: Fracture of the lateral half of the distal tibial physis during differential closure of the physis. The medial part of the tibial

physis has already fused.

Torus fracture: Impaction fracture of childhood as the bone buckles instead of fracturing completely.

Walther’s fracture: Inschioacetabular fracture that passes through the pubic rami and extends towards the sacroiliac joint. The medial wall of

the acetabulum is displaced inward

Q229. which bone takes part in the formation of lateral longitudinal arch;;

1cuboid;;

2talus;;

Page 98: DocumentQ1

3medial cuneiform;;

4lateral cuneiform;;

answer cuboid

ref keith l moore

longitudinal arch of the foot is composed of medial and lateral parts. Functionally, both parts act as a unit with the transverse arch of the

foot, spreading the weight in all directions.

The medial longitudinal arch is higher and more important than the lateral longitudinal arch (Fig. 5.69A & D). The medial longitudinal arch

composed of the calcaneus, talus, navicular, three cuneiforms, and three metatarsals. The talar head is the keystone of the medial

longitudinal arch. The tibialis anterior, attaching to the 1st metatarsal and medial cuneiform, helps strengthen the medial longitudinal arch.

The fibularis longus tendon, passing from lateral to medial, also helps support this arch

The lateral longitudinal arch is much flatter than the medial part of the arch and rests on the ground during standing. It is made up of the

calcaneus, cuboid, and lateral two metatarsals.

Q230. korners septum , is formed by which of the following;;

1sqamous tympanic fissure;;

2lateral margin of tegmen tympani;;

3petrosqamous suture;;

4petrosqamous fissure;;

CT scan, temporal bone. An axial view through the superior portion of the temporal bone, which demonstrates the anterior (a) and posterior

(b) crura of the superior semicircular canal. The surrounding otic capsule is observed as dense white bone. The mastoid air cells (c) are

lateral to the otic capsule. The squamous air cells are separated from the petrous air cells by the Koerner septum (d). The posterior

of the temporal bone, or cerebellar plate, (e) forms the anterior margin of the posterior cranial fossa.

Posted: Mon Aug 02, 2010 12:42 pm Post subject:

Q241. most of surface of pinnae is supplied by

1greater auricular;;

2lesser occipital;;

3auricotemporal;;

4auricular branch of vagus;;

ans auriculotemporal

pinna:

Sensory

The auriculotemporal nerve (from cranial nerve V) to the tragus,crus of helix & adjacent part of helix.

The lesser occipital nerve (C2) to the upper 1/3 of the medial surface.

The greater auricular nerve (C2,3) to the posterior part of the lateral and most(lower 2/3rd) of the

Page 99: DocumentQ1

medial surfaces.

Arnold's nerve(auricular branch of Vagus) to the area around the concha at the external meatus &

corresponding area on medial surface.

facial nerve -distribute with arnold's nerve,supplies concha & retroauricular groove.

external auditory canal:

Nerve supply: Sensory only

Anterior wall & roof:auriculotemporal nerve.

Posterior wall & floor: Arnold’s nerve(auricular branch of vagus nerve)

posterior wall of auditory canal also receives sensory fibres of facial nerve through auricular branch of

vagus

tympanic membrane:

Nerve supply: Sensory only

anterior ½ of the lateral surface: The auriculotemporal nerve.

posterior ½ of the lateral surface: Arnold’s nerve

Medial surface: Jacobson’s nerve.(Tympanic branch of glossopharyngeal nerve)

note:Arnold's cough reflex-due to irritation of auricular branch of vagus nerve.also known as Ear

cough.

Q242. wave sign of mulvay or sail sign is seen in ;;

1craniopharyngioma;;

2fourth ventricular cyst;;

3thymic hyperplasia;;

4hydatid in lung;;

ANS-THYMIC HYPERPLASIA REF-sumer sethi 4th

On radiographs of children, the normal thymus is seen as a triangular structure with well-defined

borders. It may blend with the heart, or have a notch between its inferior border and heart.

Sometimes, it may

produce an acute angle between its inferior border and cardiac outline, producing the so-called ‘sail

sign’

The borders may be wavy in outline due to indentation by the costal cartilages; this has been named

the ‘wave sign of Mulvey’ or the ‘thymic wave sign’.

Q243. gobiff campaign in india is supported by which agency;;

1care;;

2rockfeller foundation;;

3bill and melenda gates foundation;;

4unicef;;

ans unicef...

the concept of "selective primary health care" was proposed to allow for the scarcity of resources

available to achieve health for all. It involved defining strategies focusing on priority health problems

(including infant and child mortality), using interventions that were feasible to implement, of low cost,

and with proven efficacy(25,26). UNICEF's GOBI strategy of 1982 emerged from this. At its foundation

were four child health interventions which met the above criteria and which were considered to be

synergistic -- growth monitoring (G), oral rehydration therapy for diarrhoea (O), the promotion of

breastfeeding (B) and childhood immunizations (I). Birth spacing/family planning (F), food

supplementation (F) and the promotion of female literacy (F) were added subsequently (GOBI-FFF)

Q244. which of the following is not located within the abductor canal;;

1 saphenous nerve;;

2 nerve to vastus medialis;;

3 femoral vein;;

4 deep femoral artery;;

Page 100: DocumentQ1

Q245. which one of the following human genes lacks introns;;

1 dystrophins;;

2 immunoglobuling;;

3 histone;;

4 ldl receptor;;

Q246. which type of mri demonstrates best anatomica; detail;;

1 t1 weighed;;

2 t2 weighed;;

3 spin-echo sequences;;

4 fat suppressed images;;

Q247. in the treatment of a burn ;;which of the following is a hypertonic formula;;

1haila formula;;

2warden formula;;

3brooke formula;;

4demling formula;;

Q248. stokes gritti operation is performed through the ;;

1ankle joint;;

2knee joint;;

3cervical spine;;

4 elbow;

Rocco Gritti, (10) a surgeon at the Ospedale Maggiore in Milan, Italy, described a through-knee

procedure in 1857 in which the patella and surrounding tissues were used as an osteo-plastic flap to

provide an end-bearing stump. Thirteen years later, the procedure was modified by Sir William Stokes,

(11) and the operation is now known as the Gritti-Stokes amputation

just above the level of the knee, to be covered by the skin in front of the patella (Stokes-Gritti

amputation). Both these end-bearing stumps are excellent. Properly fashioned, they will bear the body

weight well, without distress to the patient, and without the development of secondary troubles, due

to the artificial manner in which weight must be borne on any amputation stump. For both these

stumps, excellent prostheses have been devised, which fit and function well. If circumstances preclude

amputation at the level of the Syme or the Stokes-Gritti stump, it is necessary to fashion a stump

which will carry the weight of the body at its upper end,-below the knee, on the flaring condyles of the

tibia; and above the knee, on the ischial tuberosity. Stumps, therefore, are divided into two classes:

(1) those which bear weight upon the lower end; and (2) those which bear weight at or near the

upper end. In the latter group, with which this paragraph is concerned, the length of the stump is of

importance only for the purpose of motivating the artificial limb. Its useful length, therefore, is more

or less constant. From the foregoing, it is evident that there are four types of amplitations of the lower

ext,remity which are of value.

249. people with impaired dna ligase activity are immunodeficient;;dna ligase joins;;

1parallel and antiparallel strands;;

2okazaki fragments;;

3primer and template strands;;

4major and minor grooves;;

answer 2

Seals the single strand nick between

the nascent chain and Okazaki fragments

on lagging strand

Page 101: DocumentQ1

vinay. Guest

Q250. one of the following is an example of atavistic epiphysis;;

1head of first metacarpal;;

2head of femur;;

3coracoid process of scapula;;

4trochanters of femur;;

Back to top

Posted: Tue Aug 03, 2010 12:44 pm Post subject:

Q251. which of the following drug is most likely to cause impaired glucose tolerance;;

1sulfasalazine;;

2azathioprine;;

3methotrexate;;

4tacrolimus;;

ans-tacrolimus ref kdt 5th

Side effects can be severe and include infection, cardiac damage, hypertension, blurred vision, liver

and kidney problems (tacrolimus nephrotoxicity), hyperkalemia, hypomagnesemia, hyperglycemia,

diabetes mellitus, itching, and various neuropsychiatric problems such as loss of appetite, insomnia,

Posterior reversible encephalopathy syndrome, confusion, weakness, depression, cramps, neuropathy,

seizures, tremors, and catatonia

Q252. all of the following are projective personality tests, except ;;

1rorschach inkblot test;;

2thematic appreciation test;;

3sentence completion test;;

4bender gesalt test;;

The Bender Gestalt Test, or the Bender Visual Motor Gestalt Test, is a psychological assessment

instrument used to evaluate visual-motor functioning and visual perception skills in both children and

adults. Scores on the test are used to identify possible organic brain damage and the degree

maturation of the nervous system. The Bender Gestalt was developed by psychiatrist Lauretta Bender

in the late nineteenth century.

Q253. in which of the following condition -snow banking is seen ;;

1leprotic uveitis;;

2candidiasis;;

3para planitis;;

4fuchs iridocyclitis;;

ANS-PARS PLANITIS REF KANSKI 6TH PG456

SNOW BANKING SEEN IN INTERMEDIATE UVETIS (pars planitis)

In early cases cells are seen in anterior vitreous. Later they assume a sheet-like configuration & small

gelatinous exudates (

'snowballs' or 'cotton-balls' ) appear.

Page 102: DocumentQ1

vinay. Guest

Posterior vitreous detachment is common..

The hallmark is the presence of a grey-white fibro vascularplaque involving the inferior pars plana. The

plaque which is referred to as 'snowbanking.

Back to top

Posted: Wed Aug 04, 2010 1:03 pm Post subject:

Q253. most important toll like receptors in human is ;;

1tlr-2

2tlr -4;;

3tlr-5;;

4tlr-9;;

Q254. which of the following is not a component of moros reflex;;

1extension of elbow;;

2flexion of shoulder;;

3opening of hand;;

4abduction of shoulder;;

Q255. leucocyte adhesion def type 2 diseae is due to a defect in ;;

1beta chain of cd11/cd18 integrins;;

2fucosyl transferase;;

3mpo-hp2 system;;

4protein involved in organelle membrane docking and fusion;;

ans-fucosyl transferase REF-ROBBINS 8TH pg50

Leukocyte adhesion deficiency type II is extremely rare; only a handful cases have been reported and

most of them are of Middle Eastern decent. It is a defect in the expression of ligands for selectins due to

lack of enzymes required for expression of selectin ligands. Patients have leukocytosis, recurrent

infections (more prominent in infants and toddlers), and severe growth and mental retardation. This

disease is a defect in fucose metabolism (lack of fucosylation of the carbohydrate selectin ligands) that

results in failure to express the ligand for E and P selectin, sialyl Lewis-X (CD15s) expressed on

leukocytes and endothelial cells. The patients are unable to fucosylate other glycoproteins, including the

H blood group polysaccharide.

fucosyl transferase

Leukocyte adhesion deficiency type I (LAD I)

is a failure to express CD18, which composes the common ß2 subunit of LFA1 family (ß2 integrins).

CD11a/CD18 (LFA-1) expressed on lymphocytes is known to play an important role in lymphocyte

trafficking (adhesion to vascular endothelium), as well as interactions to antigen presenting cells (APC).

LFA-1 also plays a role of cytotoxic killing by T cells. Another member of this family is CD11bCD18

(MacA or CR3) and CD11cCD18(CR4). These 2 members mediate leukocyte adhesions to endothelial

cells but they also serve as receptors for iC3b (inactivated C3b). These patients succumb to life-

Page 103: DocumentQ1

vinay. Guest

threatening infection, usually within 2 years of life in severe cases of leukocyte adhesion deficiency I

(<1% expression of CD18). In milder forms of leukocyte adhesion deficiency I (1-30% expression of

CD8), patients may survive to adulthood.

Leukocyte adhesion deficiency type II

is extremely rare; only a handful cases have been reported and most of them are of Middle Eastern

decent. It is a defect in the expression of ligands for selectins due to lack of enzymes required for

expression of selectin ligands. Patients have leukocytosis, recurrent infections (more prominent in

infants and toddlers), and severe growth and mental retardation. This disease is a defect in fucose

metabolism (lack of fucosylation of the carbohydrate selectin ligands) that results in failure to express

the ligand for E and P selectin, sialyl Lewis-X (CD15s) expressed on leukocytes and endothelial cells. The

patients are unable to fucosylate other glycoproteins, including the H blood group polysaccharide.

Patients with leukocyte adhesion deficiency II manifest the Bombay phenotype (ie, negative for O and H

blood group antigens with potential production of anti-H antibody). The immunoglobulin M (IgM) and

immunoglobulin G (IgG) heavy chains are also not fucosylated. However, IgM and IgG serum levels are

within the reference range in patients with leukocyte adhesion deficiency II.

Leukocyte adhesion deficiency II may be classified as one of the congenital disorders of glycosylation

(CDG), a rapidly expanding group of metabolic syndromes with a wide symptomatology and severity. All

stem from dysfunctional N -glycosylation of proteins. Currently, 18 subtypes have been reported: 12 are

type I (dysfunctional lipid-linked oligosaccharide precursor synthesis), and 6 are type II (dysfunctional

trimming/processing of the protein-bound oligosaccharide), including leukocyte adhesion deficiency II

(CDG-IIc).

Back to top

aayush. Guest

Posted: Mon Aug 09, 2010 11:46 am Post subject:

Q254. A pregnant woman positive for hepatitis B surface antigen, anticore antibody IgM, e-antigen, and

hepatitis B DNA. After delivery, appropriate treatment of the baby would be:

A. Checking LFT in a week before initiating treatment

B. Administration of I/V serum immune globulin

C. Administration of hepatitis B immune globulin followed by hepatitis B vaccination

D. Administration of hepatitis B vaccination alone

Back to top

Posted: Tue Aug 17, 2010 12:47 pm Post subject:

254. Charecteristic radiological feature of transient tachypnoea of newborn is ;;

Page 104: DocumentQ1

aayush. Guest

1. reticulogranular appearance.

2. low volume lungs.

3. prominent horizontalfissure.

4. air bronchogram.

The characteristic findings include prominent perihilar streaking, which correlates with the engorgement

of the lymphatic system with retained lung fluid, and fluid in the fissures. Small pleural effusions may be

seen. Patchy infiltrates have also been described.

255. X-ray films are least sensitive to which coloured light;;

1. violet.

2. blue.

3. yellow.

4. red.

256. The most common finding on chest radiograph done early during the course of p.jirovici pneumonia

is ;;

1. normal film.

2. ground glass appearance.

3. perihilar infiltrates.

4. pleural effusion.

257. On MRI-the differential dignosis of spinal cord oedema is;

1. myelodysplasia.

2. myelomalacia.

3. myeloschisis.

4. cord tumors.

258. mould therapy is used in the treatment of ;;

1. pelvic cancer.

2. lung cancer.

3. prostate cancer.

4. skin cancer

Back to top

Posted: Tue Aug 17, 2010 12:52 pm Post subject:

Q:: Hot water Bag for abdomen colic works by inhibiting::

a. cold receptors on skin

b. Adrenergic receptors

c. Cholinergic receptors

D. Peritoneal sensory supply

Q:: Transfer of carbon diaoxide is said to be diffusion limited. because

a. the binding of co to hb is of high avidity

b. the diffusion of co across blood gas barrier is slow

Page 105: DocumentQ1

aamir. Guest

c. alveoli basement memb is less permeable to co

d. the partial pressure of co rises rapidly in blood the moment it is exposed to it.

Q:: corneal tatooing is C/I

a. traumatic loss of iris

b. adherant leukoma

c. iris coloboma

d. leucoma

Q:: Essential Atrophy of choroid is due to inborn error of metabolism of::

a. cysTine

b. lySine

c. aRginine

d. oRnithine

e. Cysteine

Back to top

Posted: Tue Aug 17, 2010 12:53 pm Post subject:

Q259. preg induced changes accountnfor about 9 to 10 kgnof wt gain in normal cond.wt gain blood vol is

a. 3.4 kg

b. 0.9 kg

c. 1.6 kg

d. 0.45 kg

Q260: Hot water Bag for abdomen colic works by inhibiting::

a. cold receptors on skin

b. Adrenergic receptors

c. Cholinergic receptors

D. Peritoneal sensory supply

Q261: Transfer of carbon diaoxide is said to be diffusion limited. because

a. the binding of co to hb is of high avidity

b. the diffusion of co across blood gas barrier is slow

c. alveoli basement memb is less permeable to co

d. the partial pressure of co rises rapidly in blood the moment it is exposed to it.

Q262: Intravascular hemolysis in hemolytic anemia is associated decreased serum haptoglobulin which

condition maskes this

a. pregnancy

b. parencymal liver disease

c. bile duct obs

d. malnutrition

Q263. a pt wth hereditary blood clotting problems presents wth pain in back of her knee.ateriogram

reveals bld clot in popliteal artery at its prox end.which of the arteries allow blood to reach foot??

a. post tibial

b. peroneal

Page 106: DocumentQ1

aayush. Guest

c. lat circumflex femoral

d. superior medial genicular artery

Q264. corneal tatooing is C/I

a. traumatic loss of iris

b. adherant leukoma

c. iris coloboma

d. leucoma

Q265. Essential Atrophy of choroid is due to inborn error of metabolism of::

a. cysTine

b. lySine

c. aRginine

d. oRnithine

e. Cysteine

Back to top

Posted: Wed Aug 18, 2010 12:49 pm Post subject:

q 259. preg induced changes account for about 9 to 10 kg of wt gain in normal cond. wt gain in blood vol

is

a.3.4 kg

b.0.9 kg

c.1.6 kg

d.0.45 kg

ans-c)1.6 kg

ref.dutta5th, williams23rd

# The average weight gain in pregnancy is 10-12 kg.

# This increase occurs mainly in the second and third trimesters at a rate of 350-400 gm/week.

# Six kg of the average 11 kg weight gain is composed of maternal tissues (breast 0.5 kg, fat-3.5kg,

blood-1.5 kg and uterine tissue-1kg) and 5 kg of foetus, placenta and amniotic fluid.

# Of this 11 kg, 7 kg are water, 3 kg fat and 1 kg protein

Blood Volume

* The total blood volume increases steadily from early pregnancy to reach a maximum of 35-45% above

the non-pregnant level at 32 weeks.

* Plasma volume increases by 40% whereas red cell mass increases by 20% leading to haemodilution

(Physiological anaemia)

Q:260: Hot water Bag for abdomen colic works by inhibiting::

Page 107: DocumentQ1

a. cold receptors on skin

b. Adrenergic receptors

c. Cholinergic receptors

D. Peritoneal sensory supply

ans- REF.GANONG 21ST PG 143

ans is -ADRENERGIC receptors

visceral pain is carried by UNMYELINATED C adrenergic fibres.

it works by GATE CONTROL HYPOTHESIS SEEN IN COUNTER IRRTANTS USED ON SKIN

Q:262: Intravascular hemolysis in hemolytic anemia is associated decreased serum haptoglobulin which

condition maskes this

a. pregnancy

b. parencymal liver disease

c. bile duct obs

d. malnutrition

ANS-c) bile duct obstruction ref-robbins 8th pg 642, Cecilmedicine23

Haptoglobin is a protein that in humans is encoded by the HP gene. In blood plasma, haptoglobin binds

free hemoglobin (Hb) released from erythrocytes with high affinity and thereby inhibits its oxidative

activity. The haptoglobin-hemoglobin complex will then be removed by the reticuloendothelial system

(mostly the spleen).

In clinical settings, the haptoglobulin assay is used to screen for and monitor intravascular hemolytic

anemia. In intravascular hemolysis free hemoglobin will be released into circulation and hence

haptoglobin will bind the Hb. This causes a decline in Hp levels. Conversely, in extravascular hemolysis

the reticuloendothelial system, especially splenic monocytes, phagocytose the erythrocytes and

hemoglobin is not released into circulation and hence haptoglobin levels are normal.

Higher-than-normal levels may be due to:

* Acute rheumatic disease

* Biliary obstruction

* Peptic ulcer

* Ulcerative colitis

* Other inflammatory conditions

Lower-than-normal levels may be due to:

* Chronic liver disease

* Drug-induced immune hemolytic anemia

* Erythroblastosis fetalis

* Hematoma

* Hemolytic anemia due to G6PD deficiency

* Idiopathic autoimmune hemolytic anemia

* Immune hemolytic anemia

* Primary liver disease

* Transfusion reaction

Q. 264. corneal tatooing is C/I

a. traumatic loss of iris

Page 108: DocumentQ1

divakar. Guest

b. adherant leukoma

c. iris coloboma

d. leucoma

ans-b)adherant leucoma ref-internet

The leading reason for corneal tattooing is to cosmetically alter the appearance of the eye. Usually, the

need for this alteration stems from corneal opacities. Corneal opacities, the scarring of the cornea which

creates an opaque or semi-transparent area on the eye, can be caused by leucoma, keratitis, and

cataracts. These opacities can be cosmetically disruptive for patients in their everyday lives. Tattooing

the cornea can cosmetically alter the discoloration and therefore blend the opacity to a normal eye color.

Occasionally, corneal tattooing can be performed in cases of albinism, aniridia, coloboma, iridodialysis,

keratoconus, or diffused nebulae of the cornea. Corneal tattooing is also occasionally performed on eyes

that still have vision in order to reduce the symptomatic glare associated with large iridectomies or

traumatic iris loss

Suggested contraindications to corneal tattooing procedures are adherent leucoma, keratectasia, anterior

staphyloma, neurotrophic cornea.

Q. 265. Essential Atrophy of choroid is due to inborn error of metabolism of::

a. cysTine

b. lySine

c. aRginine

d. oRnithine

e. Cysteine

ans-ornithine ref-KANSKI 6TH pg 692

GYRATE ATROPHY (GA) of the choroid and retina is a rare, autosomal recessive disease causing

progressive chorioretinal degeneration resulting in blindness. It is caused by a deficiency of ornithine -

aminotransferase (OAT)

Back to top

Posted: Wed Aug 18, 2010 12:52 pm Post subject:

:: In artificial rupture of membranes fetal heart rate shows::

a. acceleration

b. deceleration

c. variable deceleration

d. any of the above.

ans-acceleration or variable deceleration

RCOG STUDY(1980)-most frequently observed patterns were short-lived FHR acceleration and increased

band-width. Short-lived bradycardia was recorded in only few patients after amniotomy. The majority of

the alterations observed were not indicative of fetal distress as a direct result of artificial membrane

rupture.

Page 109: DocumentQ1

pub med study(1977)-This study reports the effects of spontaneous rupture of membranes and artificial

rupture of membranes on fetal heart rate patterns during labour in 87 normal and 25 complicated

pregnancies. The incidence of early deceleration patterns following membrane rupture was 6.25 per cent

during the first 15 minutes and 1.78 per cent after 45 minutes, regardless of whether the pregnancy was

normal or complicated. It is concluded that rupture of membranes does not significantly increase the

incidence of early deceleration patterns. However, the occurrence of other heart rate alterations such as

late decelerations, loss of beat-to-beat variability, changes in base line and tachycardia, need to be

investigated further in larger collaborative studies.

Q. Splenic macrophages in gauchers disease differ from those in ceroid histiocytosis by staining positive

for::

a. lipids

b. phospholipids

c. acid fast stain

d. iron

ans-lipids ref-ROBBINS 8TH pg 153,

Gaucher’s disease

Autosomal recessive disease, due to accumulation of glucocerebroside (a sphingolipid) in

reticuloendothelial cells in liver, spleen and bone marrow, due to a defect in lysosomal beta-

glucocerebrosidase

Increased risk (14x) of hematologic malignancies and 4x for other malignancies

Type 1 - chronic nonneuronopathic form - often completely asymptomatic; disease discovered

incidentally; does not involve the nervous system, high prevalence among Ashkenazi Jews (1/12 are

carriers)

Type 2 - fatal neurodegenerative disorder of infancy, similar to Tay-Sachs disease

Type 3 - slowly progressive neurologic disease with survival into adulthood

Treatment: glucocerebrosidase (enzyme replacement therapy)

Gross: massively enlarged spleens up to 10 kg

m in diameter and have one or more dark eccentrically placed nuclei. Periodic Acid Schiff (PAS) Staining

is usually intensely positive. With the electro microscope, the fibrillary cytoplasm can be resolved as

elongated distended lysosomes, containing the stored lipid in Stacks of BilayerµMicro images: Gaucher

cells rarely appear vacuolated but instead have a fibrillary type of cytoplasm likened to “crumpled tissue

paper”. Gaucher cells are often enlarged, sometimes up to 100

Positive stains: iron, PAS (but weak)

Negative stains: phospholipids stains, acid-fast stains

DD: chronic myelogenous leukemia (similar looking cells)

Ceroid-containing histiocytes are described for the first time in the spleen and marrow of a patient with

ITP. Ceroid results from the oxidation and polymerization of unsaturated lipids and may appear in

histiocytes in a variety of diseases. With the Wright or Giemsa methods, ceroid granules are stained sea-

blue, and ceroid-containing macrophages appear as "sea-blue histiocytes." This sea-blue color is not

Page 110: DocumentQ1

divakar. Guest

pathognomonic for ceroid. Therefore, the finding of seablue histiocytes in the bone marrow should be

followed by histochemical studies to confirm the presence of ceroid. The spleen from a case of the

recently described "syndrome of the sea-blue histiocyte" showed numerous ceroid-containing

macrophages. In view of the lack of specificity of the sea-blue histiocyte, it is suggested that the

syndrome be renamed "idiopathic ceroid histiocytosis of spleen and marrow."

Back to top

Posted: Wed Aug 18, 2010 12:58 pm Post subject:

Q.268:: Which of the following is not seen in interphotoreceptor matrix::

(MK)

a. Metalloproteinase

b. Sialoprotein Associated with Rods and Cones

c. Tissue Inhibitors of Metaloproteinases (TIMP)

d. Memicane.....ans!!

a. Glycoproteins

B. Metalloproteinase....ans!!

c. Proteoglycans

d. Glycolipids

For the interphotoreceptor matrix, the answer is memicane as memicane if given in the option is not at all

found in the interphotoreceptor matrix but if the options of Across are taken into account, then the

concentration of metalloprotinase's is least in the retina with respect to the other 3 so both the books are

right.

Q269:: all show miliary mottling except::

a. HMD

b. Klebsiella

C. Mitral Stenosis

d. Sarcoidosis

e. Stap. Pneumonia

D/d for miliary mottling:

Miliary tuberculosis

Varicella pneumonia(and other viral pneumonia's)

Acute hypersensitivity pneumonitis

Fungal pneumonia

Thyroid cancer metastases

Renal cell carcinoma metastases

Staphylococcal pneumonia

Tropical eosinophilia

Aspiration pnemonia

Pnemocystis carinni pneumonia

Sarcoidosis

Lymphoma

Leukemia

Page 111: DocumentQ1

Guest

Methotrexate lung disease

Uremic lung disease

Pulmonary alveolar proteinosis

Pulmonary alveolar microlithiasis

Idiopathic pulmonary hemosidirosis

Ref : Differential diagnosis in pediatrics by Suraj Gupte.4/e.

Q270:: chalky white optic disc on fundus A/E::

a. syhilis

b. lebers

c. Post papilledema Optic neuritis (ans:: across)

d. trauma to optic nerve (ans:: MK)

Across is right in c/o chalky white optic disc on ophthalmoscopy. The reason is : In conditions with primary

optic atrophy (eg, pituitary tumor, optic nerve tumor, traumatic optic neuropathy, multiple sclerosis),

optic nerve fibers degenerate in an orderly manner and are replaced by columns of glial cells without

alteration in the architecture of the optic nerve head. The disc is chalky white and sharply demarcated,

and the retinal vessels are normal. Lamina cribrosa is well defined.

Back to top

Page 112: DocumentQ1

aayush. Guest

Posted: Wed Aug 18, 2010 12:59 pm Post subject:

271. the chief surgical risk to which pt wth polycyathemia vera are exposed is that due to

a. anemic disturb

b. hge

c. infection

d. renal dysfunc

272. posner schlossmann synd typically affects young adults..40% of them positive for

a. hla bw 54

b. hla b 27

c. hla b 5

d. hla dr4

273. which of the following will decrease the oxygen consumption of resp muscles

a. a dec in lung compliance

b. a dec in airway resistance

c. a dec in diff capacity of lung

d. a inc in rate of resp

Back to top

Posted: Thu Aug 19, 2010 1:12 pm Post subject:

Glaucomatocyclitic crisis is a condition with self-limited recurrent episodes of markedly elevated

intraocular pressure (IOP) with mild idiopathic anterior chamber inflammation. It is most often classified

as secondary inflammatory glaucoma.

In 1948, Posner and Schlossman first recognized glaucomatocyclitic crisis and described the features of

this syndrome.1 For this reason, the entity is often termed Posner-Schlossman syndrome (PSS).

Posner and Schlossman identified the following features2 :

Recurrent episodes of mild cyclitis

Uniocular involvement

Duration of attack varying from a few hours to several weeks

Signs of a slight decrease in vision, elevated IOP with open angles, corneal edema with a few keratic

precipitates, heterochromia with anisocoria, and a large pupil in the affected eye

Normal visual fields

Normal optic disc

Normal IOP and outflow facility, and all provocative tests normal between episodes

Associations with immunogenetic factors also exist; in one study, the presence of human leukocyte

antigen Bw54 (HLA-Bw54) was found in 41% of patients

Differential Diagnoses

Anisocoria

Glaucoma, Uveitic

Glaucoma, Angle Closure, Acute

Ocular Hypertension

Page 113: DocumentQ1

aayan. Guest

Glaucoma, Angle Closure, Chronic

Glaucoma, Primary Open Angle

Glaucoma, Unilateral

Back to top

aayan. Guest

Posted: Thu Aug 19, 2010 1:13 pm Post subject:

Q269:: all show miliary mottling except::

a. HMD

b. Klebsiella

C. Mitral Stenosis

d. Sarcoidosis

e. Stap. Pneumonia

D/d for miliary mottling:

Ref: harrison/ Grainger

bacterial:: Miliary tuberculosis, brucellosis, broncho pneumonia

fungal:: histoplasma, coccidiodomy, blastomycosis

viral:: Varicella pneumonia(and other viral pneumonia's)

allergic:: Loefflers, tropical eosinophilia, drug reaction

neoplastic:: secondaries, leukemia, lymhoma

cardiac:: mitral stenosis, pulmonary edema, multi pulm infract

Pneumoconiosis

Misc:: sarcoiosis, RA, Hemosiderosis, HMD, Interstitial Pulm fibrosis

ans: staph. pneumonia and klebsiella, b+e

Back to top

Posted: Thu Aug 19, 2010 1:14 pm Post subject:

Q 275:: In Oguchi s disease ::

a. ERG is abnormal

Page 114: DocumentQ1

aayan. Guest

b. associated with vita A def

c. visual acuity is under normal photopic conditions

d. abnormality of visual purple

e. cellular differentiation of retina is normal

Q:: 276 Most severe Papillodema will be caused by::

a. Pituitary tumor

b. Frontal lobe tumor

c. post cranial fossa tumor

d. medulloblastoma

Back to top

Posted: Fri Aug 20, 2010 12:41 pm Post subject:

Q.276:: Bispectral Index is used to ::

a. measure Fio2

b. predict difficult intubation

c. measure obesity

d. measure depth of anesthesia (ans)

Bispectral index (BIS) is one of several recently developed technologies which purport to monitor depth of

anesthesia. BIS monitors can replace or supplement Guedel's classification system for determining depth

of anesthesia. Titrating anesthetic agents to a specific bispectral index during general anesthesia in adults

(and children over 1 year old) allows the anesthetist to adjust the amount of anesthetic agent to the

needs of the patient, possibly resulting in a more rapid emergence from anesthesia. Use of the BIS

monitor may reduce the incidence of intraoperative awareness in high risk procedures or patients and may

also have a role in predicting recovery from severe brain injury.[citation needed] The introduction of BIS

to the intensive care environment allows physicians and nurses to titrate sedative drugs safely, and to

monitor the patient (together with measurement of intracranial pressure) during therapeutic burst

suppression

bispectral index is a statistically based, empirically derived complex parameter. It is a weighted sum of

electroencephalographic subparameters, including a time domain, frequency domain, and high order

spectral subparameters. The BIS monitor provides a single dimensionless number, which ranges from 0

(equivalent to EEG silence)to 100 (equivalent to fully awake and alert). A BIS value between 40 and 60

indicates an appropriate level for general anesthesia, as recommended by the manufacturer.

Q 277:: Prognosis in a newborn with congenital diaphragmatic hernia is most closely related to ::

a. size of hernial defect

b. extent of herniation

c. degree of pulmonary hypoplasia

d. associated congenital anomalies

ANSWER degree of pulm hypoplasia

Ventilatory predictors of pulmonary hypoplasia in congenital diaphragmatic hernia, confirmed by

morphologic assessment.

Bohn D, Tamura M, Perrin D, Barker G, Rabinovitch M.

Page 115: DocumentQ1

Abstract

We carried out a prospective study in 66 infants with congenital diaphragmatic hernia within the first 6

hours of life to determine whether outcome is related to the degree of underlying pulmonary hypoplasia,

as predicted by preoperative PaCO2, when correlated with an index of ventilation (VI = mean airway

pressure X respiratory rate) and confirmed by postmortem analysis of the lung. Those infants with PaCO2

greater than 40 mm Hg before surgery had a 77% mortality; when PaCO2 reduction could be achieved

only with VI greater than 1000, the mortality was still greater than 50%. After repair, however, the ability

to hyperventilate to PaCO2 less than 40 mm Hg proved to be an important determinant of survival; only

one of 31 infants in this group died, whereas only two of 27 infants with PaCO2 greater than 40 mm Hg

survived. In 16 infants with PaCO2 greater than 40 mm Hg despite hyperventilation, high-frequency

oscillatory ventilation was started. This resulted in a rapid fall in PaCO2, but 14 of the 16 infants had only

temporary improvement in oxygenation, and died. In five of the infants who died, alveolar number was

assessed by postmortem morphometric analysis; there was a severe reduction to less than 10% of

published normal neonatal values.

Pulmonary vascular changes of increased muscularization were less remarkable than those observed in

infants with persistent pulmonary hypertension. Our findings suggest that the degree of pulmonary

hypoplasia (which would not be influenced by surgical repair), rather than the pulmonary vascular

abnormality, mainly determines survival.

Consideration could therefore be given to an initial nonsurgical approach to congenital diaphragmatic

hernia, with the expectation that pulmonary function might improve and pulmonary vascular resistance

decrease.

Q 278. Von Brun's nest is seen in ?

A) Normal urothelium

B) Transitional cell carcinoma

C) Squamous cell carcinoma

D) Adeno carcinoma

Von Brunn’s nest refers to nests of transitional cells within the lamina propria of the urinary bladder that

arise from invagination of the overlying urothelium. It is the most common reactive proliferative change

within the urothelium and is common enough to be considered a normal feature of the bladder mucosa by

some investigators. Cystitis glandularis is a common change that occurs most often in the trigone. It is an

inflammatory process that arises within and merges imperceptibility with von Brunn’s nests. With cystic

dilatation of the glands, a progressive flattening of the lining cells occurs, and the lesion takes on the form

of cystitis cystica. 3, 4 Cystitis glandularis is microscopic or occurs as small mucosal nodules in most

occasions but it may occur as polypoid mass that suggests a neoplastic process macroscopically and

endoscopically.

Q 279. floating teeth is seen in{delhi pg 2008}

a. hyperparathyroidism

b. hypoparathyroidism

c. struge weber

d. vhl syndrome

Q280. ALL of the following are features of eisenmergers syndrome, except

1. triangular heart.

2. large main and central pulmonary aretry.

3. large peripheral vessels in the lung parenchyma.

4. ipulmonary arterial calcification

Page 116: DocumentQ1

Q281. mechanism of action of pertusis toxin

a. adp ribosylation

b. adenyl cyclase

c. open ca channel

d. dec affinity of g protein

Q282. which of the following skin lesion has an 'almost pathognomic ' assoc with hodgkins disease?

a. acanthosis nigricans

b. dermatomyositis

c. gen erythroderma

d. ichthyosis

Q283. which of the following condition least likely assoc with platelet disorder?

a. epistaxis

b. retroperitoneal bleed

c. ecchymoses

d. petechiae

Q284.: risk factors for local recurrence in extremity soft tissue sarcoma include all except::

a. High histologic grade

b. Deep location

c. Fibrosarcoma histopath

d. Previous recurrence

e. positive microscopic margins

Q285. Lemierre's disease MC cause ::

a. Staph aureus

b. Fusobacterium Necrophorum

c. H. Influenza

d. Beta hemolytic Streptococci

Lemierre's syndrome (or Lemierre's disease, also known as postanginal sepsis and human necrobacillosis)

is a form of thrombophlebitis usually caused by the bacterium Fusobacterium necrophorum, and

occasionally by other members of the genus Fusobacterium (F. nucleatum, F. mortiferum and F. varium

etc.) and usually affects young, healthy adults.

Lemierre's syndrome develops most often after a sore throat caused by some bacterium of the

Streptococcus genus, has created a peritonsillar abscess, a crater filled with pus and bacteria near the

tonsils. Deep in the abscess, anaerobic bacteria (microbes that do not require oxygen) like Fusobacterium

necrophorum can flourish. The bacteria penetrate from the abscess into the neighboring jugular vein in

the neck and there they cause an infected clot (thrombosis) to form, from which bacteria are seeded

throughout the body by the bloodstream (bacteremia). Pieces of the infected clot break off and travel to

the lungs as emboli blocking branches of the pulmonary artery bringing the heart's blood to the lungs.

This causes shortness of breath, chest pain and severe pneumonia. Fusobacteria are normal inhabitants of

the oropharyngeal flora

answer fusobacteria

Q286:: Low dose spiral ct is evolving as a screening tool for

a. ca lung

b. ca breast

c. ca prostate

d. ca oesophagus

Page 117: DocumentQ1

divakar. Guest

Q287:: Skeletal metastases are rare in

a .epithelial ovarian cancers and sarcomas

b. malanomas and rcc

c. lung cancers and breast cancer

d. GI cancers and urothelial tumors

Metastases involve bone by means of 3 main mechanisms:

(1) direct extension,

(2) retrograde venous flow, and

(3) seeding with tumor emboli via the blood circulation.

Seeding occurs initially in the red marrow; this process accounts for the predominant distribution of

metastatic lesions in the red marrow–containing areas in adults.

In contrast, bone metastases are usually widespread in children. Retrograde venous embolism is probably

the major mechanism when spread from intra-abdominal cancer involves the vertebrae. Increased intra-

abdominal pressure causes blood to be diverted from the systemic caval system to the valveless vertebral

venous plexus of Batson; this diversion allows the caudal and cranial flow of blood.

Metastases from certain primary sites (eg, renal cell or thyroid carcinomas) are almost always osteolytic,

whereas those from other sites (eg, prostatic carcinoma) are predominantly sclerotic Other malignancies

associated with sclerotic metastases include breast carcinoma, colonic carcinoma, melanoma, bladder

carcinoma, and soft-tissue sarcoma. The findings of sclerotic metastases virtually exclude an untreated

renal tumor or hepatocellular carcinoma

Q288:: Bitter taste is mediated by action of

a. guanyl cyclase

b. g protein

c. tyrosine kinase

d. epithelial sodium ion channels

Q289:: Pseudoexotropia is a feature of

a. high hypermetropia

b. high myopia

c. epicanthus

d. high astigmatism

Back to top

Posted: Sat Aug 21, 2010 1:12 pm Post subject:

Q288:: Bitter taste is mediated by action of

a. guanyl cyclase

b. g protein-answer

c. tyrosine kinase

d. epithelial sodium ion channels

Q290: Transferance-Counter transferance is important in

Page 118: DocumentQ1

a. hypnosis

b. psychoanalysis

c. counselling

d. behaviour therapy

Q291:: ROKITANSKY system of Postmortem :

a. organs are removed one by one

b. in situ -dissection is done

c. thoracic abnd cervical organs,abdominal organs and the urogenital system is removed in separate organ

blocks

d. thoracic abnd cervical organs,abdominal organs and the urogenital system is removed as one large

block

Rokitansky is associated with :

Superior Mesenteric Artery Syndrome

Mayer-Rokitansky-Küster-Hauser syndrome

Rokitansky's diverticulum

Rokitansky's triad (pulmonary stenosis)

Rokitansky-Aschoff sinuses (in the gallbladder)

Rokitansky-Cushing ulcer

Rokitansky-Maude Abbott syndrome

Von Rokitansky's syndrome

Rokitansky nodule - teratomas

Rokitansky's diverticulum

Associated persons:

Karl Freiherr von Rokitansky

Description:

A traction diverticulum resulting from shrinking of a bifurcator gland coalesced with the oesophagus.

Q.292 Generally atropinization+beta blockade cause HR of::

a. 110-100/min

b. 60-70/min

c. 160-150/min

d. 140-150/min

Q. 293:: Benda's Sign ( A dolichocephalic head with bulging Occiput) is diagnostic of::

a. Communication Hyrdocephalus

b. Otitic Hydrocephalus

c. Obs at Aqueduct site

d. Obs at cisterna magna

answer c

seen in dandy walker malfrm

Patients with Dandy-Walker malformation present with developmental delay, enlarged head circumference,

or signs and symptoms of hydrocephalus. The clinical presentation depends to some extent on the

particular combination of developmental anomalies in each infant

Macrocrania is usually the consequence of hydrocephalus but, in some patients, it results from massive

enlargement of the posterior fossa by the posterior fossa cyst.

In this situation, macrocrania precedes the development of hydrocephalus, giving the skull a characteristic

Page 119: DocumentQ1

dolichocephalic shape, with bulging of the occiput.

Dandy-Walker malformation as consisting of

(1) complete or partial agenesis of the vermis,

(2) cystic dilatation of the fourth ventricle, and

(3) an enlarged posterior fossa with upward displacement of lateral sinuses, tentorium, and torcular

herophili.

This triad is typically found in association with supratentorial hydrocephalus, which should be considered a

complication rather than part of the malformation complex.

Common findings of Dandy-Walker malformation include the following:

Enlarged posterior fossa

Varying degrees of cerebellar and vermian hypoplasia or complete vermian absence

Cyst formation in the posterior fossa

Vermian remnant everted above the posterior fossa cyst

Hypoplastic cerebellar hemispheres winged anterolaterally (outward) in front of the cyst

Absence of the foramina of Luschka and Magendie

Obstructive hydrocephalus secondary to cystic dilatation of the fourth ventricle

Abnormally high position of the straight sinus, torcular herophili, and tentorium

Sinus confluence and lateral sinuses elevated above the lambdoid sutures (high tentorial insertion, also

called lambdoid-torcular inversion;

Aqueductal obstruction, which may affect the need for supratentorial decompression (and is, therefore, an

important component)

If callosal agenesis coexists (20-25%), development of dilatation of the occipital horns )

Possible brainstem compression and hypoplasia; the degree of pontine hypoplasia is directly related to the

degree of cerebellar hypoplasia

Thinning and bulging of the occipital bones

Q. 294: Spinal Tap is for::

a. Spinal Fluid Pressure

b. Whether block exists

c. Types and number of cells

d. Protein and sugar levels

e. all

Q. 295: Csf Pressure in sitting ::

a. 4-6 mm hg

b. 7-10 mm hg

c 12-14 mm hg

Page 120: DocumentQ1

remi. Guest

d. 18-20 mm hg

e. 21-24 mm hg

Q. 296:: Foster s sign ::

a. spastic CP

b. hypotonic CP

c. Choreoathetotic CP

d. Myaesthenia gravis

Q. 297: Which Of the following BEST predicts the PROGNOSIS of patients with recent diagnosis of cutaneous melanoma and no cli

metastasis?

a. Breslow Thickness

b. Clark level

c. Ulceration

d. Gender

e. Celtic Complexion

Breslow's depth is one of the cornerstones of the current AJCC TNM staging of malignant melanoma. A large study validated the importance

of Breslow's depth as one of the three most important prognostic factors in melanoma (the others being T stage and ulceration). Breslow's

depth also accurately predicted the risk for lymph node metastasis, with deeper tumors being more likely to involve the nodes.

The above studies showed that depth was a continuous variable correlating with prognosis. However, for staging purposes, the most

AJCC guidelines use cutoffs of 1 mm, 2 mm, and 4 mm to divide patients into stages.

Q. 298:: Which bone tumors are radiographic diagnosis and do not require biopsy?

a. Chondrosarcoma

b. Metastasis

c. Giant cell tumor

d. Osteochondroma

Q. 299:: Which of the following concerning HTN induced Choroidopathy is false::

a. Elschnig spots characteristic

b. Exudative RD may develop

c. Hypertensive choroidopathy may be associated with chronic elevation in BP

d. Hypertensive choroidopathy may be associated with acute elevation in BP

Q. 300.OSHA GUIDLINESfor needle stick injury and other sharp injury requires all except.

1. record maintaining of all injuries

2. information and training.

3. pre-exposure prophylaxis.

4. post-exposure prophylaxis

Occupational Health and Safety Administration (OSHA) regulations and the Needlestick Safety and Prevention Act.

In 1992 the OSHA issued the Bloodborne Pathogen regulations (29 CFR 1910). In November of 2000, the Needlestick Safety and

Prevention Act was signed into law and took effect on April 18th, 2001. This new act required that OSHA revise the Bloodborne Pathogen

standard to add the following components:

1.Provide new examples in the definition of engineering controls.

2.Require that exposure control plans reflect how employers implement a needleless/safety and needle stick prevention program.

3.Requires the employer to solicit input from direct patient care employees in the identification, evaluation and selection of safer needle

Page 121: DocumentQ1

devices and work practices.

4.Require employers to establish and maintain a log of sharps related injuries.

answer choice 3 pre exp prophylaxis

OSHA Standards

This section highlights the OSHA standard requirements, preambles to final rules (background to final rules), directives (instructions for

compliance officers), and standard interpretations (official letters of interpretation of the standards) related to bloodborne pathogens and

needlestick prevention.

:For the most part, these States adopt standards that are identical to Federal OSHA. However, some States have adopted different

standards applicable to this topic or may have different enforcement policies.

Bloodborne pathogens.

Revisions to as a result of the Needlestick Safety and Prevention Act:

(i) requires the use of engineering and work practice controls to eliminate or minimize employee exposure to bloodborne pathogens.

Employers must keep a Sharps Injury Log for the recording of percutaneous injuries from contaminated sharps

The Exposure Control Plan shall:

Reflect changes in technology that eliminate or reduce exposure to bloodborne pathogens

Document annually consideration and implementation of appropriate commercially available and effective safer medical devices designed to

eliminate or minimize occupational exposure .

Solicit input from non-managerial employees responsible for direct patient care, who are potentially exposed to injuries from contaminated

sharps, in the identification, evaluation, and selection of effective engineering and work practice controls and shall document the solicitation

in the Exposure Control Plan

Appendix A, Hepatitis B vaccine declination (Mandatory).

Q. 301. PHOTOCOAGULATION IS done with all, except

1. argon.

2. krypton.

3. diode.

4. aluminium garnet

Q. 302. Ureteral duplication is commonly associated with

a) VUR.

b) PUJO.

c) Ureterocele.

d) Hypospadias.

Duplicated ureter is a congenital condition in which the ureteric bud, the embryological origin of the ureter, splits (or arises twice), resulting

in two ureters draining a single kidney. It is the most common renal abnormality, occurring in approximately 1% of the population.

additional ureter may result in a ureterocele, or an ectopic ureter.

Pathophysiology:

Ureteral development begins in the human fetus around the 4th week of embryonic development. A ureteric bud, arising from the

mesonephric (or Wolffian) duct, gives rise to the ureter, as well as other parts of the collective system. In the case of a duplicated ureter,

the ureteric bud either splits or arises twice. In most cases, the kidney is divided into two parts, an upper and lower lobe, with some

overlap due to intermingling of collecting tubules. However, in some cases the division is so complete as to give rise to two separate par

each with its own renal pelvis and ureter.

Classification:

Ureteral duplication is either:

Partial - i.e. the two ureters drain into the bladder via a single common ureter. Partial, or incomplete, ureteral duplication is rarely clinically

significant .

or

Complete - in which the two ureters drain separately. Complete ureteral duplication may result in one ureter opening normally into the

bladder, and the other being ectopic, ending in the vagina, the urethra or the vulval vestibule. These cases occur when the ureteric bud

arises twice (rather than splitting)

Page 122: DocumentQ1

Prevalence:

Duplicated ureter is the most common renal abnormality, occurring in approximately 1% of the population [2]. Race: Duplicated ureter is

more common in Caucasians than in African-Americans. Sex: Duplicated ureter is more common in females. However, this may be due to

the higher frequency of [urinary tract infection]s in females, leading to a higher rate of diagnosis of duplicated ureter.

Clinical Presentation

Prenatally diagnosed hydronephrosis suggest post-natal follow-up examination. The strongest neo-natal presentation is urinary tract

infection. A hydronephrotic kidney may present as a palpable abdominal mass in the newborn, and may suggest an ectopic ureter or

ureterocele. In older children, ureteral duplication may present as:

Urinary tract infection - most commonly due to vesicoureteral reflux (flow of urine from the bladder into the ureter, rather than vice

versa).

Urinary incontinence in females occurs in cases of ectopic ureter entering the vagina, urethra or vestibule.

Although, urterocoele is also seen in Duplicated ureter, but, since the most common presentation is UTI which itself is due to VUR, a

duplicated ureter is mostly associated with VUR.

Q. 303. Gonococcal stricture involves

a) Bulbar urethra.

b) Penile urethra.

c) Prostatic urethra.

d) External meatus.

trictures can be of any length from 5 mm to 10 cm. The commonest sites for gonococcal ones are: (1) the bulbous urethra, (2) at the

junction of the penis and scrotum, and (3) in the glans penis[md]in this order. Gonococcal strictures are the result of fibrosis in the corpus

spongiosum

Q304. Which vitamin in milk is extremely sensitive to light & requires storage of milk in a container that protects against photo

degradation

a. riboflavin

b. thiamin

c. niacin

d. vit.A

BACKGROUND & AIMS: Vitamins A and E are the most light-sensitive vitamins. Vitamin A is degraded by photolysis, while vitamin E

degrades by photo-oxidation. The composition of the parenteral nutrition mixture and the container could therefore influence degradation

during daylight administration. The aim of this study was therefore to determine the influence of fat emulsion and the type of bag on the

photo-degradation of vitamins A and E in Parenteral Nutrition (PN) mixtures during simulated infusion in daylight.

Q305. the following are associated with fibular hemimelia, except;

1. short tibia.

2. anteritr bowing of the leg.

3. equinovalgus deformity of the foot and ankle;

4. presence of polydactyly.

Q306. Pathological findings encountered in the frozen shoulder, commonly include all the following, except;

1. osteopenia of the underlying bone.

2. acute inflammatory chenges in the capsule.

3. fibrosis between the capsule and rotator cuff.

4. calcim deposits in the rotator cuff.

Q307. All are true about Testicular atrophy Except::

a. Leyding cell Hypoplasia

b. lymphocyte infiltrate

c. increased interstitial stroma

d. normal sertoli cell population

Q308. System for diabetic foot::

Page 123: DocumentQ1

a. Robson s Classification

b. Modified Wagener

c. Mullikens And Glowacki

d. Columbias.

Robsons Classification- it is for renal cell carcinoma---

Two staging symptoms commonly used are the Robson classification and the American Joint Committee on Cancer (AJCC) staging sy

The following table is taken from the Robson classification system. The prognosis generally depends on stage and there are four main

stages (I to IV):

Treatment

Localized Tumors - The standard management for stage I or II tumors and selected cases of stage III disease is radical nephrectomy.

Advanced Disease - Investigational therapy is the first-line treatment for metastatic disease as no immune approach or chemotherapy

agent has shown significant anti-tumor activity. The prognosis is highly variable. Surgery has a limited role for patients with metastatic

disease. One indication for nephrectomy is to alleviate pain or hemorrhage from a primary tumor.

Interferon alpha and interleukin-2 produce regressions in 10 to 20% of patients but these are rarely durable.

Wagner Classification of Diabetic Foot Ulcers

Grade 0: No ulcer in a high risk foot.

Grade 1: Superficial ulcer involving the full skin thickness but not underlying tissues.

Grade 2: Deep ulcer, penetrating down to ligaments and muscle, but no bone involvement or abscess formation.

Grade 3: Deep ulcer with cellulitis or abscess formation, often with osteomyelitis.

Grade 4: Localized gangrene.

Grade 5: Extensive gangrene involving the whole foot.

University of Texas Wound Classification System of Diabetic Foot Ulcers

Grade I-A: non-infected, non-ischemic superficial ulceration

Grade I-B: infected, non-ischemic superficial ulceration

Grade I-C: ischemic, non-infected superficial ulceration

Grade I-D: ischemic and infected superficial ulceration

Grade II-A: non-infected, non-ischemic ulcer that penetrates to capsule or bone

Grade II-B: infected, non-ischemic ulcer that penetrates to capsule or bone

Grade II-C: ischemic, non-infected ulcer that penetrates to capsule or bone

Grade II-D: ischemic and infected ulcer that penetrates to capsule or bone

Grade III-A: non-infected, non-ischemic ulcer that penetrates to bone or a deep abscess

Grade III-B: infected, non-ischemic ulcer that penetrates to bone or a deep abscess

Grade III-C: ischemic, non-infected ulcer that penetrates to bone or a deep abscess

Grade III-D: ischemic and infected ulcer that penetrates to bone or a deep abscess

Page 124: DocumentQ1

In 1982, Mulliken and Glowacki described a clinically relevant classification of vascular anomalies in which these lesions were categorized

according to their endothelial cell characteristics. (1) This classification, which was adopted by the International Society for the Study of

Vascular Anomalies in 1996, differentiates proliferating tumors (most of which are hemangiomas) from vascular malformations, which are

structural anomalies involving capillaries, venules, veins, lymphatic channels, and combinations of these structures. ---

* Tumors

Juvenile hemangioma

Rapidly involuting congenital hemangioma

Noninvoluting congenital hemangioma

Kaposiform hemangioendothelioma

Tufted angioma

* Vascular malformations

High-flow

Arteriovenous malformation

Low-flow

Venous malformation

Lymphatic malformation

Lymphatic-venous malformation

Capillary (or venular) malformation

Columbia Classification is a Algorithm of Suicide Assessment (C-CASA)

Q309. Which does not stabilize new formed vessels::

a. VEGF

b. PDGF

c. Ang1

d. TGF-beta

Q310. Edavarone is used in Rx::

a. Cva

b. Asthma

c. Hiv

d. Diabetes

Edaravone (3-methyl-1-phenyl-2-pyrazolin-5-one), a strong novel free radical scavenger, is used for treatment of patients with acute brain

infarction. Edaravone has preventive effects on myocardial injury following ischemia and reperfusion in patients with acute myocardial

infarction.

Antioxidant actions of edaravone include::

enhancement of prostacyclin production,

inhibition of lipoxygenase metabolism of arachidonic acid by trapping hydroxyl radicals,

inhibition of alloxan-induced lipid peroxidation,

quenching of active oxygen, leading to protection of various cells, such as endothelial cells, against damage by reactive oxygen species

(ROS).

Q311. Lawrence Moon syn consists A/e::

a. RP

b. AR

c. Poast axial Polydactly

d. Hypogonadism

e. Spastic paraplegia

Page 125: DocumentQ1

Posted: Tue Aug 24, 2010 12:13 pm Post subject:

Q. 306. Pathological findings encountered in the frozen shoulder, commonly include all the following, except;

1. osteopenia of the underlying bone.

2. acute inflammatory chenges in the capsule.

3. fibrosis between the capsule and rotator cuff....hallmark

4. calcim deposits in the rotator cuff....ans

Adhesive capsulitis is characterized by pain and restricted motion of the shoulder. Usually this occurs in the absence of intrinsic shoulder

disease, including osteoarthritis and avascular necrosis.

It is, however, more common in patients who have had bursitis or tendinitis previously as well as patients with other systemic illnesses,

such as chronic pulmonary disease, ischemic heart disease, and diabetes mellitus. The etiology isnot clear, but adhesive capsulitis

appears to develop in the setting of prolonged immobility. Reflex sympathetic dystrophy may also occur in the setting of adhesive

capsulitis. Clinically, this disorder is more commonly seen in females over age 50. Pain and stiffness develop over the course of months

to years. On physical examination, the affected joint is tender to palpation, with a restricted range of motion.

The gold standard for diagnosis is arthrography with limitation of the amount of injectable contrast to less than 15 mL. In most patients,

adhesive capsulitis will regress spontaneously within 1 to 3 years. NSAIDs, glucocorticoid injections, physical therapy, and early

mobilization of the arm are useful therapies.

Q. 312:: Aldactone bodies seen in::

a. zona glomerulosa

b. zona reticulosa

c. zona fasiculata

d. adrenal medulla

Q. 313:: RCC worst prognosis::

a. vascular invasion

b. hypercalcemia

c. Hematuria

d. > 5 cm size

Temperature is the first sensation lost, followed by light touch, pain and then deep pressure. Sensory loss usually begins in the

extremities, toes and fingertips

Q. 314: x-ray appearance of HMD::

a. homogenous ground glass

b. Reticulonodular

c. normal

d. air bronchogram

Q. 315:: Moro Disappears at::

a. 3 months

b. 9 months

c. 7 months

d. 6 months

Posted: Tue Aug 24, 2010 1:43 pm Post subject:

Q. 311:: Aldactone bodies seen in::

Page 126: DocumentQ1

a. zona glomerulosa

b. zona reticulosa

c. zona fasiculata

d. adrenal medulla

aldactone bodies a ka spirnolactone bodies

Spironolactone (aldactone) bodies: concentric lamellar formations in the adrenal cortices of patients treated with spironolactone.

Numerous spironolactone bodies have been detected in the zona glomerulosa cells of the adrenal cortex of a 36-year-old spironolactone

treated woman whose non-tumorous right adrenal gland was removed surgically because of primary hyperaldosteronism.

Electron microscopy revealed spherical laminated whorls which consisted of a central core composed of an amorphous electron-dense

material surrounded by numerous smooth-walled concentric membranes. Continuous with and deriving from the endoplasmic reticulum,

they were present in viable cells and were not associated with ultrastructural features indicating cellular injury. Cytoplasmic inclusions

similar to spironolactone bodies can be detected in other organs after the administration of various compounds. Thus, they can be

regarded as neither specific to spironolactone treatment nor exclusively inducible in the zona glomerulosa of the adrenal cortex.

Spironolactone bodies were observed in an adrenal cortical adenoma that was removed from a patient with primary hyperaldosteronism

(Conn syndrome) treated preoperatively with spironolactone. The electron microscopical evaluation of this adrenal cortical adenoma

shows origin of spironolactone bodies from whorls of endoplasmic reticulum in cells with the cytoplasmic features of those from the zona

glomerulosa of the adrenal gland. There was no evidence that the bodies were derived from mitochondria, which confirms recent

ultrastructural findings in patients treated with spironolactone. These bodies have been described in the adrenal cortex only in patients

who have received spironolactone, and the pharmacologic specificity of the bodies strongly suggests a direct mode of action by

spironolactone on aldosterone production by cells of the adrenal zona glomerulosa.

answer zona glomerulosa

Q. 314: x-ray appearance of HMD::

a. homogenous ground glass

b. Reticulonodular

c. normal

d. air bronchogram

On x-ray, the lungs may have a characteristic, but not pathognomonic appearance that includes a fine reticular granularity of the

parenchyma and air bronchograms, which are often more prominent early in the left lower lobe because of superimposition of th

shadow ( Fig. 101-4 ). The initial roentgenogram is occasionally normal, with the typical pattern developing at 6–12 hr. Considerable

variation in films may be seen, depending on the phase of respiration and the use of CPAP or positive end-expiratory pressure (PEEP);

this variation often results in poor correlation between roentgenograms and the clinical course.

Q. 315:: Moro Disappears at::

a. 3 months

b. 9 months

c. 7 months

d. 6 months (ans)

Moro's till 4-5 months is considered normal. persistence beyond is a problem. 6 months is the best possible answer here.

Q 316:: The following is of serious pathological significance in infants::

a. Loss of weight

b. palpable left kidney

c. palpable spleen

d. deviation of trachea from midline

Page 127: DocumentQ1

Q. 317:: Sneezing in a neonate ::

a. nasal allergy

b. syphilis

c. normal

d. Nasopharyngitis

Q. 317:: Sneezing in a neonate ::

a. nasal allergy

b. syphilis

c. normal

d. Nasopharyngitis

Posted: Wed Aug 25, 2010 12:52 pm Post subject:

Q312. RCC worst prognosis::

a. vascular invasion

b. hypercalcemia

c. Hematuria

d. > 5 cm size

its chance to combine our knowledge abt 1 of most imp topics rcc

lil bit of copy paste from standard books for last min revisons ..

Renal cell carcinoma accounts for approximately 3% of adult malignancies and 90-95% of neoplasms arising from the kidney.

While the optimal treatment strategy continues to evolve, three agents that target angiogenesis (sunitinib, bevacizumab, and pazopanib)

and an mTOR-targeted therapy (temsirolimus) have been approved as front-line agents. These have largely replaced cytokines

(immunotherapy) in treatment-naive patients

The tissue of origin for renal cell carcinoma is the proximal renal tubular epithelium. Renal cancer occurs in both a sporadic

(nonhereditary) and a hereditary form, and both forms are associated with structural alterations of the short arm of chromosome 3 (3p).

Genetic studies of the families at high risk for developing renal cancer led to the cloning of genes whose alteration results in tumor

formation. These genes are either tumor suppressors (VHL, TSC) or oncogenes (MET)

syndromes associated with renal cell carcinoma are recognized:

(1) von Hippel-Lindau (VHL) syndrome,

(2) hereditary papillary renal carcinoma (HPRC),

(3) familial renal oncocytoma (FRO) associated with Birt-Hogg-Dube syndrome (BHDS), and

(4) hereditary renal carcinoma (HRC)

familial renal oncocytoma can develop bilateral, multifocal oncocytoma or oncocytic neoplasms in the kidney. Birt-Hogg-Dube syndrome

is a hereditary cutaneous syndrome. Patients with Birt-Hogg-Dube syndrome have a dominantly inherited predisposition to develop

benign tumors of the hair follicle (ie, fibrofolliculomas), predominantly on the face, neck, and upper trunk, and are at risk of developing

renal tumors, colonic polyps or tumors, and pulmonary cysts.

median age at diagnosis was 64 years of age

Renal cell carcinoma has a male-to-female preponderance of 1.6:1.

Page 128: DocumentQ1

classic triad of flank pain, hematuria, and flank mass is uncommon (10%) and is indicative of advanced disease

Most common presentations

Hematuria (40%)

Flank pain (40%)

Palpable mass in the flank or abdomen (25%)

Cytokine release by tumor (eg, IL-6, erythropoietin, nitric oxide) causes these paraneoplastic conditions.

risk factors

1.Cigarette smoking doubles the risk of renal cell carcinoma and contributes to as many as one third of all cases. The risk appears to

increase with the amount of cigarette smoking in a dose-dependent fashion.

2.Obesity is another risk factor, particularly in women; increasing body weight has a linear relationship with increasing risk.

3.Hypertension may be associated with an increased incidence of renal cell carcinoma.

4.Phenacetin-containing analgesia taken in large amounts may be associated with increased incidence of renal cell carcinoma.

5.There is an increased incidence of acquired cystic disease of the kidney in patients undergoing long-term renal dialysis; this

predisposes to renal cell cancer.

6.Tuberous sclerosis

7.Renal transplantation: Acquired renal cystic disease of the native kidney also predisposes to renal cell cancer in renal transplant

recipients.

8.VHL disease: This inherited disease is associated with renal cell carcino

The most common locations of metastasis are the lungs (more than 50%) and bones (33%), followed in frequency by the regional lymp

nodes, liver, adrenal, and brain.

The average 5-year survival rate of persons with renal cell carcinoma is about 45% and as high as 70% in the absence of distant

metastases. With renal vein invasion or extension into the perinephric fat, the figure is reduced to approximately 15% to 20%.

Nephrectomy has been the treatment of choice, but partial nephrectomy to preserve renal function is being done with increasing

frequency and similar outcome.

###

The Robson modification of the Flocks and Kadesky system is uncomplicated and is used commonly in clinical practice. This system was

designed to correlate stage at presentation with prognosis. The Robson staging system is as follows:

Stage I - Tumor confined within capsule of kidney

Stage II - Tumor invading perinephric fat but still contained within the Gerota fascia

Stage III - Tumor invading the renal vein or inferior vena cava (A), or regional lymph-node involvement (B), or both (C)

Stage IV - Tumor invading adjacent viscera (excluding ipsilateral adrenal) or distant metastases

treatment:::

Sorafenib (Nexavar), a small-molecule Raf kinase and vascular endothelial growth factor (VEGF) multireceptor kinase inhibitor, is

approved by the U.S. Food and Drug Administration for the treatment of patients with advanced renal cell carcinoma.

toxicities of sorafenib::

reversible skin rashes in 40% and hand-foot skin reaction in 30%. Diarrhea was reported in 43%, treatment-emergent hypertension in

17%, and sensory neuropathic changes in 13%. Alopecia, oral mucositis, and hemorrhage also were reported more commonly on the

sorafenib arm. The incidence of treatment-emergent cardiac ischemia/infarction events was higher in the sorafenib group (2.9%)

compared with the placebo group (0.4%).

Sunitinib is another multikinase inhibitor approved by the FDA for the treatment of metastatic kidney cancer that has progressed after a

trial of immunotherapy.

toxicities::Major toxicities (grade II or higher) include fatigue (38%), diarrhea (24%), nausea (19%), dyspepsia (16%), stomatitis

(19%), and decline in cardiac ejection fraction (11%). Dermatitis occurred in 8%, and hypertension occurred in 5% of patients.

Page 129: DocumentQ1

Temsirolimus inhibits mTOR (mammalian target of rapamycin), which is a serine/threonine kinase important in the regulation of

growth and division

toxicities::::Common toxicities of temsirolimus include asthenia, rash, anemia, hypophosphatemia, and hyperlipidemia.

Temsirolimus has FDA approval for the treatment of advanced renal cell carcinoma at a dose of 25 mg weekly IV until progression.

Everolimus (Afinitor) is a serine-threonine kinase inhibitor of mTOR, an important regulatory protein in cell signaling. Everolimus was

approved by the US Food and Drug Administration in March 2009 for advanced renal cell carcinoma after failure of treatment with

sunitinib or sorafenib.

ct:::

1.intravenous gemcitabine (600 mg/m2 on days 1, 8, and 15) with continuous infusion fluorouracil (150 mg/m2/d for 21 d in 28

in patients with metastatic renal cell cancer produced a partial response rate of 17%

2.Floxuridine (5-fluoro 2'-deoxyuridine [FUDR]), 5-fluorouracil (5-FU), and vinblastine, paclitaxel (Taxol), carboplatin, ifosfamide,

gemcitabine, and anthracycline (doxorubicin) all have been used. Floxuridine infusion has a mean response rate of 12%,

3.Renal cell carcinoma is refractory to most chemotherapeutic agents because of multidrug resistance mediated by p -glycoprotein.

Normal renal proximal tubules and renal cell carcinoma both express high levels of p -glycoprotein. Calcium channel blockers or other

drugs that interfere with the function of p -glycoprotein can diminish resistance to vinblastine and anthracycline in human renal cell

carcinoma cell lines.

surgery:::

Radical nephrectomy, which remains the most commonly performed standard surgical procedure today for treatment of localized r

carcinoma, involves complete removal of the Gerota fascia and its contents, including a resection of kidney, perirenal fat, and ipsilateral

adrenal gland, with or without ipsilateral lymph node dissection

In patients with renal vein involvement without metastases, radical nephrectomy is performed with early ligation of the renal artery but

no manipulation of the renal vein. If the inferior vena cava is involved, then vascular control of the inferior vena cava is obtained both

above and below the tumor thrombus, and the thrombus is resected intact, with subsequent closure of the vena cava. Patients with

actual invasion of the inferior vena caval wall have poor prognoses, despite aggressive surgical approaches.

(1) the transperitoneal approach,

(2) the flank approach, and

(3) the thoracoabdominal approach.

The thoracoabdominal approach offers the advantage of palpation of the ipsilateral lung cavity and mediastinum, as well as the ability to

resect solitary pulmonary metastases.

2.Laparoscopic nephrectomy is a less invasive procedure, incurs less morbidity, and is associated with shorter recovery time and less

blood loss.

3.Palliative nephrectomy should be considered in patients with metastatic disease for alleviation of symptoms such as pain, hemorrhage,

malaise, hypercalcemia, erythrocytosis, or hypertension.

4.Renal artery embolization with ethanol and gelatin sponge pledgets has been found effective for palliative treatment in patients who

are not candidates for surgery, or who refuse surgery.

rt:::::

Radiation therapy may be considered as the primary therapy for palliation in patients whose clinical condition precludes surgery, either

because of extensive disease or poor overall condition.

A dose of 4500 centigray (cGy) is delivered, with consideration of a boost up to 5500 cGy.

Page 130: DocumentQ1

ref : emedicine

robbins 8th ed

harisson 17th

is answer vascular invasion

couldnt find prognostic significnce of hypercalcemia in terms of percentages{High "corrected" serum calcium (>10 mg/dL) is a one of bad

prognostic indicators though}

tumor 5cm :: comes in t1 stage which is wth good prognosis...

Primary tumor (T)

TX - Primary tumor cannot be assessed

T0 - No evidence of primary tumor

T1 - Tumor 7 cm or smaller in greatest dimension, limited to the kidney

T2 - Tumor larger than 7 cm in greatest dimension, limited to the kidney

T3 - Tumor extends into major veins or invades adrenal gland or perinephric tissues but not beyond the Gerota fascia

T3a - Tumor invades adrenal gland or perinephric tissues but not beyond the Gerota fascia

T3b - Tumor grossly extends into the renal vein(s) or vena cava below the diaphragm

T3c - Tumor grossly extends into the renal vein(s) or vena cava above the diaphragm

T4 - Tumor invading beyond the Gerota fascia

Laterality does not affect the N classification

NX - Regional lymph nodes cannot be assessed

N0 - No regional lymph node metastasis

N1 - Metastasis in a single regional lymph node

N2 - Metastasis in more than 1 regional lymph nod

Posted: Wed Aug 25, 2010 1:02 pm Post subject:

Q318. Erythroblastosis fetalis not involved::

a. Antigen Cw

b. Antigen Cx

c. Antigen Diego

d. Anti Lewis

answer anti lewis

Other fetomaternal incompatibilities that can cause erythroblastosis fetalis involve the Kell, Duffy, Kidd, MNSs, Lutheran, Diego, Xg, P,

Ee, and Cc antigen systems, as well as other antigens. Incompatibilities of ABO blood types do not cause erythroblastosis fetalis.

Q319. CYSTATIN C is elevated in::

a. CVD

b. Alzheimer s

c. chronic kidney disease

d. liver diseases

e. brain infraction

ans-c)chronic renal disease,a)cardiovascular disease,b)alzheimers, e)brain infarction

ref-Robbins 8th,wikepedia

Page 131: DocumentQ1

CYSTATIN C-a protein encoded by the CST3 gene, is mainly used as a biomarker of kidney function. Recently, it has been studied for its

role in predicting new-onset or deteriorating cardiovascular disease. It also seems to play a role in brain disorders involving amyloid (a

specific type of protein deposition), such as Alzheimer's disease

.

Cystatin C levels are decreased in atherosclerotic (so-called 'hardening' of the arteries) and aneurysmal (saccular bulging) lesions of the

aorta.

Since cystatin 3 also binds amyloid β and reduces its aggregation and deposition, it is a potential target in Alzheimer's disease. Althou

not all studies have confirmed this, the overall evidence is in favor of are role for CST3 as a susceptibility gene for Alzheimer's disease.

Cystatin C levels have been reported to be higher in subjects with Alzheimer's disease

Mutations in the cystatin 3 gene are responsible for the Icelandic type of hereditary cerebral amyloid angiopathy, a condition

predisposing to intracerebral haemorrhage, stroke and dementia

A few studies have looked at the role of cystatin C or the CST3 gene in age-related macular degeneration. Cystatin C has also been

investigated as a prognostic marker in several forms of cancer. Its role in pre-eclampsia remains to be confirmed.

For women, the average reference interval is 0.52 to 0.90 mg/L with a mean of 0.71 mg/L. For men, the average reference inter

0.56 to 0.[snip] mg/L with a mean of 0.77 mg/L. The normal values decrease until the first year of life, remaining relatively stable before

they increase again, especially beyond age 50

Diazyme’s Cystatin C assay is a convenient cost effective dual liquid stable PEIA method for this critically important emerging marker in

the early detection and diagnosis of renal disease.

Cystatin C in the sample binds to the specific anti-Cystatin C antibody, which is coated on latex particles, and causes agglutination. The

degree of the turbidity caused by agglutination is measured optically and is proportional to the amount of Cystatin C in the sample

Intended use

The assay is for the quantitative in vitro determination of Cystatin C in human serum or plasma. There is a growing body of evidence

suggesting that Cystatin C can be used to detect kidney disease at earlier stages, which may help facilitate prevention efforts in the

elderly and those with diabetes, hypertension, or cardiovascular disease.

Cystatin C, a serum measure of renal function, is a stronger predictor of the risk of death and cardiovascular events in elderly persons

than is creatinine.

Cystatin C or cystatin 3 (formerly gamma trace, post-gamma-globulin or neuroendocrine basic polypeptide),[1] a protein encoded by the

CST3 gene, is mainly used as a biomarker of kidney function. Recently, it has been studied for its role in predicting new-onset or

deteriorating cardiovascular disease. It also seems to play a role in brain disorders involving amyloid (a specific type of protein

deposition), such as Alzheimer's disease.

Q320. The average BP in a 1 year old child is::

a. 120/80

b. 120/90

c. 110/70

d. 95/55

e. 60/30

Normal blood pressure in children less than 1 year is usually 70/30 or greater. It can be higher or lower, depending on activity and

circumstances.

When calculating a child's (greater than 1 year) normal blood pressure consider this formula: 90 + 2(age in years) is normal; 70 + 2(age

in years) is the lower end. For example: A 7-year-old child's systolic (top number) pressure would be between 70 + 2(7) =84 and 90 +

2(7) =104. The bottom number (diastolic) should be 30-45 less than the systolic number. So the healthy 7-year-old child's pressure

should be from 84/39 to 104/59.

Page 132: DocumentQ1

normal blood pressure in adults is less than 120/80. 120-139/80-89 is indicative of pre-hypertension; 140-159/90-99 is Stage I

hypertension; greater than 160/100 is Stage II hypertension.

. A newborn has a normal blood pressure of about 70 over 42, but when the infant reaches 1 month of age, the normal blood pre

rises to approximately 85 over 45. At 6 months of age normal blood pressure is 93 over 55, rising again at 1 year to a normal average of

94 over 56.

answer d

Q321. Cranipharyngioma contains?

(1) HCG

(2) AFP

(3) BETA - GLOBULIN

(4) CA 19.9

Q322. Picibanil all are true except::

a. ok-432

b. better suited for monocystic lesions

c. for unilocular lesions

d. useful for keloids

A lyophilized preparation of a low-virulence strain (SU) of Streptococcus pyogenes (S. hemolyticus), inactivated by heating with penicillin

G. It has been proposed as a noncytotoxic antineoplastic agent because of its immune system-stimulating activity

Intralesional injection of OK-432 (lyophilized incubation mixture of group A Streptococcus pyogenes of human origin) is safe and effective

therapy for lymphangioma. The authors evaluated the mechanism of this therapy in 6 patients who had cystic lymphangioma. The

intracystic fluid of the cystic lymphangioma was aspirated before and after (on days 1 and 4) the OK-432 therapy. Changes in cell

populations and cytokine productions in each aspirated fluid were analyzed. White blood cells in the intracystic fluid increased markedly

in number. Before OK-432 therapy, 96% of the intracystic white blood cells were lymphocytes, and the remaining were neutrophils and

macrophages.

predominantly macrocystic lymphangiomas had a successful outcome. CONCLUSIONS: OK-432 should be efficacious in the treatment of

lymphangiomas. Our study design is well structured to clearly define the role of this treatment agent.

sclerotherapy using OK-432 is an effective and safe treatment modality for BCC branchial cleft cyst, especially for unilocular cysts.

Sclerosing of unilocular BCC with OK-432 should therefore be considered before surgical excision.

Q323. a child is brought to u with history of swallowing a coin, your next step is::

a. fibreoptic endoscopy

b. rigid endoscopy

c. laproscopy

d. bacche key sath chor police khelo

In most cases,of a coin in the lower esophagus,intially x ray is done. the child may be watched closely over a day or two to see if the

object passes on its own. If the child is in pain or has vomiting, this approach is not recommended. since option of xray n observation is

not given hence we assume tat child is symptomatic

Endoscopy is the definitive way to evaluate, diagnose and remove a foreign body in the stomach or esophagus. The child is placed under

anesthesia or sedation while a surgeon uses an endoscope. . A rigid endoscope is better for protecting the esophagus from damage as

the object is removed but can only be advanced to the esophagus. A flexible endoscope can reach as far as the first part of the small

intestines. A small clamp or net can be used to retrieve the object.

Q324. a 25 yr old man had DM-1 for 10 yrs. now mild proteinuria. as per JNC-7 his target Bp::

Page 133: DocumentQ1

a. <125/<75

b. <130/<80

c. <140/<90

d. <140/<80

ANS-b)<130/<80 ref. cmdt 2010,harrison 17th

Hypertension develops with progressive kidney involvement, and coronary and cerebral atherosclerosis seems to be accelerated.

Approximately two-thirds of adult patients with diabetes have hypertension. Once diabetic nephropathy has progressed to the stage of

hypertension, proteinuria, or early chronic kidney disease, glycemic control is not beneficial in influencing its course. In this

circumstance, antihypertensive medications, including ACE inhibitors, and restriction of dietary protein to 0.8 g/kg body weight per day

are recommended. ACE inhibitors have been shown to protect against deterioration in kidney function in type 1 diabetic patients with

clinical nephropathy. This beneficial effect appears to be due to improved glomerular hemodynamics that cannot be explained only by the

antihypertensive action of these drugs. Captopril (25 mg three times daily) has shown a 50% reduction in the risk of the combined end

points of death, dialysis, and transplantation in type 1 subjects with diabetic nephropathy and clinical proteinuria. During initiation of ACE

inhibitor therapy, an increment in serum creatinine greater than 2 mg/dL due to a rapid fall in intraglomerular pressure—or the

occurrence of persistent hyperkalemia (above 6 mEq/L) due to hyporeninemic hypoaldosteronism—is an indication to stop this

medication.

Q. 325) D and L enantiomers occurs in which drug?

a) amoxicillin

b) diazepam

c) verapamil

d) losartan

answer verapamil{ref :goodman pg590}

Verapamil (CALAN, ISOPTIN, VERELAN, COVERA-HS) is prescribed as a racemate. L-Verapamil is a more potent calcium channel blocker

than is D-verapamil. However, with oral therapy, the L-enantiomer undergoes more extensive first-pass hepatic metabolism. For this

reason, a given concentration of verapamil prolongs the PR interval to a greater extent when administered intravenously (where

concentrations of the L- and D-enantiomers are equivalent) than when administered orally

Q326. polyhydramnios occurs in all except?

a) Diabetes

b) multiple pregnancy

c) renal agenesis

d) anencephaly

answer c :: it results oligohydramnios

Q327. commonest site of thyroglossal cyst is?

a) supra hyoid

b) sub hyoid

c) infra thyroid

d) foramen caecum

Q328. LEMON ON MATCH STICK contour is seen in

a) diabetes mellitus

b) cushings syndrome

c) myxoedema

d) acromegaly

answer :b:: central obesity manifests as this appearance

Q329. DURCK granuloma is seen in ?

Page 134: DocumentQ1

a) tuberculosis

b) malaria

c) hiv

d) sarcoidosis

Q330. Pancytopenia Gap for allogenic BMT::

a. 10-14 days

b. 6-10 days

c. 3-4 days

d. 2-3 days

Posted: Wed Aug 25, 2010 1:03 pm Post subject:

Q331. Machine Gun sneezing::

a. Pandas

b. Vein of galen malformation

c. Rhinoscleroma

d. Psychiatric

chronic “machine gun” sneezing, which means the individual started to sneeze and continuously sneezes for weeks on end, there really

aren’t complications associated with sneezing. Long term chronic, “machine gun” sneezing is very rare, and can complicate daily living by

interfering with the individual’s ability to function normally. This type of sneezing can become very painful and can interfere with the

individual’s ability to sleep, eat, or even speak.

Per an allergist interviewed for this story, the condition is extremely rare and "can be triggered by hundreds of causes, including

allergies, sinus problems, or growths in the nasal passage.

his girl now has a diagnosis called PANDAS (Pediatric Autoimmune Neuropsychiatric Disorder Associated with Streptococcus)

Q332. Fiduciary markers::

a. gold

b. silver

c. Aluminium

d. copper

Gold fiducial markers are tiny gold markers that are 2.0 to 3.0 mm in size. That's about 0.11 inch in size. The markers are inserted u

the same method a prostate biopsy is performed. A transrectal ultrasound is performed in the urologist's office. The prostate is

measured. Depending on the patient, the prostate can be anesthetized with 1.0 % lidocaine. In general, three gold fiducial markers are

then inserted through a needle which is passed through the ultrasound probe. The ultrasound provides the urologist with the ability to

precisely place the markers in strategic positions.

fiduciary marker or fiducial is an object used in the field of view of an imaging system which appears in the image produced, for use as a

point of reference or a measure. It may be either something placed into or on the imaging subject, or a mark or set of marks in the

reticle of an optical instrument

answer ::gold:: used in imaging of prostate

Q333. Cyberknife is::

a. frameless system

b. shameless system

c. with frame

d. with rotating gantry

Page 135: DocumentQ1

answer a

CyberKnife is a frameless robotic radiosurgery system invented by John R. Adler, a Stanford University Professor of Neurosurgery and

Radiation Oncology, and Peter and Russell Schonberg of Schonberg Research Corporation.

The two main elements of the CyberKnife are

(1) the radiation produced from a small linear particle accelerator and

(2) a robotic arm which allows the energy to be directed at any part of the body from any direction.

The original (and still utilized) method is called 6D or skull based tracking. The X-ray camera images are compared to a library of

computer generated images of the patient anatomy. Digitally Reconstructed Radiographs (or DRR's) and a computer algorithm

determines what motion corrections have to be given to the robot because of patient movement. This imaging system allows the

CyberKnife to deliver radiation with an accuracy of 0.5mm without using mechanical clamps attached to the patient's skull.[8] The use of

the image guided technique is referred to as frameless stereotactic radiosurgery. This method is referred to as 6D because corrections

are made for the 3 translational motions (X,Y and Z) and three rotational motions

Small metal markers (fiducials) made out of gold for bio-compatibility and high density to give good contrast on X-ray images are

surgically implanted in the patient. This is carried out by an interventional radiologist, or neurosurgeon. The placement of the fiducials is

a critical step if the fiducial tracking is to be used. If the fiducials are too far from the location of the tumor, or are not sufficiently spread

out from each other it will not be possible to accurately deliver the radiation.

One of the most widely known stereotactic radiosurgery systems is the Gamma Knife. The Gamma Knife was originally developed by Lars

Leksell, remains the gold standard method for delivery of stereotactic radiosurgery to the brain and is manufactured by Elekta. John

Adler, the inventor of the CyberKnife system spent time training with Lars Leksell in Stockholm at the Karolinska Institute in 1985. The

GammaKnife system uses 201 Cobalt-60 sources located in a ring around a central treatment point ("isocenter"). The Gamma Knife

system is equipped with a series of 4 collimators of 4mm, 8mm, 12mm and 16mm diameter, and is capable of submillimeter accuracies.

The Gamma Knife system does however require a head frame to be bolted onto the skull of the patient, and is only capable of treating

cranial lesions. As a result of frame placement, treatment with Gamma Knife does not require real time imaging capability as the frame

does not allow movement during treatment. This is the reason that the Gamma Knife system is likely to be more accurate than C

Knife. The Cyberknife Society and Accuray maintain that there are no peer-reviewed published papers that establish Gamma Knife as

being more accurate than CyberKnife.

Q334. ALL are DATE RAPE drugs except::

a. Chloral hydrate

b. Flunitrazepam

c. Gamma hydroxybutyrate

d. insulin

e. Mickey finn

Date rape drug refers to a drug that can be used to assist in the commission of a sexual assault, such as date rape. Drugs used

facilitate rape may have sedative, hypnotic, dissociative, and/or amnesiac effects, and can be added to a food or drink without the

victim's knowledge.

The act of adding such substances to drinks is known as "drink spiking". The reasons for drink spiking range from personal amusement

or maliciousness to theft or (sexual) assault

three most commonly used drugs for date rape are alcohol and two prescription-strength sleep aids. The two prescription drugs are GHB,

also known as gamma-hydroxybutyric acid, and benzodiazepines (such as flunitrazepam, also known as Rohypnol or "roofies"); however,

an American 1997 study showed that alcohol still remains the drug most frequently implicated in substance-assisted sexual assault

Mickey Finn (or simply Mickey) is a slang term for a drink laced with a drug (especially chloral hydrate) given to someone without their

knowledge in order to incapacitate them (see drink spiking). Serving someone a Mickey Finn is most commonly referred to as slipping a

mickey, sometimes spelled "slipping a mickie".

Page 136: DocumentQ1

answer insulin

Q 335)Hutchinson-Gilford progeria syndrome (HGPS), false statement is?

(1) Mutation in LMNA gene

(2) Profound growth failure from 1'st yr of life

(3) Death is mostly due to renal failure

(4) Mutation is DOMINANT NEGATIVE MUTATION

Q 336) Not a disease of affluent societies?

(1) Obesity

(2) Diverticular diseases

(3) Dental caries

(4) Diarrhea

(5) Constipation

Posted: Thu Aug 26, 2010 1:30 pm Post subject:

Q337. Which is most common lid ca?

Q338. Which most common after chalazion?

A vaccine vial monitor, or VVM, is a circular indicator, printed directly on the vaccine vial label or affixed to the top of the vial or

ampoule. The inner square of the VVM is made of heat-sensitive material that is initially light in colour and becomes darker when

exposed to heat over time. By comparing the colour of the square to the reference ring, health workers can determine the extent to

which the vaccine has been exposed to heat. The vaccine can be used as long as the colour of the inner square is lighter than that of the

reference ring.

Although developed as a heat-exposure indicator, VVM also contributes significantly to the reduction of vaccine freezing. VVM makes it

possible to detect and avoid excessive heat exposure to vaccines when methods are employed to store and transport vaccines without ice

and equipment that is a known source of freeze damage

Vaccine vial monitors were introduced in 1996, and it has been 10 years of successful implementation. Today, all vaccines (with a couple

of exceptions) come with VVMs through UN procuremenmt agencies

In order to mark the 10 years of successful implementation of VVMs and acknowledge the efforts put into this device by individuals,

organizations, institutions and manufacturers, a celebration event took place in Geneva on 3 May 2007.

Q339. Pseudotumour like growth in hemophilic arthropathy is commonly seen in

A) Hamstring muscle

B) Gastrocnemius

C) Iliopsoas

D) Quadriceps femoris

answer is choice c :: iliopsoas

intramuscular Hemorrhage: If bleeding into deep muscle masses remains untreated, it may result in adjacent

joint contractures and pressure neuropathy, with the extent of damage depending upon the extent of bleeding and the containme

provided by the various fascial compartments. Small hematomas in large muscles are generally reabsorbed without complications;

however, similar hematomas in tight fascial compartments may cause significant ischemic myopathy and neuropathy .

The iliopsoas is one large muscle in which extensive hemorrhage may occur but produce minimal symptoms.

Page 137: DocumentQ1

Emile-Weil described it as the most common site of intramuscular hemorrhage in hemophilia .Significant clinical features include su

severe pain in the groin accompanied by marked flexion deformity at the hip, followed by the formation of a mass in the iliac fossa.

Generally, there is no history of trauma.Bleeding may be severe, extending into the adjacent retroperitoneal structures; a 4-g drop in

hemoglobin is not unusual. As pressure increases, thefemoral nerve may be compressed against the inguinal ligament, resulting in

neuropathy.

Large hematomas with progressive enlargement can secondarily affect the bone by pressure necrosis. While we havenot seen a ca

simultaneous bilateral iliopsoas hemorrhage, it is not unusual for each of the two sides to bleed during separate episodes.Any

hemorrhage within the psoas or iliopsoas muscle is likely to cause asymmetric enlargement.

Q340. Thurston- holland sign is seen in ;

1. oblique fracture of lower 1/3 of humerus.

2. reverse oblique fracture of intertrochanterisc femur.

3. salter-harris type2 fracture.

a. coronal fracture of femoral condyles.

Q341. Turn buckle splint is for ;;

1. brachual plexus palsy.

2. V.I.C (ans)

3. ulnar n palsy

4. radial n palsy

Facts about Compartment Syndrome :-

- Most common fracture causing Compartment syndrome/VIC = Supracondylar #

- Most common compartment = Deep flexor fore-arm

- Most common muscle involved = FDP

- Most common nerve involved = AIN

- First symptom = pain out of proportion to injury

- First sign = Pain on passive flexion

Splint for VIC = Turn Buckle Splint

Surgery = Maxpage Muscle slide

Turn Buckle Cast = Scoliosis

Q342. Site for first order bone grafting?

a. pelvis

b. tibial metaphysis

c. medial malleolus

d. femoral condyle

e. greater trochanter

Q343. PRAYERS SIGN IS OBSERVED IN PATIENTS OF ::

1. diabetes mellitus.

2. obstructive jaundice.

3. asthma.

4. mitral stenosis.

Seen in Diabetic Cheiroathropathy.

Prayer sign in a patient with long-standing type II diabetes mellitus. ...and neurogenic arthropathies, such as tabes dorsalis,

syringomyelia

Q344. Deformities due to malignancy classified by?

Page 138: DocumentQ1

(1) Ollier

(2) MESS

(3) Massada

(4) Tscherne's

scherne Soft Tissue Classification

Grade 0-Minimal soft tissue damage, indirect violence, simple # pattern

Grade 1-Superficial abrasion of contusion caused by pressure from within, mild to mod. # pattern

Grade 2-Deep contaminated abrasions assoc. with localized skin or muscle contusion, impending compartment syndrome, severe #

Grade 3-Extensive skin contusion or crush, underlying severe muscle, decompensated compartment syndrome, assoc. major vascular

injury, severe #

Q345. Enchondroma's seen in?

(1) MaCune Albright Syndrome

(2) Maffucie's syndrome

(3) Ollier's syndrome

(4) Goltz syndrome

(5) Terry's Syndrome

Maffucci syndrome is a rare genetic disorder that affects both males and females. Maffucci syndrome is characterized by benign

enlargements of cartilage (enchondromas); bone deformities; and dark, irregularly shaped hemangiomas. No racial or sexual predilection

is apparent in Maffucci syndrome. No familial pattern of inheritance has been shown, but Maffucci syndrome manifests early in life,

usually around age 4-5 years, with 25% of cases being congenital. Maffucci syndrome appears to develop from mesodermal dysplasia

early in life. Patients apparently are of average intelligence, and no associated mental or psychiatric abnormalities seem to be present.

Ollier disease

Enchondromas are common intraosseous, usually benign cartilaginous tumors, that develop in close proximity to growth plate ca

When multiple enchondromas are present, the condition is called enchondromatosis also known as Ollier disease (WHO terminology). The

estimated prevalence of Ollier disease is 1/100,000. Clinical manifestations often appear in the first decade of life. Ollier disease is

characterized by an asymmetric distribution of cartilage lesions and these can be extremely variable (in terms of size, number, location,

evolution of enchondromas, age of onset and of diagnosis, requirement for surgery). Clinical problems caused by enchondromas

skeletal deformities, limb-length discrepancy, and the potential risk for malignant change to chondrosarcoma. The condition in which

multiple enchondromatosis is associated with soft tissue hemangiomas is known as Maffucci syndrome. Until now both Ollier disease and

Maffucci syndrome have only occurred in isolated patients and not familial. It remains uncertain whether the disorder is caused by a

single gene defect or by combinations of (germ-line and/or somatic) mutations. The diagnosis is based on clinical and conventional

radiological evaluations. Histological analysis has a limited role and is mainly used if malignancy is suspected. There is no medical

treatment for enchondromatosis. Surgery is indicated in case of complications (pathological fractures, growth defect, malignant

transformation). The prognosis for Ollier disease is difficult to assess. As is generally the case, forms with an early onset appear more

severe. Enchondromas in Ollier disease present a risk of malignant transformation of enchondromas into chondrosarcomas.

Q346. Not a stage of Fracture healing?

(1) Stage of Hematome

(2) Stage of granulation

(3) Stage of Remodelling

(4) Stage of Modelling

(5) Stage of Consolidation

Q347. Non-union never seen in?

(1) #clavicle

(2) Colle's #

(3) Inter-trochanteric #

(4) Tibial #

(5) SupraCondylar #

Page 139: DocumentQ1

Q348. Most common muscle involved in Myositis ossfican's?

(1) Anconeus

(2) Brachialis

(3) Brachioradialis

(4) Biceps Brachii

Q349. Most common joing replacement affected by Myositis Ossificans?

(1) Shoulder

(2) Elbow

(3) Hip

(4) Knee

Q350. Most common cause of death after THR?

(1) MI

(2) SEPSIS

(3) PULMONARY EMBOLISM

(4) OTHERS

Q351. Milkman's # seen in?

(1) Pagets disease

(2) Osteomalacia

(3) Osteopetrosis

(4) Osteosarcoma

Q352. Most common joint in Charcots joint?

(1) Subtalar

(2) Mid-tarsal

(3) Inter-tarsal

(4) Phalngeal

Q353. Drop attacks are seen in a/e

a. Dermatomyositis

b. Polymyositis

c. ALS

d. Inclusion BM

e. Arnold chiari

Q354. Most specific test for Dengue?

Q355. Most specific test for Dengue?

Q356. Bombay blood group?

Q357. Cystic fibrosis show s fecal elastase-1 positivity. T/F

Q358. Babes nodules are seen in Rabies. T/F

Histopathologic evidence of rabies encephalomyelitis (inflammation) in brain tissue and meninges

Mononuclear infiltration

Perivascular cuffing of lymphocytes or polymorphonuclear cells or inflammation around a blood vessel CDC

Lymphocytic foci

Babes nodules consisting of glial cells

Negri bodies

Page 140: DocumentQ1

Q359. Metabolic Syndrome criteria involves raised LDL levels. T/F

Q360. Neurexin mutation is seen in Autism. T/F

Posted: Fri Aug 27, 2010 1:24 pm Post subject:

Q353. Drop attacks are seen in a/e

a. Dermatomyositis

b. Polymyositis

c. ALS

d. Inclusion BM

e. Arnold chiari

Drop attacks are sudden spontaneous falls while standing or walking, with complete recovery in seconds or minutes. There is usually no

recognized loss of consciousness and the event is remembered. It is a symptom, not a diagnosis, and it can have diverse causes.

Sheldon (1960) reported that drop accounts accounted for about 1/4 of 500 consecutive falls in older patients. This estimate seems a bit

high to us. Nevertheless, drop attacks are a very serious problem.

DIAGNOSIS OF DROP ATTACKS

1.Drop attacks due to cardiac disorders are similar to brief fainting episodes. They are best diagnosed with "ambulatory event

monitoring".

2.Drop attacks due to seizures and related problems are diagnosed with EEG test and its variants.

3.Drop attacks due to Meniere's disease, also called "Otolithic Crises of Tumarkin", are diagnosed with an audiogram and ENG. There are

a variety of other tests also

4.Drop attacks from. SCD is diagnosed with a VEMP and a CT scan of the temporal bone (if the VEMP is abnormal).

5.Drop attacks due to psychological problems are very difficult to confirm but sometimes this can be established via inpatient EEG

monitoring.

6.Cataplexy (a variant of narcolepsy) can present as drop attacks. Narcolepsy is diagnosed with EEG.

CAUSES

Proximal weakness of the legs

- some causes include muscular dystrophy, neurogenic atrophy, polymyositis, inclusion body myositis, myasthenia gravis and spin

ischemic attacks

- Duchenne's muscular dystrophy is the most common muscle dystrophy to cause weakness of the hip and knee extensors, which are

important for maintaining an erect posture

- rarely, middle-aged patients may develop drop attacks due to isolated quadriceps weakness, secondary to diseases like polymyositis,

inclusion body myositis and neurogenic atrophy, and the legs may suddenly give way without warning to cause falls

- intermittent spinal ischemia can produce sudden attacks of transient paraparesis or falls, and causes include syphilitic arteritis,

arteriosclerosis of the distal aorta, adhesive arcachnoiditis, spinal compression due to spinal stenosis or disc herniation; the attacks

often precipitated by exercise and relieved by rest

Neurodegenerative diseases

- patients with Parkinson's disease often fall due to defective postural reflexes, sudden freezing, and uncoordinated turning; their

stooped flexed posture predisposes them to falling forward because their centre of gravity is displaced forward when standing or walking;

drug-induced dyskinesias and orthostatic hypotension can also predispose them to falls

Page 141: DocumentQ1

- patients with Parkinon's plus syndromes (other akinetic-rigid disorders) are more prone to fall

- falling is the most frequent initial presenting symptom in progressive supranuclear palsy, and PSP patients tend to extend their necks

and fall backwards; stiff legs + wide-based gait + absent postural reflexes + characteristic down-gaze palsy predisposes PSP patients to

frequent falls

- other neurodegenerative diseases associated with frequent falls include corticobasal degeneration, Huntingdon's chorea, multiple

system atrophy and Alzheimer's disease

Transient ischemic attacks

- the patient with vertebro-basilar artery ischemia can present with a sudden loss of tone in their legs, causing their knees to buckle =>

the patient falls powerlessly without protective arm movements; there may be no precipitating head or neck movements or loss

consciousness, and the patient can ususally get up and walk immediately or within one minute; usually other symptoms of brainstem

ischemia are present (vertigo, diplopia, bluured vision, sensory changes, motor weakness and variable LOC), which suggest the

diagnosis

Epilepsy

- drop attacks can be due to sudden muscle contractions associated with generalized convulsive epilepsy, or a sudden loss of postural

tone due to atonic epilepsy or epileptic negative myoclonus

- the Lennox-Gastaut syndrome consists of a variable mixture of myoclonic, atonic, tonic and abscence seizures; these patients have a

past history of brain damage and mental retardation, and seizures occur between 6 months and 7 years

- atonic epileptic attacks in children can occur when standing or sitting or lying, and there is no aura or postictal state of confusion;

sudden drop attacks can cause the patient to collapse like an accordion with triple flexion of the legs and no involvement of the arms =>

the patient can usually get up immediately after an attack

Various myoclonic syndromes

- mycoclonus is defined as a sudden brief, shock-like muscle movement caused by a brief muscle contraction (positive myoclonus) or

inhibition of a muscle contraction (negative myoclonus)

- they can mimic idiopathic drop attacks, and the patient can fall suddenly with buckling of the legs

- many patients exhibit repetitive bouncing of the legs when they attempt to stand, due to repetitive negative myoclonus in the extensor

support muscles of the trunk and hips

Startle reactions

- pathological startle reactions (hyperekplexia) are characterized by an exaggerated motor response to unexpected auditory stimuli, and

less frequently to visual or somasthetic stimuli

- pathologic startle reactions consist of both brief myoclonic jerks and longer tonic spasms

- hyperekplexia may have a variety of causes eg. idiopathic, hereditary, or secondary to acquired brainstem diseases - post-anoxic

encephalopathy, viral or paraneoplastic brainstem encephalitis, multiple sclerosis, sarcoidosis or brainstem strokes

- myoclonic jerks occur frequently during the day, but also at night when the patient is asleep, and mainly affect the legs

- sterotyped tonic spasms consist of facial grimacing, flexion of the neck and trunk, flexion or extension of the hips and knees, and

characteristic abduction of the shoulders with flexion of the elbows => the patient falls forwards rigidly like a log, but sometimes falls

backwards; protective reflexes are impossible and injuries common; consciousness in maintained during the fall and recovery is fast

Page 142: DocumentQ1

Cataplexy and paroxysmal kinesigenic choreoathetosis

- cataplexy can cause sudden falls without LOC, but the patient cannot speak during the fall

- characteristic precipitating factors include laughter or a sudden emotional stimulus

- minor symptoms of cataplexy may consist of a brief drop of the jaw and sagging of the head, but severe attacks can cause loss of tone

in all the antigravity muscles => the patient falls powerlessly and loosely and remains immobile for many seconds (longer than in

hyperekplexia)

- attacks of paroxysmal kinesigenic choreoathetosis are usually precipitated by sudden leg movements during standing or running or

social embarrassment => the patient may suddenly lose control due to tonic spasms or an involuntary choreoathetotic muscle mo

and fall like a log

Focal structural lesions

- mass lesions around the foramen magnum can cause drop attacks (eg. odontoid process fractures, Arnold-Chiari type I malformations),

and drop attacks can be precipitated by neck movements, coughing or sneezing, or vertical head compression; the drop attack patient

presents with a stiff neck, spastic paraplegia. Lhermitte's sign and limb paresthesias

- other posterior fossa pathology can also cause drop attacks eg. posterior fossa arachnoid cysts, midline metastatic cerebellar tumors,

vermis hemangioblastomas, and obstructive hydrocephalus due to third ventricle cysts

Meniere's drop attacks (Tumarkin's otolithic crisis)

- seen in the early or late stages of Meniere's disease when there is a sudden change in endolymphatic pressure which causes end

stimulation with a reflex-like vestibulospinal loss of postural tone

- the patient suddenly loses control of balance and falls; the patient does not lose consciousness and he sometimes has the distinct

feeling of being pushed or thrown to the ground

(* a patient with Wallenberg's syndrome, due to a lateral medullary stroke, may have a similar transient sensation of being pulled or

pushed to the side, and the patient suddenly falls to the side without realizing that it is his active shift of the centre of gravity -

lateropulsion - that causes the momentary imbalance)

Idiopathic drop attacks

- proposed diagnostic criteria include:- "A fall without warning, not associated with loss of consciousness, not apparently due to any

malfunction of the legs, not induced by changes of posture or movement of the head, and not accompanied by vertigo or other c

sensation, and not associated with myoclonic jerks"

- usually appear between the ages 40 - 60 years

- patients only fall when standing or walking, and they usually buckle at the knees and fall forwards

- the attacks occur very suddenly and the patient cannot remember if he lost consciousness, or not; the patient can get up imme

unless accidental injury occurs (minor upper limb fractures and/or dislocations are common in the elderly)

- serial attacks are rare, and spontaneous remission is common (25 - 80%)

als often referred to as Lou Gehrig's disease, is a progressive neurodegenerative disease that attacks nerve cells in the brain and the

spinal cord.

Page 143: DocumentQ1

Early symptoms vary with each individual but usually include tripping, dropping things, abnormal fatigue of the arms and/or legs, slurred

speech, muscle cramps and twitches and/or uncontrollable periods of laughing or crying.

Q354. Most specific test for Dengue?

answer pnrt

NS-1 antigen tests for the dengue antigen whereas the IgM capture Elisa — which is the only government approved test for dengue

tests for antibodies. ''NS-1 is the fastest and more accurate test for dengue virus. It takes three to four days for the antibodies to form

and by the time you test it, the disease has either subsided or become serious. But with NS-1 we can test the antigen and pick up the

disease earlier

CDC 2010

Dengue can be diagnosed by

isolation of the virus,

by serological tests, or

by molecular methods.

Diagnosis of acute (on-going) or recent dengue infection can be established by testing serum samples during the first 5 days of

symptoms and/or early convalescent phase (more than 5 days of symptoms).

Acute infection with dengue virus is confirmed when the virus is isolated from serum or autopsy tissue specimens, or the specific dengue

virus genome is identified by reverse transcription-polymerase chain reaction (RT-PCR) from serum or plasma, cerebrospinal fluid, or

autopsy tissue specimens during an acute febrile illness.

Patients who have IgM antibodies to dengue detected in their serum specimen via an IgM antibody capture enzyme-linked

immunosorbent assay (MAC-ELISA) and had either

1.) A negative RT-PCR result in the acute phase specimen or

2.) Did not submit an acute phase specimen, are classified as having a recent probable dengue infection.

This is due to the fact that IgM antibodies for dengue may remain elevated for 2 to 3 months after the illness. The elevated IgM observed

in a sample could be the result of an infection that occurred 2 to 3 months ago. In addition, there is cross reactivity with other

flaviviruses including

West Nile virus (WNV),

St. Louis encephalitis virus (SLE),

Japanese encephalitis virus (JEV) and

yellow fever virus (YFV).

So if a patient with suspected dengue infection submits a late acute phase specimen that is negative (e.g., by RT-PCR and MAC-

and they do not submit a convalescent specimen, they are classified as a laboratory-indeterminate case.

MAC-ELISA has become an important tool for routine dengue diagnosis, MAC-ELISA has a sensitivity and specificity of approximately

90% and [snip]%, respectively but only when used five or more days after onset of fever (i.e., in convalescent phase)

. Different formats such as capture ELISA, capture ultramicroELISA, dot-ELISA, AuBioDOT IgM capture and dipsticks have been

developed. Serums, blood on filter paper, and saliva (but not urine) are useful for IgM detection if samples are taken in convalescent

phase of illness

algorithm:::

a. PCR

$ DENV can be detected in the blood (serum) from patients for approximately the first 5 days of symptoms.

$Real time RT-PCR assays have been developed and automated; but none of these tests are yet commercially available. Because

Page 144: DocumentQ1

antibodies are detected later, RT-PCR has become a primary tool to detect virus early in the course of illness. Current tests are between

80-90% sensitive, and more that 95% specific.

$ A positive PCR result is a definite proof of current infection and it usually confirms the infecting serotype as well. However, a negative

result is interpreted as “indeterminate”.

b.MAC ELISA

IgM antibody capture ELISA (MAC-ELISA) format is most commonly employed in diagnostic laboratories and commercial available

diagnostic kits. The assay is based on capturing human IgM antibodies on a microtiter plate using anti-human-IgM antibody followed by

the addition of dengue virus specific antigen (DENV1-4)

antigens used for this assay are derived from the envelope protein of the virus

limitation of this testing is the cross reactivity between other circulating flaviviruses

c.IgG ELISA

The IgG ELISA used for the detection of a past dengue infection utilizes the same viral antigens as the MAC ELISA. This assay correlates

with the hemagglutination assay (HI) previously used.

IgG ELISA lacks specificity within the flavivirus serocomplex groups

differentiate primary n secondary

Primary versus secondary dengue infection can be determined using a simple algorithm. Samples with a negative IgG in the acut

and a positive IgG in the convalescent phase of the infection are primary dengue infections

Samples with a positive IgG in the acute phase and a 4 fold rise in IgG titer in the convalescent phase (with at least a 7 day interval

between the two samples) is a secondary dengue infection.

d. NS1 ELISA

The non-structural protein 1 (NS1) of the dengue viral genome has been shown to be useful as a tool for the diagnosis of acute dengue

infections. Dengue NS1 antigen has been detected in the serum of DENV infected patients as early as 1 day post onset of sympt

(DPO), and up to 18 DPO

e. PRNT

Plaque Reduction and Neutralization Test (PRNT) and the microneutralization PRNT can be used when a serological specific diagnostic is

required, as this assay is the most specific serological tool for the determination of dengue antibodies

The PRNT test is used to determine the infecting serotype in convalescent sera

microneutralization assay is based on the same principle however instead of counting the number of plaques per well the assay

colorimetric measurement of the virus induced cell lysis to determine the end-point dilution. This assay was developed to utilize less

reagents and for high throughput purposes for larger number of samples for testing.

Q355. 355:: Stool Osmolal gap?

Stool Osmolal Gap = Stool Osm - (2 * (Na + K) )

The 290 is the value of the stool osmolality

Page 145: DocumentQ1

Causes of osmotic diarrhea include:{ sog >100:: i e high}

Bile salt deficiency

Pancreatic insufficiency

Celiac/Tropical Sprue

Whipple's Dz

Intestinal Lymphoma

Medications

Lactose Intolerance

Laxative abuse (depending on the type of laxative)

Causes of secretory diarrhea include:{sog <100::: i.e low}

Laxative abuse (depending on the type of laxative)

Hormonal, Endocrine Tumors

Q356. Bombay blood group?

Bombay blood group is a type of blood group which is present in people who are of O+ve blood group. O+ve and Bombay O+ve blood

groups are different. Bombay O+ve people are very less in population. There is no A, B, H antigen in the blood of these people. The H

antigen present in O+ve group people is not there in the Bombay O+ve ‘s. Instead of H antigen they have H antibody, this is what

doctors are saying. People, who are having Bombay O + ve blood group, can donate only to Bombay O+ve people and can only acce

from Bombay O+ve blood group people

detection:::There is a serum grouping also called as reverse grouping (accurate test of a person’s ABO group i.e. O group). If this test is

conducting then only we can detect the presence of H antibody, which indicate Bombay O+ve blood group. This test is conducted

the help of a reagent called H – Lectin. We can find these people mostly in Maharashtra and some places of Karnataka which is the

boarder of Maharashtra.

Dr. Bhende discovered Bombay O+ve blood group in 1952 at Mumbai. This is the reason why this blood group got the name Bombay

O+ve blood group

Posted: Fri Aug 27, 2010 1:29 pm Post subject:

Q360. Neurexin mutation is seen in Autism.

T/F

it is seen in autism spectrum diseases-true

Cell adhesion represents the most direct way of coordinating synaptic connectivity in the brain. Recent evidence highlights the

importance of a trans-synaptic interaction between

a. postsynaptic neuroligins and

b. presynaptic neurexins.

These transmembrane molecules bind each other extracellularly to promote adhesion between dendrites and axons. This signals t

recruitment of presynaptic and postsynaptic molecules to form a functional synapse. Remarkably, neuroligins alone can induce the

formation of fully functional presynaptic terminals in contacting axons. Conversely, neurexins alone can induce postsynaptic

differentiation and clustering of receptors in dendrites.

Page 146: DocumentQ1

Therefore, the neuroligin–neurexin interaction has the unique ability to act as a bi-directional trigger of synapse formation.

Many studies have supported a genetic etiology for autism. Mutations in two X-linked genes encoding neuroligins NLGN3 and NLGN4 in

siblings with autism-spectrum disorders.

These mutations affect cell-adhesion molecules localized at the synapse and suggest that a defect of synaptogenesis may predispose to

autism.

Q361. which of the following CANNOT cause pseudo tumor cerebri::

a. Iron def

b. Polio

c. GBS

d. Chronic Otitis media

e. Oudated Salicylic acid

Q362. Blood and mucus in stool in a/e

a. Yersinia

b. Camphylobacter

c. Vibrio cholera

d. vibrio parahemolyticus

e. EIEC

answer vibrio cholera and v .parahemolyticus{c & d}

Vibrio parahaemolyticus: Although it is a common worldwide pathogen, in the United States, V. parahaemolyticus infection is restricted

geographically to the Atlantic and Gulf coasts. The diarrhea is profuse and watery, and blood is not commonly present in the stool

others are example of enteroinvasive bacteria

Q363. NORMAL protein CSF levels at 4 weeks infant maybe as high as::

a. 200 mg%

b. 150 mg%

c. 100mg%

d. 70mg%

e. 25mg%

The cell and protein content of the cerebrospinal fluid (CSF) and the CSF/serum ratio of glucose, sodium, potassium, calcium, phosphate

and chloride were investigated in 71 children without cerebromeningeal illness and aged 0-4 weeks, 5-8 weeks, 3-6 months, 7-12

months, 1-6 years and 7-14 years. The protein content of the CSF was 730 +/- 146 (530-950) mg/l for the age group 0-4 weeks, and

530 +/- 221 (270-950) mg/l for the age group 5-8 weeks

Q364. Agranulocytosis not seen in::

a. Captopril

b. Phenylbutazone

c. Gold salts

d. Cyclosporine

Although all the drugs mentioned can cause AGRANULOCYTOSIS (YES EVEN CAPTOPRIL INDUCED AGRANULOCYTOSIS has been

reported) but still it's very rare with captopril as compared with others...

So Ans = Captopril

Q365. True about CPEO a/e::

a. ragged red fibers

Page 147: DocumentQ1

b. single MC sign of mitochon myopathy

c. diplopia in all directions

d. ptosis may be present

Q366. A 30 year old para 6 delivers vaginally following normal labour with spontaneous delivery of intact placenta.Excesive bleeding

continues,despite manual exploration,bimanual massage,intravenous oxytocin and I.V methergin..which one of the following will be next

step in the management of thi spatient:-

a) Packing of the uterus

b) Immediate hysterectomy

c) Bilateral internal iliac ligation

d) IM injn of PGF 2 alpha

Q367. Platelet Serotonin is superior to 5HIAA levels in urine for Foregut carcinoid. T/F

Q368. Vit B12 massive dose can overcome perniciopus anemia. T/F

false

A 2003 study found that oral and sublingual B12 were absorbed equally well in a group of patients with very low B12. In this study, 22%

of the subjects that agreed to undergo the test (5 of 23), had abnormal Schilling tests, but showed no difference in treatment level

the other subjects. When oral tablets are used to treat PA, higher-than-normal doses may be needed.The efficacy of using high dose B12

tablets to treat ordinary PA (i.e. anemia due to atrophic gastritis) is well established. Oral supplementation allows B12 to be absorbed in

places other than the terminal ileum (where B12 absorption usually takes place). A 2006 study found that oral B12 repletion has the

potential to be as effective as injections.

Q369. Bulimia Nervosa is associated with anti-social behaviour. T/F

Q370. Haller cells can be confused for Agger nasi Cells T/F

Posted: Fri Aug 27, 2010 1:29 pm Post subject:

Management after Delivery of Placenta Ref:-Williams Obstertrics 22nd Ed

The fundus should always be palpated following placental delivery to make certain that the uterus is well contracted.

If it is not firm, vigorous fundal massage is indicated. Most often, 20 U of oxytocin in 1000 mL of lactated Ringer or

normal saline proves effective when administered intravenously at approximately 10 mL/min (200 mU of oxytocin per

minute) simultaneously with effective uterine massage. Oxytocin should never be given as an undiluted bolus dose,

because serious hypotension or cardiac arrhythmias may occur

Uterine Atony

Failure of the uterus to contract properly following delivery is a common cause of obstetrical hemorrhage. In many

women, uterine atony can at least be suspected well in advance of delivery (see Table 35–5). For example, the

overdistended uterus is prone to be hypotonic after delivery. Thus, the woman with a large fetus, multiple fetuses, or

hydramnios is prone to hemorrhage from uterine atony. The woman whose labor is characterized by uterine activity

that is either remarkably vigorous or barely effective is also likely to bleed excessively from uterine atony after

delivery. Similarly, labor either initiated or augmented with oxytocin is more likely to be followed by postdelivery

uterine atony and hemorrhage.

The woman of high parity may be at increased risk for uterine atony. Fuchs and colleagues (1985) described the

outcomes of nearly 5800 women para 7 or greater. They reported that the 2.7 percent incidence of postpartum

hemorrhage in these women was increased fourfold compared with that of the general obstetrical population.

Babinszki and colleagues (1999) reported the incidence of postpartum hemorrhage to be 0.3 percent in women of low

parity, but it was 1.9 percent in those para 4 or greater. Another risk is if the woman has previously suffered

postpartum hemorrhage. Finally, mismanagement of the third stage of labor involves an attempt to hasten delivery of

Page 148: DocumentQ1

the placenta short of manual removal. Constant kneading and squeezing of the uterus that already is contracted likely

impedes the physiological mechanism of placental detachment, causing incomplete placental separation and increased

blood loss.

Ergot Derivatives

If oxytocin given by rapid infusion as described in Management after Delivery of Placenta does not prove effective,

some clinicians administer intramuscular methylergonovine (0.2 mg). This may stimulate the uterus to contract

sufficiently to control hemorrhage. Any superior therapeutic effects of ergot derivatives over oxytocin are speculative,

and if these agents are intravenously administered, they may cause dangerous hypertension, especially in women

with preeclampsia.

Prostaglandins

The 15-methyl derivative of prostaglandin F2 (carboprost tromethamine) was approved in the mid-1980s by the U.S.

Food and Drug Administration (FDA) for treatment of uterine atony. The initial recommended dose is 250 g (0.25 mg)

given intramuscularly, and this is repeated if necessary at 15- to 90-minute intervals up to a maximum of eight

doses. Oleen and Mariano (1990) studied use of carboprost for postpartum hemorrhage at 12 cooperating obstetrical

units. Arrest of bleeding was considered successful in 208 (88 percent) of 237 women treated. Another 17 women (7

percent) required other oxytocics for control of hemorrhage. The remaining 12 women (5 percent) required surgical

intervention.

Carboprost is associated with side effects in about 20 percent of women (Oleen and Mariano, 1990). In descending

order of frequency, these include diarrhea, hypertension, vomiting, fever, flushing, and tachycardia. We have

encountered serious hypertension in a few women so treated. In addition, Hankins and colleagues (1988) observed

that intramuscular carboprost was followed by arterial oxygen desaturation that averaged 10 percent and developed

within 15 minutes. They concluded that this was due to pulmonary airway and vascular constriction.

Rectally administered prostaglandin E2 20-mg suppositories have been used for uterine atony but not studied in

clinical trials. A few reports have suggested that misoprostol (Cytotec), a synthetic prostaglandin E1 analogue, may

be effective for the treatment of uterine atony (Abdel-Aleem and associates, 2001; O'Brien and colleagues, 1998). In

the larger of these uncontrolled studies, misoprostol (1000 g) given rectally was effective in 16 of 18 women

unresponsive to usual oxytocics. The mean response time was 1.4 minutes. Misoprostol has also been studied as a

potential prophylactic treatment for preventing postpartum hemorrhage. Based on their study of 325 women,

Gerstenfeld and Wing (2001) concluded that rectal misoprostol (400 g) was no more effective than intravenous

oxytocin in preventing postpartum hemorrhage. Moreover, Villar and co-workers (2002) found in their systematic

review that oxytocin and ergot preparations administered during the third stage of labor were more effective than

misoprostol for the prevention of postpartum hemorrhage.

Bleeding Unresponsive to Oxytocics

Continued bleeding after multiple administrations of oxytocics may be from unrecognized genital tract lacerations,

including in some cases uterine rupture. Thus, if bleeding persists, no time should be lost in haphazard efforts to

control hemorrhage, but the following management should be initiated immediately:

1. Use bimanual uterine compression (Fig. 35–16). The technique consists simply of massage of the posterior aspect

of the uterus with the abdominal hand and massage through the vagina of the anterior uterine aspect with the other

fist. This procedure controls most hemorrhage.

2. Obtain help!

3. Add a second large-bore intravenous catheter so that crystalloid with oxytocin may be continued at the same time

blood is given.

4. Begin blood transfusions. The blood group of every obstetrical patient should be known, if possible, before labor

and an indirect Coombs test done to detect erythrocyte antibodies. If the latter is negative, then crossmatching of

blood is not necessary (see Type and Screen Versus Crossmatch). In an extreme emergency, type O D-negative

"universal donor" blood is given.

5. Explore the uterine cavity manually for retained placental fragments or lacerations.

6. Thoroughly inspect the cervix and vagina after adequate exposure.

7. Insert a Foley catheter to monitor urine output, which is a good measure of renal perfusion.

Resuscitation is then carried out as described subsequently in Resuscitation and Acute Management. Blood transfusion

should be considered in any woman with postpartum hemorrhage in whom abdominal uterine massage and oxytocic

agents fail to control the bleeding. With transfusion and simultaneous manual uterine compression and intravenous

oxytocin, additional measures are rarely required. Intractable atony may mandate hysterectomy as a lifesaving

Page 149: DocumentQ1

measure (see Chap. 25, Indications). Alternatively, uterine artery ligation (Fig. 35–17), internal iliac artery ligation

(Figs. 35–18 and 35–19), uterine compression sutures (Fig. 35–20), or angiographic embolization as described in

Angiographic Embolization may prove successful.

Internal Iliac Artery Ligation

Ligation of the internal iliac arteries at times reduces the hemorrhage appreciably (Allahbadia, 1993; Clark and

colleagues, 1985). However, the procedure may be technically difficult and is successful in less than half of the

patients in whom it is attempted (American College of Obstetricians and Gynecologists, 1998). With adequate

exposure, ligation is accomplished by opening the peritoneum over the common iliac artery and dissecting down to

the bifurcation of the external and internal iliac arteries (see Figs. 35–18 and 35–19). The areolar sheath covering the

internal iliac artery is incised longitudinally, and a right-angle clamp is carefully passed just beneath the artery. Care

must be taken not to perforate contiguous large veins, especially the internal iliac vein. Suture, usually

nonabsorbable, is then inserted into the open clamp, the jaws are locked, the suture is carried around the vessel, and

the vessel is securely ligated. Pulsations in the external iliac artery, if present before tying the ligature, should be

present afterward as well. If not, pulsations must be identified after arterial hypotension has been successfully treated

to ensure that the blood flow through the external iliac vessel has not been compromised by the ligature.

The most important mechanism of action with internal iliac artery ligation is an 85-percent reduction in pulse pressure

in those arteries distal to the ligation (Burchell, 1968). This converts an arterial pressure system into one with

pressures approaching those in the venous circulation and more amenable to hemostasis via simple clot formation.

Bilateral ligation of these arteries does not appear to interfere with subsequent reproduction. Nizard and associates

(2003) documented 21 pregnancies in 17 women after bilateral internal iliac artery ligation including three abortions,

three miscarriages, two ectopic pregnancies, and 13 normal pregnancies.

Uterine Compression Sutures

In 1997, B-Lynch and colleagues described a surgical technique performed in five women with severe postpartum

hemorrhage in which a pair of vertical brace #2 chromic sutures were secured around the uterus, giving the

appearance of suspenders, to compress together the anterior and posterior walls (see Fig. 35–20). A simpler

modification of the technique has been described by Hayman and associates (2002). We have found this suture to be

effective in some cases, however, published experience with these techniques remains limited.

Uterine Packing

This technique should be considered in women with refractory postpartum hemorrhage related to uterine atony who

wish to preserve fertility. Popular during the first half of the 20th century, uterine packing subsequently fell out of

favor because of concerns about concealed bleeding and infection (Hsu and co-workers, 2003). Newer techniques,

however, have allayed some of these concerns (Roman and Rebarber, 2003). In one technique, the tip of a no. 24F

Foley catheter with a 30-mL balloon is guided into the uterine cavity and filled with 60 to 80 mL of saline. The open

tip permits continuous drainage from the uterus. If bleeding subsides, the catheter is typically removed after 12 to 24

hours (Roman and Rebarber, 2003). Alternatively, the uterus or pelvis may be packed directly with gauze (Gilstrap,

2002). After hysterectomy, another technique, the umbrella pack, can be constructed of a sterile x-ray cassette bag,

filled with gauze rolls knotted together, providing enough volume to fill the pelvis (Howard and co-workers, 2002).

The pack is introduced transabdominally with the stalk exiting the vagina. Mild traction is applied by tying the stalk to

a one-liter intravenous fluid bag and hanging the bag over the foot of the bed. An indwelling urinary catheter is used

to monitor urine output and to prevent urinary obstruction. Placement of percutaneous pelvic drains should be

considered to monitor ongoing bleeding within the peritoneal cavity (Dildy, 2002). Broad-spectrum antimicrobials

should be administered, and the umbrella pack is removed vaginally after 24 hours.

Posted: Sat Aug 28, 2010 12:29 pm Post subject:

Q.372 A hemodynamically stable nullliparous patient with ectopic pregnancy has adnexal mass of 2.5 X 3 cms and beta hCG titre

miu/ml. What modality treatment is suitable for her:

a) Conservative management

b) Medical management

c) Laparoscopic surgery

Page 150: DocumentQ1

d) Laparotomy

Q. 373 What is the treatment of choice of unruptured tubal pregnancy with serum beta hcg 2000 iu/l

a) Single Dose methotrexate

b)Variable Dose

c) Expectant Treatment

d) Laparoscopic Salpingostomy

Q374. UROTENSIN-2 associated with a/e::

a. Diabetes Mellitus

b. Palosuran-antagonist

c. Renal failure

d. Conivaptan-commercial name

e. Hypertension

Q. 375. Human Colustrum is rich in Lactose T/F

Q376. Phenylephrine has little effect on HR of Transplanted heart. T/F

Q377. Sumatriptan suppresses both pain and vomiting in migraine like ergotamine. T/F

Posted: Mon Aug 30, 2010 11:24 am Post subject:

Q. 375. Human Colustrum is rich in Lactose T/F

TRUE

Table 25–10 Composition of Colostrum and Milk.*

Units are weight per decilitre(per 100gm)

1 ounce = 28.3 gm

Component Human Colostrum Human Milk Cows' Milk

Water, g . . . 88 88

Lactose, g 5.3 6.8 5.0

Protein, g 2.7 1.2 3.3

Casein: lactalbumin ratio. . . 1:2 3:1

Fat, g 2.9 3.8 3.7

Linoleic acid . . . 8.3% of fat 1.6% of fat

Sodium, mg 92 15 58

Potassium, mg 55 55 138

Chloride, mg 117 43 103

Calcium, mg 31 33 125

Magnesium, mg 4 4 12

Phosphorus, mg 14 15 100

Iron, mg 0.09a 0.15a 0.10a

Vitamin A, g 89 53 34

Vitamin D, g . . . 0.03a 0.06a

Thiamine, g 15 16 42

Riboflavin, g 30 43 157

Nicotinic acid, g 75 172 85

Page 151: DocumentQ1

Ascorbic acid, mg 4.4a 4.3a 1.6a

a=square

Ref Ganong

Q:- Lactose and energy minimal in cow milk

Q:- ADV. of cow milk:- Less lactose,max proteins,good minerals,max calcium and phosphorus,max riboflavin and thiamine,less ascorbic

acid and vit A

Q:- Protein ,fat and calcium max in buffalo milk

Q:- Lactose and water max in Human Milk

Q:- Vit A max in Human Colostrum

Q:- Casein Lactalbumin Ratio:- Imp 1:2 IN Human Milk

Q:- Calcium: 1200 per litre or 120 per 100 gm in cow milk,210 IN BUFFALO

Q:- Cow Milk:- Adv is more VIT K

Q. 378 A 20 year old young female comes to a male gynaecologist for pelvic examination -

The consent for examination is :

a) Implied consent

b) Written. consent

c) Verbal consent

d) Informed consent

Q. 379 Who can commute a death sentence-(JIPMER 80,99)

a) High court

b) Supreme court

c) President

d) Any of the above

Q. 380. Dying declaration can be received by - (AI89)

a) Medical officer

b) Lawyer

c) Police officer

d) All

Q381. If a patient survives after having given dying declaration then it stands as - (Jipmer 04)

a) Corroborative evidence

b) Non valued

c) Valid for 48 hours

d) None

Q382. Law does not consider the following doctrine in a Charge of criminal negligence - (PGI 93, AP 91)

a) Vicarious laibility

b) Contributory negligence

c) Resipsa loquitur

d) Novus actus interveniens

Q383. imp fatty acid prsnt in brst milk which is imp for growth??

a. dha

b. palmitic acid

c. linoleic acid

d. linolenic acid

Docosa Hexanoic Acid - The natural sources of DHA include:

(1) Breast milk

Page 152: DocumentQ1

(2) Fish oil

Commercially it's manufacture is done from the microalgae - Crypthecodinium cohnii and another of the genus Schizochytrium.

In pure vegetarians and in individuals not eating sea food, DHA in body is derived from alfa-linolenic acid.

DHA is an imp. fatty acid in Sperm, Brain, Retina.

Synthesis of DHA from EPA = Spercher's Shunt.

DHA in breast milk = 0.07% to 1% with a mean of 0.34% and directly proportional to mother's diet in fish.

RDA of DHA for Pregnant and Lactating Women = 300 mg/day

Q384. in disc prolapse c5 -6 which of following nerve root pressed??

a. c5

b. c6

c. c7

d. c4

Posted: Tue Aug 31, 2010 12:28 pm Post subject:

Q385. The age of a 15 year old female is best determined by the radiograph of - (AIIMS 86, PGI 8 7)

a) Lower end of Radius and Ulna

b) Upper end of humerus

c) Upper end of Radius and Ulna

d) Xiphisternum

Ans is C

acc to parikh

in females

13-14 yrs -- Elbow

16-17 Yrs -- Wrist

for 16 yrs female ofcourse ans is lower end of radius and ulna...Im not sure about 15 yrs...

Both ages have been asked and these ages are imp as before 15 yrs sex with wife even with consent is rape. And below 16 yrs sex with

anyone even with consent is rape but not with wife.

Q 386. the operative procedure known as "microfracture" is done for ?

A. delayes union of femur

B. nonunion of tibia

C. loose bodies of ankle joint

D. osteochondral defect of femur

Q :387::m/n of smith # is?

A. ORIF

B. POP in pronation

C. CR with below elbow cast

D. POP in supination

Page 153: DocumentQ1

Q 388::in children, the displacement of fractured fragment in which plane is not remodeled?

A. angualtion

B. side to side displacement

C. rotation

D. shortening

Q 389::TOC for displaced fracture of radius and ulna in an adult is?

A. POP 8wks

B. closed reduction and cast bracing

C. ORIF--by plating

D. Im nailing

Q 390::pathological # seen in which stage of paget's?

a. phase of repair

b. vascular soft phase

c. phase of remission

d. phase of sarcomatous change

Typical radiologic appearance includes areas of focal bone resorption and formation and a disordered trabecular pattern .

* Bone size is expanded and the cortices are thickened. Trabeculae are coarse and widely separated. Density is decreased in the vascular

stage and increased in the sclerotic stage. Disease involvement is usually seen at one end of the bone (generally proximal).

* A flame-shaped osteolytic wedge may be seen(in vascular stage), as may bony deformities and frank pathologic fractures.

* Vertebrae may have a framed picture appearance. Acetabular involvement may have protrusion.

* Pagetic bone may also have small transverse lucencies or pseudofractures, typically on the convex surfaces of the weight-bearing long

bones. Bowing if the femur or tibia may also occur (as above) either with or without pseudofracture.

* Only 65% of the lesions seen on bone scan will be seen on x-rays.

Posted: Wed Sep 01, 2010 12:50 pm Post subject:

Q:: 391:: In NARP there is salt and pepper retina. T/F

Q:: 392:: Cannabinidol is used in Pharmacological Preconditioning T/F

Q:: 392:: Cannabinidol is used in Pharmacological Preconditioning T/F

Q:: 394:: To treat Morphine poisoning acidify the urine. T/F

Q:: 395:: H2O in Liquor amnii is recycled every 6 hrs T/F

answer:: false

water in amniotic fluid is completely changes and replaced by 3 hours.

as you quoted...decreasing amnionic fluid osmolality with advancing gestation.>>>and other point is 250mOsm/l indicates fetal

maturity.

396 To detemine the age of a 16 year old girl which of -

the following is to be rediographed - (PGI88)

a) Wrist

b) Knee

c) Ankle

d) Elbow

Page 154: DocumentQ1

397. Crural index is calcutated as-(PGI 80, AIIMS 82)

a) Length of Radius b) Length of tibia

c) Length of Humerus d) Length of ulna

398 .A girl of 10 years wilt have ---- permanent and --temporary teeth - (PGI 79, DNB 90)

a) 8,12 b)8,16

c) 12,12 d) 16,8

399. The age of a 15 year old female is best determined by the radiograph of - (AIIMS 86, PGI 8 7)

a) Lower end of Radius and Ulna --answer?? NOT SURE

b) Upper end of humerus

c) Upper end of Radius and Ulna

d) Xiphisternum

400. In case of malpractice punishment is given

by- (Calcutta 2K)

a) State medical council

b) MCI

C) IMA

d) High court

401. Which causes epiphyseal closure?

LH

FSH

Estrogen

Testosterone

402. Priapism occurs in - (AIIMS 06)

a) Snakebite

b) Rat poisoning .

c) Cantharide poisoning

d) Arsenic poisoning

403. Forceps are applied in all the following except :-

A. After coming head in breech presentation

B. Face presentation

C. Occipitoposterior

D. Brow presentation

404. Which is false regarding nutritional surveillance

a. It is a Diagnostic Method

b. Precise weight reading required

c. Done in all children less than 5 years

d. Done by a trained worker

405. About Delphi method ????

a. Formation of a team to undertake and monitor a Delphi on a given subject.

b. Selection of one or more panels to participate in the exercise. Customarily, the panelists are experts in the area to be investigated.

c. Development of the first round Delphi questionnaire

d. All are true

406. In full term neonates ,the ductus arteriosus closed by?

Cardiac output changes

Prostaglandins

Page 155: DocumentQ1

high pressure of o2

Low pressure of o2

Posted: Thu Sep 02, 2010 1:05 pm Post subject:

biosafety level 1 - exposure only to infectious agents that do not ordinarily cause human disease

biosafety level 2 - exposure to infectious agents that can cause disease in humans but whose potential for transmission is limited

biosafety level 3 - exposure to infectious agents that can be transmitted by the respiratory route and which can cause serious infection

biosafety level 4 - exposure to exotic infectious agents that pose a high risk of life-threatening disease and can be transmitted as an

aerosol and for which there is no vaccine or therapy

Posted: Mon Sep 06, 2010 1:07 pm Post subject:

The following are radiological features of sigmoid vovulus except:

a-Inverted U shaped bowel loop

b-liver overlap sign

c-Bird of prey sign

d-Cupola sign........................ans

The cupola sign (1) is seen at supine radiography as an arcuate lucency overlying the lower thoracic spine and projecting caudad to the

heart (Fig 1). The superior border is well defined and the inferior margin is poorly delineated. The term cupola is used to indicate the

inverted cupshaped

configuration of the lucency

Cupola sign refers to peritoneal air that is superior to the left lobe of the liver, having the appearance of a long, flat hat, hence the term

"cupola" meaning hat

Posted: Mon Sep 06, 2010 1:21 pm Post subject:

A 16-yrs-old girl with primary amenorrhea has poor secondary sex characteristics, short stature, normal appearing external genitalia,

and elevated serum gonadotropins. Which test would be most appropriate for confirming the diagnosis?

A. GnRH stimulation

B. Serum prolactin level

C. Serum TSH

D. Buccal smear blank

option D:: Turner presenting with high or "castrate" gonadotrophin levels.

this is most probably a case of turner's syndrome.a buccal smear shows absence of barr body.

A buccal smear is a test where cells are taken from the tongue. Cells are collected by scraping the tongue with a spatula. The cel

then placed on a slide and the sample is taken to the laboratory for evaluation. The cells are evaluated for the presence of Barr bodies (a

mass seen in a normal female sex chromosome). The buccal smear test can confirm whether the patient is a male or female.

Page 156: DocumentQ1